You are on page 1of 218

Esophagus

Scenario #1 - Pediatric Esophageal Burn


8 year old child drinks strong alkali solution accidentally.
Issues:
a.) E.R. management - airway management, lines, resuscitation.
b.) Role of antibiotics and/or steroids?
c.) Endoscopy - when and how far should scope be passed?
d.) Barium Swallow - if so, when?
e.) How is stricture managed?
f. ) If stricture persists and esophageal replacement is needed, what is the best conduit
and when should it be performed?

Answer:
Background - alkali is more destructive than acids, producing liqifactive necrosis which
almost ensures deep penetration. Liquid (as opposed to solid) allows contact to the
esophagus and stomach with concommitant damage which can extend to the adjacent
structures. Reflex antral and pyloric spasm cause the alkali to pool in the stomach and
reflux into the esophagus. May cause relatively little oropharyngeal burn andstill have
severe injury to the esophagus and stomach.

Initial management - Airway - oral intubation or tracheostomy is indicated if there is


laryngeal edema or laryngeal destruction from ingestion.
Breathing - 02
Circ. - oral ingestion of liquids should be held, as this has little
benefit. IV fluids should be instituted to correct hypovolemia.
Antibiotics - broad spectrum antibiotics should be started ASAP to
diminish risk of aspiration as well as direct bacterial invasion thru the damaged wall of
the esophagus.
?steroids - NO. Have been advocated to prevent stricture but
efficacy has not been established. May also mask sepsis and injury while slowing
healing.

Contrast Esophagram - best way to evaluate an esophageal perforation and should be


used if a perforation is suspected either initially or in followup. Initial esophagram
should be performed with water souble gastrograffin. However, dilute barium should
be used if perforation is suspected and not seen with water soluble contrast . Diagnosis
of perforation and identifying site of perforation is important to plan operative
intervention.

1
Esophagoscopy - should be used ASAP after admission to establish diagnosis and grade
the severity of the injury. Should use pediatric flexible endoscope and sedation to
minimize the risk of perforation. It is acceptable to advance the scope past the area of the
first burn, though conservative answer is to pass the scope to the level of the first burn.

Grading: First degree - mucosal hyperemia and edema.


Second degree - mucosal ulceration with vesicles and exudates,
pseudomembranes present.
Third Degree - deep ulceeration with charring and eschar formation :
severe edema obliterating lumen.
TREATMENT : FIRST DEGREE - no specific treatment for 24-48 hrs. May begin
liquids and treated as an outpatient.

SECOND DEGREE/THIRD DEGREE - must be more closely observed for


evidence of gastric or esophageal perforation during the acute phase of injury.
Problem - full thickness necrosis requires resection but it is very difficult to tell which
have full thickness necrosis. Therefore, those not showing free air but with persistent
back pain (mediastinitis), peritonitis, or acidosis (visceral injury) should be explored.

Operative intervention - should be done thru the abdomen to assess the abdominal organs.

Though only the distal esophagus is seen thru the abdomen, if an esophageal resection is
needed, a transhiatal esopagectomy should be performed. ( So prep the neck, chest
abdomen to axillary lines.) Rare that gastric resection is needed without esophageal
resection, but if this happens, the esophagus should be brought out as an esophagostomy.

- NONOPERATIVE TREATMENT - as a rule second and third degree burns will cause
stricture formation. Dilatation has been the traditional therapy. Important to wait 6-8
weeks for dilation to allow re-epithelialization to take place and decrease risk of
perforation.

Grading of Strictures:
mild - minimal narrowing
moderate - narrowing of lumen to half of its diameter.
severe - lumen greatly narrowed or pinhole.

Technique - thru existing gastrostomy, placed at time of admission to the hospital and
allowing 6-8 weeks to heal.
Goal - 32-38/40 Gfr. dilator for a 2- 3 yr. old child over a series of dilations for 2-3
weeks.
- goal is for 46 Fr. or larger for adults.

Stricture Perforation during dilatation - should be treated with esophagectomy and


replacement.

2
Esophageal Replacment - needed for strictures that are refractory to dilation after 6-12
months or strictures that cannot be adequately dilated should be replaced.
Choice of Replacement - stomach as conduit after transhiatal esophagectomy is best but
often scarring or contracture from original injury may necessitate replacement with
colon interposition. If this is performed the original esophagus should be removed
because of the small risk of esophageal CA with history of alkali ingestion.

Esophageal Stricture
I
I I
Mild or Moderate Severe
I I
Antegrade Dilation <------------------------------------------------ Retrograde
response I
I
Noresponse
Esoph. Replac

Scenario #2 - Achalasia
55 y.o. female presents with dysphagia and occasional regurgitation of food.
Issues:
a.) Appropriate workup for dysphagia. If scope for biopsy, decompress pouch first with
NGT
b.) What is diagnosis if “bird’s beak” found on UGI?
c.) How should UGI be performed?
d.) What are manometric finding’s consistent with achalasia? Esoph. body, LES,
contraction? What is normal LES? What are manometric findings of DES?
e.) Appropriate nonsurgical therapy? Approriate surgical therapy and how is this
performed?

ANSWER :
Background: failure of the distal esophageal sphincter to relax on deglutition. Additional
features include partial or complete loss of progressive peristalsis in the body of the
esophagus, elevation of the intraluminal esophageal pressure, and hypertension of the
LES at rest. Nonrelaxation of the sphincter and loss of peristalsis lead to dilated
esophagus.

3
Presentation: Initially complain of dysphagia, primarily of solid foods. As the organ
dilates into a reservoir organ, the sensation of dysphagia may pass. In this situation,
regurgitation and aspiration of esophageal contents is common.

Diagnosis : suggested by clinical presentation of dysphagia and regurgitation.

WORKUP:
UGI: “Bird’s Beak” - deformity of the esophagus at the level of the esophagogastric
junction. Varying degrees of esophageal dilatation may be present. Useful first test to
confirm diagnosis, identify anatomy and rule in achalasia so that esophagus can be
emptied prior of endoscopy and/or manometry.

Manometry - Achalasia
1.) absent or incomplete relaxation of the LES. (Normal LES = 15 cm.)
2.) complete absence of peristalsis in the smooth muscle portion of the esophagus - all
contractions are simultaneous - no stripping wave to empty the esophagus. ( The
nonrelaxing sphincter produces a functional distal obstruction which is worsened by the
lack of peristalsis of the esophagus).
3.) other abnormalities - high resting pressure in the LES.
- low amplitude of esophageal contraction. ( < 30 mmHg)

Differentiating from DES - early in the course of the disease, when eophageal
contractions may be of normal or even increased amplitude making differentiating the
two disorders difficult.
** - DES will have relaxation of the LES, but achalasia will not.

Manometry of DES - frequent simultaneous esophageal contractions. For diagnosis of


DES, must have minimal frequency of simultaneous contractions of more than 10-20%
of all esophageal contractions in response to wet swallow challenges. ( Simultaneous
contractions will occur in 25% of volunteers) Emphasis is on manometric criteria of
DES is the pattern rather than the force of esophageal contractions.

TREATMENT :

Pneumatic Dilatation - goal is to rapidly distend and actually disrupt the muscle of the
LES while the esophageal mucosa remains intact. Balloon is placed fluoroscopically at
the GE junction and rapidly inflated to diameter of 3-4 cm.
Complication - esophageal perforation rate of 3-4%.
Poor candidates for dilation - dilated esophagus
- vigorous alchalasia making passing balloon difficult.

Results - 75% achieve good result with no more than 2 sessions and never require further
therapy.

4
SURGICAL TREATMENT - recommended for those without success after 2 dilations
or those in whom dilation cannot be performed because the esophagus is tortuous or
dilated.

- 90% improvement with surgery vs. 70% with forceful dilation.

TECHNIQUE - Preop Prep - esophageal dilation is associated with retention esophagitis


and submucosal inflammation which will make the dissection difficult. It is necessary to
decompress the esophagus with NG sump preoperatively to allow the retention
esophagitis to resolve. Esophagoscopy is important to rule out a tumor at the GE
junction which is causing a” pseudoachalasia” or a tumor in the middle third of the
esophagus (hidden by distention).

Left Thoracotomy - thru the left 8th interspace. Esophagus is dissected from the hiatus
to the aortic arch, ligated the esophageal arteries and protecting the vagus nerves. The
phrenoesophageal ligament is incised circumferentially and the cardia is mobilized. The
residual phrenoesophageal ligament and fat pad of the cardia (from vagus nerve to vagus
nerve) is resected to provide exposure to the cardia for the myotomy.
Myotomy - proximally from the aortic arch distally on to the stomach for a distance of
no more than 1 cm. The incision is carried thru the muscle to the level of the mucosa.
The fibers are then separated to expose about 1/2 the mucosal tube to prevent rehealing.

Associated antireflux Procedure - a total fundoplication is not performed as aperistaltic


esophagus cannot overcome the pressure at the LES with a Nissan.
modified Belsey Mark IV - sutures placed posteriorly in the diaphragmatic crus to
tighten it so that it admits an index finger. 4 horizontal mattress sutures are used (at 12
0’clock, 3 o’clock, 6 o’clock and 9 o’clock) - placed circumferentially in the
hypertrophied , thickened esophagus on either side of the “pouting” mucosa and secured
to the diaphragm adjacent to the crus.
- 2 heavy sutures placed between the edges of the right crus posteriorly and tied
down with enough room to admit a finger.

Antireflux Procedure or Not? - if the myotomy is not carried onto the cardia, an antireflux
procedure is not necessary. Problem is that if the myotomy is not carried down far
enough onto the GE juction, the patient’s dysphagia may persist. Can assure that the
myotomy is adequate by carrying it down onto the cardia at least 1 cm, but this
necessitates an antireflux procedure. Should consider an antireflux procedure (Belsey
Mark IV) with every myotomy.

5
Scenario #3 - Esophageal CA
55 y.o man with dysphagia and 20 lb. weight loss. UGI shows midesophageal stricture.
Issues:
a.) Appropriate workup of esophageal stricture. What if scope can’t pass stricture?
Bronchoscopy? Criteria for resectability. Preop TPN?
b.) What are operative options? How are each performed. How would you mobioize the
stomach for a pull up? What if it can’t be made to reach?
c.) Complications of blunt esophagectomy?
c.) Difference in prognosis for adeno or squamous?
d.) Adjuvant therapy based on nodal status?
e.) Intraop treatment of liver nodule? What if unable to get tissue? Celiac node?
ANSWER: 90% present with dysphagia and weight loss.
Workup: all patients with dysphagia should have barium swallow and CXR.
Radiology - Barium swallow with upper GI - unusual to show proximal dilatation with
CA. Length of tumor correlates with the extent and resectability of the tumor.
- CT scan of chest and abdomen - if CA present on biopsy.
-Bone scan only for bone pain orsymptoms of bone pain.Assist in
identifying the location and extension of the primary tumor. Note the mets to the lung,
celiac axis lymph nodes, liver, adrenal glands or other thoracic or abdominal
abnormalities.Can also see extension into the trachea, left main stem bronchus, or
mediastinum.
-Brain CT for symptoms only.

Esophagoscopy - should be performed for histologic diagnosis of the tumor and for
evaluation of intramural metastasis. Biopsy and brushings should be performed. Biopsy
70% diagnostic, brushings 90% diagnostic, together 95% diagnostic. Should note
location of cricopharyngeus, length and location of tumor and GE junction. Should use
pediatric endoscope to visualize GE junction and stomach if tumor is constricting.
Bronchoscopy - should be performed for upper and middle third tumors. Bulging of the
distal membranous trachea and left main stem bronchus may be tumor and it is essential
that these be biopsied /brushed.

**THESE STEPS WILL COMPLETE THE CLINICAL STAGING.

Nutritional Status - studies show that only those with severe malnourishment benefit
from preop TPN. Those with less than 10% wieght loss did better with chemo than those
with weight loss over 10% (presumably from less advanced disease.)
- enteral nutrition is chosen over TPN for preop nutritional support. This can be
done with soft silastic catheters placed blindly or endoscopically if severe obstruction is
present. Answer - need nutritional support for weight loss>10%, enteral is preferred.

6
SURGICAL OPTIONS AND CONSIDERATIONS:

Palliation - surgery provides excellent palliation from dysphagia and is not exceeded by
any other modality. Surgical resection must be considered the preferred option if the
patient is fit enough to withstand esophagectomy and the tumor is considered to be
relatively localized and resectable.

If patient felt to be resectable and localized --> Operative Staging for Resectability
Celiac Node -----> positive with distal third tumor ---> may still be able to resect.
------> positive with cervical or mid-third tumor -----> incurable disease,
should not resect unless resection is necessary for palliation of significant dysphagia.

Liver -----> positive I


Bone metastasis -------> I -> should consider other palliation as patient’s
Extensive positive regional nodes----> I survival is 6 months wont benefit fr.
surgery.

OPTIONS FOR SURGICAL RESECTION

TRANSHIATAL ESOPHAGECTOMY
Description:
1.) Abdomenal exploration - the tumor is palpated thru the hiatus to determine
resectability and involvement of tumor with aorta, spine and tracheobronchial tree. If
no mets are found (or if there are celiac nodes and dysphagia but no liver or distant
mets), the stomach is mobilized preseving the right gastric and right gastroepiploic
arcades. Further mobilization of stomach is provided by a wide Kocher maneuver. The
esophagus is dissected anteriorly and posteriorly into the mediastinum. If the tumor is
adherent to the prevertebral fascia, a right thoracotomy may be necessary for resection.
Mediastinum is inspected for bleeding.
2.) Cervical incision obliquely to the level of the left main stem bronchus. Care is taken
to to stay lateral and posterior to the TE groove and the recurrent laryngeal nerve.
3.) The esophagus is pulled thru the posterior mediastinum . 5 cm margin is taken from
the GE junction on the lesser curvature. Long gastric tube is made with 5-6 fires of a
stapler. The stomach is pulled up thru the psoterior mediastinum with a penrose drain.
A pyloromyotomy/pyloroplasty is performed. Cervical anastomosis is performed and
drain is left.NG tube left above the pyloromyotomy.

Contraindication to Transhiatal Esophagectomy without Thoractomy


Absolute - tracheobronchial invasion.
Most Important - surgeon’s assessment after palpation of the esophagus through the
diaphragmatic hiatus that the tumor is fixed to contiguous structures making this unsafe.

7
ESOPHAGECTOMY WITH THORACOTOMY
Description:
Lewis/Transthoracic Esophagectomy
1.) Laparotomy performed and patient is operatively staged for resection. If resectable,
a conduit is prepared.(usually stomach). If this is an extensive middle third tumor, a
transthoracic approach may be performed before celiotomy to assess resectability and to
mobilize the esophagus. Should the tumor be unresectable, a laparotomy is avoided.
2.) The thoracic anastomosis is place under the azygous vein. NG tube is placed under
direct guidance. 2 chest tubes are placed.

Total Thoracic Esophagectomy


1.) Lapartomy performed and patient is operatively staged for resection. If resectable, a
conduit is prepared (usually stomach).
2.) If resectable, the neck incision is made and the esophagus is dissected.
3.)Resection of the head of the left clavicle to widen the thoracic inlet. Cervical
anastomosis between the cervical esophgus and the stomach/conduit is performed.
4.) Right thoracotomy is performed for esophagectomy. Esophagus is stapled at proximal
and distal ends and resected with specimen.

COMPLICATIONS OF PROCEDURES:
1.) Anastomotic leak - 0-25%. Thoracic leak is slightly less common than a cervical leak
but less morbid because of the avoidance of mediastinitis.
2.) Anastomotic strictures. - more common with stapled than hand sewn.
3.) Respiratory insufficiency. - pneumonia, aspiration
4.) CHF.
5.) PE
6.) chylothorax - 5%.

PATHOLOGY:
Squamous Cell CA - 95% of all cases of esophageal CA. Primary tumor may extend
into aorta or left main stem bronchus. Distant mets most common to lung and liver.
Lymph node mets in 75% of cases.
- very aggressive, 5%-12% of patients survive 5 years.
- with + LN, five year survival is 3%.
- without LN mets, five year survival is 42%.
- 70% have LN mets at diagnosis.

Adeno CA - 5% of all cases of esophageal CA.


- very aggressive , 0-7% of patients survive 5 years.
- with + LN, average survival is 9 months.
- without LN mets, survival is possible.

8
** adjuvant therapy based on LN status is possible with clinical trial. Adjuvant therapy is
best with combination chemotherapy and shows enhanced survival when combined with
locoregional control. Survival is shitty.

Scenario #4 - Boerhaave’s Esophagus/Spon.


Perforation.
67 y.o. male in E.R. 3 hrs after episode of vomiting with development of chest pain and
epigastric pain.
Issues:
a.) Differential Dx (Brief). Workup of esoph. perforation.(include description of tests to
rule out differential dx.) Where is leak located? Pathophysiology. GG or Ba.
Resuscitation and antibiotics/preop preparation.
b.) Appropriate surgical treatment at this time pt. 12 hrs? 24 hrs?
36 hrs? Plans for nutrition? How are each done?
c.) Post -op leak - what do you do?

Answer:
Differential Diagnosis: Perforated esophagus, MI, perforated DU, pancreatitis,
pneumonia.
Tests: Phys. Exam (crepitus for cervical peforation or intrathoracic perforation, chest
exam for posssible pleural effusion or mediastinal “crunch” of mediastinal emphysema),
vitals (tachycardia, hypotension, fever), cbc with diff, amylase, lipase, EKG, CXR
(upright film to rule out free air). Gastrograffin swallow. Thin Barium swallow if GG
swallow negative and still with a high index of suspicion.

Pahtophysiology - increased pressure in the esophagus with wretching or vomiting


causes a linear tear, usually on the left side, almost always in the distal third of the
esophgus. Leakage of digestive juices and bacteria into the chest cause a severe
mediastinitis and may communicate with the pleural cavity.

MANAGEMENT
initial - IV fluids, antibiotics - cephalosporin (cefazolin, cefamandole) and
aminoglycoside (gent or tobra) to control oral flora.

SURGICAL THERAPY:

EARLY ESOPHAGEAL PERFORATION (WITHIN 24 HRS OF INJURY) - in those


not associated with intrinsic disease, treat with primary repair combined with wide
mediastinal drainage. This is performed by opening the mediastinal pleura from the
diaphragm to the thoracic inlet over the perforation.

9
Approach - L thoracotomy thru 6th or 7th interspace for distal third perforations.
- R thoracotomy for more proximal thoracic perforations.

Closure - prolene.
Repair - should be butressed. Choice of material is not as important. Fundoplication
wrap can be performed for perforations of the distal third of the esophagus or GE
junction. The wrap must not be left in the chest or allowed to herniate. Other options
include omentum, parietal pleura and intercostal muscle. Grillo flap is a pleural flap
wrapped circumferentially around the perforation site.
- NGT is placed until the postop ileus is resolved and then oral feedings can be
instituted.
- Barium esophagram is performed at day 10. Chest tube is left in place until this
exam is completed. If this exam shows a leak, the hest tube is left in place and
esophagocutaneous fistula is allowed to heal slowly. Most of these will heal if they are
small and there is no distal obstruction. Wide mediastinal drainage is key. As long as
fistulas are small and well drained, oral feedings can be instituted.

LATE ESOPHAGEAL PERFORATIONS (AFTER 24 HRS.) - primary repair of the


perforation is not feasible due to friability. Though this is the rule each perforation
must be inspected to decide on the ability to perform a primary repair.
Critical decision - which patients will benefit from treating an
esophagopleuralcutaneous fistula and which will benefit from diversion/exclusion or
esophagec tomy.

24-48 hrs - T - tube or wide drainage with chest tubes.


48 hrs. - diversion and total exclusion - effectively prevents further mediastinal
contamination . Favor end cervical esophagostomy /”spit fistula” with stapling or
ligation of the distal cevical esophagus, ligation of the GE junction and placement of
gastrostomy
(to prevent GE reflux into esophagus) and jejunostomy tube for feeding.
- should be used in the case of the desparately ill patient. Requires a second
operation to restore esophageal continutity, usually with a left colon replacement.

ESOPHAGEL PERFORATION ASSOCIATED WITH INTRINSIC DISEASE - while


the perforation is quickly diagnosed , primary repair is often unsuccessful because of
distal obstruction.
Examples:

Achalasia - should be treated with suture repair, esophagomyotomy, and fundoplication


over the repair (Belsey mark IV)

Esophageal CA, hard stricture , caustic injury, trauma - should be treated with resection.
If the perforation is diagnosed quickly and the mediastinal contamination is not severe,
and the stomach is adeqate, a conduit may be fashioned at this time.

10
Scenario #5 - Zenker’s Diverticulum
40 y.o. progressive dysphagia, regurgitation of not too digested food, and bad-breath.

Issues:
a.) Appropriate workup?
b.) Surgical correction? Detailed description with anatomy. Which would you excise or
pexy? What size bougie would you use for sac excision?

ANSWER:
ANATOMY - arises within the inferior pharyngeal constrictor muscle between the
oblique fibers of the thyropharyngeus and the horizontal fibers of the cricopharyngeus
muscles. The potential weak point is Killian’s triangle.
- the hernia sac gradually enlarges above the cricopharyngeus muscle, eventually
extending over the cricopharyngeus and can extend into the superior mediastinum.
- it is not only the size of the diverticula that dictates the severity of symptoms,
but also the degree of upper esophageal muscle dysfunction.

DIAGNOSIS - contrast radiography

SURGICAL THERAPY
- exposure is thru the left neck thru an incision which is parallell to the SCM. Dissection
is continued in the tracheoesophageal groove beneath the left inferior thyroid artery.
- Cricopharyngeal myotomy - releives the relative obstruction at the level of the UES.
Myotomy should extend from the pouch 3-5 cm inferiorly onto the esophagus thru the
cricopharyngeus muscle.

Sac < 2 cm - myotomy is sufficient


Sac > 2 cm - smaller - diverticulopexy - pex the sac to the prevertebral fascia.
- larger - the sac is stapled over a 50-60 Fr. bougie to prevent narrrowing of
the pharyngoesophagus and predispose suture line breakdown and leakage.

Scenario #6 - Paraesophageal/ Type II Hiatal Hernia


62 y.o. woman with early satiety and substernal and epigastric fullness after meals.

Issues:
a.) Appropriate workup of type II hernia?
b.) Appropriate surgical repair for type II hernia?

ANSWER:
TYPE I - SLIDING HIATAL HERNIA
TYPE II - ROLLING PARAESOPHAGEAL HERNIA
TYPE III - COMBINED PARAESOPHAGEAL HERNIA.

11
- TYPE II ANDIII arise from a protrusion of the peritoneum through the hiatus in
to the thoracic cavity.
- differentiating type I and type II is based on the position of the GE junction.
With a type II hernia , the GE junction is located in the normal position. With a type III
hernia the GE junction migrates into the thorax.

- Type II and III hernias arise from a weakening of the phrenoesophageal


ligament.

WorkUp:
Barium Swallow - reveals abnormality but cannot always differentiate a type II and III.
Fiberoptic bronchoscopy - should be performed to help identify the location of the GE
junction as well as detecting any areas of blood loss. Linear esoions are often seen at the
site of the stomach wall adjacent to the ring of the diaphragm which makes up the hernia
ring.

Acute complications: excessive bleeding or volvulus with gastric obstruction or


infarction.
Catastrophic complications occur in 20-30% of those with this diagnosis with diagnosed
paraesophageal hernias.

SURGICAL APPROACH - can be performed thru the abdomen or thorax with roughly
equivalent results.
Basic goals - reduction of hernia sac contents into the abdomen, resection of the hernia
sac, closure of the diphragmeatic hiatus to an appropriate caliber, anchoring the stomach
within the abdomen

Closure of the diaphragmatic hiatus - lateral fibers of the right diaphragmatic crus form
the septum between the esophageal hiatus and the aortic hiatus. These septal muscle
fibers are attenuated in the case of a type II hernia. Closure of this defect, crucial for the
prevention of a recurrent hernia, can be assisted by dissecting the diaphragm from the
pericardium for several centimeters. Prosthetic material may be used if necessary to
close the defect.

Anchoring stomach - can be performed by :


1.) gastrostomy tube.
2.) suturing the proximal lesser curve to the median arcuate ligament.

?Antireflux procedure - controversial.

Scenario #7 - Melena/Bleeding Peptic Ulcer


60 y.o. male with melena.

Issues:

12
a.) Initial treatment and diagnostic plan/fluid resuscitation and NGT/stabilize.
b.) Most likely diagnosis/ Could this be ZE?/Differential dx of UGI bleed.
c.) If NGT returns blood, what do you do?
d.) On endo, posterior penetrating duodenal ulcer which is not actively bleeding, but has
clot over it, what do you do? Remove clot or leave, inject or not, cauterize or not?
e.) Assuming patient bleedsagain 12 hrs. later, what do you do? What if the duodenum is
scarred from chronic disease?
f.) What would be your surgical procedure of choice and how would you perform it?
Oversew =/- GDA+/- V&P. What are the potential complications of that procedure?
g.) UGI bleed after bypass for unresectable CA. After resuscitation and monitoring,
what are surgical options.

Answer:
Inital treatment - 2 large bore IV’s
Fluid - LR is chosen over over .9 NS because sodium and chloride more closely
approximate that of whole blood lost. Because of the inability of renal failure patients to
clear this sodium and chloride, colloid/blood and invasive monitoring should be
instituted.
- the massively hemorrahaging patient who is hemodynamically unstable should
be transfused with blood to maintain the O2 carrying capacity. Endoscopy can be
essential in identifying those lesions which are likely to rebleed (esophageal varices)
from those which are not (Mallory Weiss) and can assist in decision to transfuse early.

Type and Cross -


cbc + platelet count - Hct will not reflect blood loss, need clinical eval.
LFT’s
Coag’s

NGT - blood - obvious upper GI bleed


- no blood + bilious return - unlikely upper GI bleed (80% accurate).
- no blood + no bilious retrun - need upper endo to rule out DU bleed

Monitoring - art. line, telemetry, Foley, ICU .

Differential Diagnosis:

3 most common:
- Posterior penetrating duodenal ulcer
- large gastric ulcers
- gastric or esophageal varices.

Peptic Ulcer Disease


- Duodenal Ulcer
- Gastric Ulcer

13
- Marginal Ulcer
Gastritis
- stress ulceration
- alcohol gastritis
- drug induced gastritis
GE varices
Mallory Weiss
Esophageal, gastric or duodenal tumor
Dieulafoy’s lesion
Aortoduodenal fistula.
Esophagitis
Angiodysplasia
Hemobilia
Pancreatitis induced pseudoaneurysm.

DIAGNOSIS:
- if bilious, nonbloody fluid is returned, lower GI bleed will be diagnosis 80% of time.
- if nonbilious/nonbloody or bloody fluid is returned, upper GI bleed is suspected and
upper endoscopy is peformed.
- if upper endoscopy is diagnostic, appropriate therapy is taken.
- if upper endoif not diagnostic, decision for next test is made based on the severity of
blood loss. In cases of massive hemorrhage (limit of detection is 0.5 -1 mL/min.) a
visceral angiogram is necessary.

Nonoperative therapy/Therapeutic Endoscopy


- Injection Therapy - should be used to inject first around the clot and then in to the center
of the fibrin clot.
- Heat/Laser Therapy - has been shown in metanalysis to shorten hospital stay and
number of units transfused.
- Visible nonbleeding vessel with clot at ulcer abase - 80% chance of rebleeding in one
study. THOSE WITH BLEEDING VESSEL AND THOSE AT HIGH RISK OF
RE-BLEEDING ( AS ABOVE ) SHOULD UNDERGO EMERGENT
ENDOSCOPIC HEMOSTATIC THERAPY - BOTH THERMAL AND
INJECTION THERAPY WILL DECREASE THE REBLEED RATE, NEED FOR
EMERGENT SURGERY AND HOSPITAL MORTALITY.

HIGH REBLEED RATE ULCERS:


- LARGE GASTRIC ULCERS
- ULCERS HIGH ON THE LESSER CURVE NEAR THE LEFT GASTRIC
ARTERY.
- POSTERIOR INFERIOR ULCERS OF THE DUODENUM CLOSE TO THE
GASTRODUODENAL ARTERY.

14
SURGICAL THERAPY -

GASTRIC ULCERS - partial gastric resection to include the ulcer. Biopsy to rule out
malignancy necessary. If location of ulcer instomach is in doubt, use intraop endoscopy
to lacate the ulcer .

DUODENAL ULCERS -
1.) Oversew of Ulcer with Vagotomy and Drainage - Rate of recurrent bleeding after
GDA ligation is lower.
2.) Gastric Resection with Excision with Ulcer -

Bleeding Duodenal Ulcer with scarred Duodenum - rather than risking leak from scarred
duodenum or performing suboptimal operation in the face of chronic disease, one should
perform a truncal vagotomy and antrectomy (resection + acid reducing procedure) with a
gastrojejunostomy vs a gastroduodenal anastomosis.

Rebleed - most often attributable to lack of control of the initial bleeding site.
If patient rebleeds after oversew and vagotomy/drainage, the vessel should be directly
visualized and ligated. If rebleed occurs after oversew of gastric ulcer , a gastric
resection plus vagotomy should be performed.

Complications of Therapy
1.) Postop GI bleeding
2.) Duodenal pyloroplasty suture line leak.

Bleeding after surgical bypass for unresectable CA - punt.

Scenario #8 - Gastric Ulcer


50 y.o. male failing H2 blockers with Type I Gastric Ulcer

Issues:
a.) Workup of nonhealing gastric ulcer?
b.) Appropriate treatment of nonhealing gastric ulcer?
c.) How does plan change if biopsy shows malignancy?
d.) Detailed description of gastric ressection for CA.
e.) What is the significance of achlorhydria?
f.) What are complications of each surgical options for nonhealing ulcer? (dumping,
marginal ulcers, gastric outlet obstruction, efferent loop syndrome)

Gastric Ulcer with Gastric Outlet Obstruction - what is the electrolyte abnormality and
how would you go about correcting it. When would you operate and what would you
do?

15
Answer:
Types of Benign Gastric Ulcer :
1.) Type I - incisura or most inferior portion of lesser curve - not associated with other
gastroduodenal disease. Often associated with achlorhydria.
2.) Type II - gastric and duodenal - associated with acid hypersecretion, aka P.U.D.
3.) Type III - pyloric and prepyloric - associated with acid hypersecretion, aka P.U.D.
4.) Type IV - juxtacardiac - like Type I, not associated with other gastroduodenal disease.

Workup - most patients present with complaints of vague epigastric pain, which can be
caused by a variety of causes, from dyspepsia to gastric CA.
- endoscopy - to identify ulcer and biopsy. All gastric ulcers to be observed should
be biopsied to rule out malignancy. When taking into account endoscopic appearance,
biopsy and brush cytology, 95% of benign gastric ulcers can be differentiated.

Medical Treatment - because most patient s with type I gastric ulcers have normal or
reduced acid levels, the use of antisecretory drugs may not make sense. However, the
further decrease of acid levels may accelerate the healing process. Unlike duodenal
ulcers, gastric ulcers healing is due to duration of therapy not the amount of acid
depression.

H2 blockers - healing rates are similar to duodenal ulcers, however - the duration of
therapy needed take 2 weeks longer
Omeprazole - similar or better healing rates to H2 blockers.
- medical therapy requires reevaluation in 6 weeks to confirm healing and rule out
malignancy.

SURGICAL THERAPY -
INTRACTABILITY - when ulcer fails to healwiyth standard medical therapy, the
possibility must be reassessed. Other causes of intractability are Crohns, eosinophilic
gastritis, gastric lymphoma, TB, CMV. If ulcer has failed to heal in 12 weeks of
maximal medical therapy. 95% of benign gastric ulcers will heal on 40 mg/d of
Omeprazole for 12 weeks.

- because the role of acid is less prominent in the pathogenesis of gastric ulceration
( esp with type I and IV) when compared to mucosal defects, acid reducing
procedures are less reliable than those that remove the defective mucosa.

Type I Ulcer - antrectomy performed in a manner to include the ulcer. BI reconstruction


is preferred when compared to BII because it is more physiologic. Because
hypersecretion of acid is not a factor, a vagotomy has no role in adddtion to the
antrectomy. Leak rate is low as the duodenum is usually normal. Truncal vagotomy
and pyloroplasty has little role because of ulcer recurrentce rates are higher than
antrectomy.

16
COMPLICATIONS OF GASTRECTOMY:
1.) DUMPING - result from extirpation or loss of the sphincter function and loss of the
storage function. The addition of a selective and truncal vagotomy may enhance this
problem as both enhance the evacuation of liquids from the remnant of the stomach.

2.) DIARRHEA - post vagotomy diarrhea has not been fully elucidated.

3.) MARGINAL ULCER

4.) GASTRIC OUTLET OBSTRUCTION - SEE 3.)

5.) AFFERENT LOOP SYNDROME - occurs after BII, in which there is stasis in the
remnant of duodenum and jejunum if there is partial or complete obstruction.
Obstruction can occur from stenosis at anastomosis, kinking, volvulus, intusseception.
Incidence is greater if the loop is longer (10-15 cm) , located on lesser curve, or antecolic
position.
Can lead to duodenal blowout or reflux symptoms (emesis of bile without food). Causes
accumulation of food in afferent limb during eating, has increased pressure resulting in
increased pressure and emesis. Diagnosis is difficult . Surgery for resuspension or
shortening of the limb may help.

6.) EFFERENT LOOP SYNDROME - symptoms are similar to small bowel obstruction
with distention, colicky abdominal pain, emesis of bile. Jejunogastric intusseception is a
rare cause and usually occurs with after antecolic BII gastrojejunostomy. Efferent loop
intussecepts into the stomach.

7.) ALKALINE GASTRITIS

SUBTOTAL GASTRECTOMY FOR CA -


RESECTABILITY - Peritoneal Seeds
- Rectum and pouch of Douglas
- Uterus and ovaries in women.
- Palpate both lobes of liver.
- dissect the greater omentum from the left half of the transverse colon.
- mobilize the left colon from the spleen.
- lift the stomach cepahlad to determine if the tumor is fixed to the pancreas
- determine if the tumor involves the tranverse mesocolon.

- identifty the left gastric artery and follow it to its origin.Biopsy celiac nodes.
(If negative , can operate for cure.)
- porta hepatis nodes are biopsied. (perihepatic and right gastric nodes are taken
with specimen)

17
- determination is made if an adeqaute gastric/esophageal margin can be obtained
with resection.

Resection
- spleen is mobilized from retroperitoneal attachmentsand the avascular plane behind the
pancreas anterior to the kidney. The spleen is mobilized.
- Left gastric artery and vein are found and ligated. The left gastric stump is palpated
alsong with the hepatic artery. The common hepatic atery is traced to the right gastric
and gastroduodenal artery. These vessels are taken and the lesser omentum is taken and
dissected toward the gastric specimen.
- Duodeenum is mobilized via Kocher maneuver.
- Line of resection is chosen and Kocher clamps placed (5cm prox and 8-9 cm distal) and
sthe stomach is divided. Spec. is taken for frozen. Once frozen is returned negative , the
small medial vessels just past the pylorus between the duodenum and the pylorus are
taken - duodenum is transected 1 cm past the pylorus and the frozen is sent. The
duodenal stump is oversewn. The proximal gastric stump is oversewn.
Gastrojejunostomy is performed.

Scenario #9 - Recurrent PUD.

Issues:
a.) Appropriate workup for recurrent PUD.
b.) Workup for ZE.
c.) Appropriate surgical treatment for ZE.

Answer:
Causes of Postoperative Recurrent PUD :
Inadequate Surgery:
- Incomplete vagotomy
- Retained Antrum
- Inadequte gastric resection
- Stenotic gastric outlet.
- Long Afferent Loop.

Other causes:
- Gastrinoma
- Primary hyperparathyroidism
- Antral G cell hyperplasia
- Duodenogastric or jejunogastric reflux.
- H.pylori

DIAGNOSIS

18
EGD - allows biopsy for evaluation of retained antrum, eval duodenal mass
(gastrinoma), or gastric outlet obstruction.

BASAL GASTRIN - 100-200 rules out gastrinoma


200-1000 suggestive of gastrinoma
>1000 diagnostic of gastrinoma
**increase of at least 200 following injection of
secretin with gastrinoma
.
Retained antrum - basal gastrin 2-3 times normal
- minimal effect on secretin stimulation
- confirm by technitium scan

Antral G cell hyperfunction - basal gastrin levels 2-3 X normal


-** 200-300% increase in gastrin levels with a high
protein meal
- minimal effect of secretin infusion.

Sham Feeding - 1 lhr basal acid output (BAO) + 1 hr stimulated acid output (SAO) + 1
hr peak acid output following pentagastrin infusion (PAO)
nonvagotomy - SAO is 40% PAO
100% vagotomy - SAO is 10% PAO(SAO is abolished and PAO is reduced)
** if BAO is elevated the sham feeding test may yield a false positive for
incomplete vagotomy.

Medical Therapy - indicated as it will heal most ulcers. 4 weeks minimal treatment is
necessary and 12 weeks may be needed if the ulcer is healing. Ulcer after 12 weeks
requires surgery.

Surgical Therapy - indications include failure of medical therapy or complication on


presentation.
Incomplete vagotomy - as determined by BAO >2mmol/hr. postoperatively or + sham
feeding - should be treated with revagotomy - transthoracic vagotomy should be used in
elective setting. This should not be used in face of a complication. Transabdominal
approach allows eval of anast stenosis, retained antrum, present of gastrinoma.

* risk of leaving vagus intact after second procedure is high, so operation of greater
magnitude should be chosen (antrectomy, subtotal gastrectomy.

WORKUP OF ZOLLINGER ELLISON:


TEST #1 - peripheral venous gastrin .

Gastrin Interpretation
0-100 Normal\

19
100 - 200 Indeterminant
200-500 Presumed Z.E.
> 500 Diagnostic

TEST #1.5 - Secretin stimulation for indeterminant gastrin


- 2 U secretin per kg of body weight - obtain serum gastrin 10 and 5 minutes before and
1, 2, 5, 10, 20, 30 minutes after injection. Absolute increase of 200 is diagnostic.

TEST #2 - CT SCAN - given with IV contrast - gastrinomas are seen as bright spots of
hypervascularity on scan. Better for detecting metastasis than for detecting primary
tumors ( detects only 30-40% of primary tumors). Sensitivity of MRI is identical to that
of CT scan.

TEST #3 - SELECTIVE ANGIOGRAPHY - single best study as it reveals 50% of


primary tumors and accurately detects liver mets. Selective venous sampling after
secretin injections can be combined with selective angiography to enhance
localization. .Catheters are placed in the gastroduodenal, hepatic and splenic arteries and
the hepatic vein. Secretin in injected into the arteries and blood is collected from the
hepatic veins as well as a peripheral vein and gastrin levels are measured. The artery
that results in highest gastrin level after injection of secretin provides the likely area of
gastrinoma.
-secreti angiography results in diagnosis of 75% of gastrinomas.

TRANSHEPATIC PORTAL VEIN SAMPLING - catheter is placed thru the liver into
the branches of the portal vein and blood is collected for gastrin levels. The branch
closest to the tumor will yield the highest gastrin level. 80% diagnostic rate , but
associated with hemobilia and bleeding in 10% of cases.

SURGICAL THERAPY OF GASTRINOMA -


80% of gastrinomas located in the gastrinoma triangle.
Important - the pancreas and duodenum should be completely mobilized as tumors can
be found within the wall of the duodenum. Intraop US has not been shown to be
beneficial in localization of tumors. Localize the CBD ( with probe) and the major and
minor papilla with secretin (1U/kg).

- rather than proceeding with a Whipple procedure for tumor s of the duodenal wall
or the head of the pancreas, it is recommended that these tumors be enucleated and
that lesion of the tail or body of the pancreas should be resected. Tumors of the
body and tail seem to be more malignant.

Liver mets - intraop US can be used to detect liver mets that can’t be felt. These should
be removed by wedge ressection if possible at the same operation. Removal by formeal
lobectomy is advocated by some. Gastrinoma in lymph nodes should be resected.

20
Metastatic gastrinoma - 40-50% will have localized disease amenable to resection.
80% of those operated on will have resections. Those with nonresectable tumors should
be treated initially with H2 blockers or omeprazole.

No Tumor Found - 1.) Treat with H2 blockers if effective preoperatively.


2.) Treat with total gastrectomy if H2 blockers have not been
effective preoperatively.

Scenario #10 - Perforated Duodenal Ulcer with Prior History


of PUD
18 y.o. foreign exchange student with history of epigastric pain treated with antacids
presents with abdominal pain.

Issues:
a.) Initial treatment of the patient including Ivs, antibiotics, and x-rays.
b.) What is the operative treatment assuming that you find a 4mm perforation just at the
distal lip of the pylorus.
c.) What are the expected results and complications of the various surgical modalities that
you have to choose from.
d.) What would you do if there is a 5 year history of epigastric pain but no definite
diagnosis of PUD?
e.) describe the operative technique of Graham patch and highly selective vagotomy.

ANSWER:

Situations:
I.) No antecedent history, no risk factors
II.) No antecedent history, + risk factors - omental patch
III.) Antecedent history , no risk factors
IV.) Antecedent history , risk factors - omental patch

Inital Treatment - IV’s , Foley, fluid resuscitation , NG tube aspiration.


Antibiotics - Broad spectrum directed at mixed enteric flora.

X-rays - upright abdominal radiographs should show free air, but 20% will not show free
air that do have perforations.
- Upper GI series performed with water soluble contrast may be helpful if
perforation is suspected but not demonstrated with pneumoperitoneum.

?ULCER HISTORY - either by history or anatomic evidence of duodenal scarring should


be sought. Many (2/3) of those with perforation will have chronic disease and will not be
treated adequately with simple omental patch.

21
3+ Goals of Treatment
1.) patient safety
2.) peritoneal cleansing
3.) closure of perforation
(4.) alteration of ulcer diathesis isminimized if applicable.

OPERATIVE RISK FACTORS:


1.) Concurrent medical illness
2.) preoperative shock
3.) perforations of greater than 48 hrs. duration.

Those with these risk factors ----> peritoneal irrigation with omental patch closure. Can
be treated postop with antisecretory meds and delayed wurgery if necessary.
Those without risk factors ( history or anatomic evidence of chronic ulceration, no shock
preoperatively, no life-threatening comorbid disease, perforation present for less than 48
hrs.) --------> definitive ulcer procedure did not produce longer hospital stay or
mortality. Should have closure of perforation and definitive ulcer procedure if
applicable.

Results of Treatment -
Omental Patch - Recurrence =
Acute disease - 36% recurrence
Chronic disease - 60-80% recurrence
- Complications - leak or recurrent ulcer

Argument for definitive surgery - by 5-6 years out the recurrence rate for acute and
chronic ulcers meet. Treatment with H2 blockers or antisecretory therapy means a
lifetime of treatement. However, the anti ulcer procedure must be one associated with
few side effects- i.e. highly selective vagotomy

Highly Selective Vagotomy a.k.a. Proximal Gastric Vagotomy - Recurrence =


Acute disease - 10%
Chronic disease - 3-16%

Study Comparing PGV, Truncal vagotomy , Omental closure -


Recurrence 4% 12% 63%

Graham patch - 3-4 000 sutures are placed in the dudenum 1-1.5 cm from the edge of
the site of perforation passed thru the site of perforation with an equivalent mirror bite on
the opposite side. ( prevents getting backwall with closure) . These sutures are placed
but not tied. A tongue of omentum is placed over the site of perforation an d the sutures
are tied down. Abdomen is irrigated with attention to the retrohepatic, subhepatic ,
perisplenic and retrogastric spaces.

22
Proximal Gastric Vagotomy - Incision - Midline or right paramedian with extension to
the xiphisternum. round ligament of the liver is divided between ligatures. Place Omni
retractor with 4 blades and retraction cephalad . The left triangular lobe of the liver is
incised to the IVC and the left lobe of theliver is retracted.

Anterior Vagus Nerve first - first identify the nerve of Latarjet which is located 5-7 cm
from the pylorus. Incise the peritoneum overlying the lesser curve beginning 7 cm from
the pylorus , the branches are identified as they pass into the anterior surface stomach.
Nerve branches are dissected individually between the anterior nerve of Latarjet and the
lesser curve of the stomach. Nerve branches are divided between ligatures. Meticulous
dissection is crucial and hematoma formation will obscure anatomy.
- dissection of the posterior leaf can be simplified by rotating the stomach anteriorly.
The posterior peritoneum is incised 7 cm from the pylorus and continued proximally
- peritoneum over the esophagus is incised and esophagus is encircled with a penrose
drain. To ensure that all branches of the GE junction are taken, the intraabdominal
esophagus is dissected circumferentially. The vagal trunks are dissected away from the
esophagus.

Scenario #11 - GERD


38 y.o. female with retrosternal pain after eating, worse at night, with emesis of
undigested food.

a.) Appropriate workup of GERD?


b.) How should each type/severity of GERD be treated?
c.) What are surgical indications ? What procedure should be performed? How is this
done? What should be done in case of stricture?

Answer:

WORKUP:
HISTORY:
Dysphagia - troublesome bit of history, that. Causes are multiple, ranging from slow
passage of food thru the distal esophagus because of severe Barrett’s or severe
esophagitis to benign stricture (Schatzki’s ring ) to CA.

Barium Swallow - will show spontaneous reflux only 40% of time. Therefore, lack of
reflux on esophagram does not rule out reflux. Benefit is ruling out a foreshortening
of the esophagus and lack of suitable intrabdominal esophagus for a Nissen wrap.

Endoscopy - allows evaluation of:


-distal esophagus
-metaplastic changes c/w Barrett’s or
malignant changes

23
- evaluation of stricture as chronic
fibrosis vs. spasm.

GRADING OF ESOPHAGITIS
I - reddening of mucosa without ulceration.
II - erosive and exudative mucosal lesions, which is found in a linear fashion.
III - “cobblestone esophagus” - extensive mucosal involvement with islands of
edematous squamous mucosa.
IV - defined by the presence of complications - large ulcers, stricture formation,
shortening or the presence of columnar metaplasia.

24 HR. pH MONITORING
pH studies - most sensitive test for the diagnosis of GERD
-Before a diagnosis of GERD can be made, it must be proven that the esophagus
is indeed exposed to excessive amounts of acid or alkaline material. - test is
performed by placing a pH probe 5 cm above the manometrically defined GE sphincter.
normal - rarely rises above pH of 7 and rarely falls below 4.
Need to take into account the frequency and duration of episodes where pH is under 4.
Daytime reflux in excess of 90 min. is abnormal. Any nighttime reflux is abnormal.

Esophageal Manometry
- resting pressure best measured above the GE junction, where respiratory wave
inversion (negative) is noted
lower limit of normal = 6mmHg
normal resting pressure = 15 mmHg

Manometry - controversial -dysphagia as a presenting symptom indicates need for


manometry especially if stricture or intra- abdominal obstruction is absent . Generally
hypotensive LES. Indicated whenever esophageal motor abnormality is suggested
by dysphagia or odynophagia and the endoscopic exam does not show clear
structural abnormality. Lack of abnormal cardia manometry contraindicates
performance of an antireflux procedure.

SELECTION OF PATIENTS:

Precise diagnosis of cause of abnormal acid exposure to the esophagus is necessary.


(antireflux procedure should not be performed in the absence of abnormal cardia
manometry - symptoms may be from duodenogastric reflux from distal obstruction or
transient relaxations of LES, neither of which will benefit from antireflux procedure)

24
Indications to perform antireflux procedure in the presence of abnormal cardia
manometry: COMPLICATIONS OF REFLUX - persistent endoscopic esophagitis,
esophageal stricture, Barret’s esophagus an d the documentation of recurrent
aspiration pneumonia.
: UNWILLINGNESS TO ACCEPT MEDICAL THERAPY
Endoscopic Esophagitis - becuase the development of esophagitis is related to the
composition of the refluxed material - not all of those with incompetent sphincters will
have esophagitis. These patients rarely respond to medical therapy and eventually
require a surgical procedure.

Benign Esophageal Stricture - presence of stricture from esophagitis secondary to


reflux in the presence of an incompetent LES is a failure of medical therapy and an
indication for surgical intervention. Malignant stricture is ruled out before surgery via
endoscopy. Prior to surgery the stricture is dilated with a 60 Fr. bougie. When
dilated, manometry and 24 hrs pH monitoring is performed.
Manometry - to asssess distal esophageal peristalsis and rule out achalasia.
pH - if normal may indicate alkaline reflux or stricture from other causes.

Barrett’s Esophagus - complication of persistent reflux esophagitis, always associated


with mechanically deficient LES. At risk for development of stricture, large ulcers and
development of adenoCA. Multiple biopsies are indicated, and grade III dysplasia or
higher indicates esophageal resection.

Scenario # 12 - Crohn’s with Enterovesical Fistula


30 y.o. with history of ileal Crohn’s well controlled presents with pneumaturia.

Issues:
a.) Appropriate workup of a patient with Crohn’s disease and complication of Crohn’s?
b.) With enterovesicle or colovesical fistula , is cystoscopy or cystogram necessary?
c.) What operation would you perform? How much ileum would you resect? Would you
perform a R hemicolectomy?

Enterovesical Fistula - 2-5% of patients with Crohn’s, usually causing pneumaturia and
symptoms associated with recurrent UTI’s (urgency, frequency, dysuria).
Recurrent UTI’s danger to permanent renal damage along with persisitent intestinal
disease make this an urgent indication for surgery.

Radiologic evaluation of Crohn’s disease - contrast studies necessary to delineate the


extent of disease and the severity of disease, especially in those being readied for
surgery.

25
SURGERY - resection of diseased segment of small bowel with extirpation of the
fistulous tract. The opening to the bladder is ususally at the dome and can be debrided
and closed without affecting the trigone. Foley catheter is left in place for several days
and is only removed after radiographic confirmation of the healing of the bladder.
Closed suction is placed near the repair . Repair only the diseased small bowel, need for
R colon is dictated by the extenyt of the diseased small bowel.

Colovesical Fistula - BaE rarely fill thefistula. Cystoscopy will show hyperemia and
inflammation consistent with chronic cystitis, but usually do not localize the fistula. The
current best test is a CT scan with intraluminal contrast.

Scenario # 13 - Progressive/refractory Crohn’s with stricture


40y.o. male with Crohn’s distal ileal stricture refractory to medical therapy.

Issues;
a.) Appropriate workup of Crohn’s disease and stricture? Nutritional status.
b.) Appropriate preparation for surgery including steroids and TPN /is enteral diet
preferable.
c.) What is in TPN, how would you write for it? How long would give TPN?
d.) What are some electrolyte abnormalities expected with diarrhea and dehydration from
Crohns, and how would TPN help this?

Indications for surgery in Crohn’s:


1.) Septic complications
2.) intestinal obstruction
3.) failure of medical therapy

WorkUp - contrast radiography - SBFT - to evaluate the extent of disease the location of
the stricture and extent of disease.
- BaE - to evaluate for colonic disease or distal strictures which may complicate a
simple small bowel resection.

Nutritional support - nutritional support of malnourished patients has been shown to


improve such nutritional parameters as body weight, visceral protein status, and
nitrogen balance.
Indications:
1. Preop nutritional support for the malnourished patient.
2. short gut syndrome
3. fluid and electrolyte repelenishment for those with jejunostomy
4. *chronic small bowel obstruction

TPN vs. Enteral formulas - no clear benefit of one over the other

26
- studies have shown that there is not a clear benefit of TPN in the preoperative patient
unless malnourished. Preop TPN is saved for those with the most severe disease and
the severely malnourished

NUTRITIONAL ASSESSMENT -
WEIGHT LOSS - weight loss of more than 10% of normal body weight may
compromise the host by altering the ability to heal wounds and ability to develop an
immune response the injury. Operative mortality is increased with loss of 20-25% of
nomral body weight. 40% weight loss is life threatening.
- those without weight loss and do not have hypercatabolic state do not require nutritional
support for 5-7 days.

ALBUMIN/SERUM PROTEIN - has been shown to correlate with protein calorie


malnutriton, increased hospital stay and poor outcome.
Problem - tend to be low in septic patients because of acute phase reactants.

TPN:

Basal Energy Expenditure - BEE -

Male BEE = 664+ 13.7W+5H- 6.7A


Female BEE = 655+ 9.6W+ 1.8H-4.6A

Additional Calories - added if the patient is desired to gain weight.


500 calories - to gain 1 lb. per week
1000 calories - to gain 2 lb per week

Protein Needs - 0.8 grams per kilogram per day in the normal nonstressed patient
- range is from 0.8-1.5 gram per kilogram per day based on the amount of
stress. 1.5 gr per kg per day is only for those with substantial losses from wounds or
fistulas.

Carbohydrate and Fat Mixture - providing the necessary glucose for function of CNS,
nervous system and blood will prevent the breakdown of protein to amino acids
which will be converted to glucose for these tissues.

Optimal Glucose Administration- 4.5 gr per kg per minute or 450 gram per day in 70 kg
male.
- excessive carbohydrate will result in fatty liver and increased C02 production.
- based on 4.5 gram per kg per minute of carbohydrate will result in 30% of calories
from fat.

ENERGY PER GRAM OF NUTRIENT:


CARBOHYDRATE - 3.4 KCAL PER GRAM

27
PROTEIN - 4 KCAL PER GRAM
FAT - 9 KCAL PER GRAM

Scenario #14 - Enterocutaneous Fistula


45 y.o. female who undergoes an LOA for SBO S/P hysterectomy. On POD#5, pt.
develops drainage from wound growing enteric organisms, persistent drainage of 2
liters per day.

Issues:
a.) How should one evaluate and treat this wound?
b.) 2 days after wound is cared for bilious drainage is noted on dressing, what does this
suggest?
c.) What is appropriate workup and therapy for enterocutaneous fistula?
d.) A SBFT shows no distal obstruction, how would you treat this fistula?
e.) Under what circumstances will this fistula not heal? What operative timing is
necessary?
f.) How specifically would you handle the TPN?
g.) How would you change your management if this patient was septic?

ANSWER:

WOUND CARE:

Initially, skin edges opened and treated with wet to dry dressings. Ostomy appliance if
drainage is too brisk.
Once fistula has been diagnosed:
Fistula drainage is usually best controlled by use of a sump, usually a soft brown latex
catheter with a vent is used ( like a Robinson nephrostomy catheter)

Skin care - ileostomy appliance with use of Karaya seal/powder placed and left for 7-
10days to allow the skin to heal underneath.

Classification of enterocutaneous fistulas:


LOW OUTPUT FISTULAS/COLONIC FISTULAS = < 200 cc/24 hrs.
MODERATE OUTPUT FISTULAS = 200-500 cc/24 hrs.
HIGH OUTPUT FISTULAS = > 500 cc/hr.

*though spontaneous closure is not strictly related to amount of output, the rate of
complications is.

Workup:
Fistula gram - usually performed at 7-10 days at which time the patient is nutritionally
more sound and the fistulous tract has had mor e time to mature so that water soluble dye
can be used. Small feeding tube is used to infuse contrast thru the feeding tube.

28
Imprtant data from fistulagram: (1.) site of entry of the bowel (2.) nature of adjacent
bowel (inflamed , strictured) (3.) presence or absence of intestinal continuity ( side
fistula or anastomotic blowout. (4.) presence or absence of distal obstruction. (5.) size
of adjacent abscess if present.

FACTORS MITIGATING AGAINST SPONTANEOUS CLOSURE OF FISTULA:


1.) ANATOMIC SITE:

More Likely to Close Spontaneously:


- lateral esophageal fistulas
- lateral duodenal fistulas
- biliary and pancreatic fistulas
- jejunal fistulas

Less Likely to Close Spontaneously:


- gastric fistulas
- ileal fistulas
- fistulas of the Ligament of Treitz

2.) NATURE OF ADJACENT BOWEL:

Disease processes which are less likely to heal:


- inflammatory bowel disease
- radiation enteritis
- CA

3.) ADJACENT ABSCESS:


LARGE -will not close spontaneously
SMALL - may close spontaneously

4.) DISTAL OBSTRUCTION


- PRESENT - will not close
- ABSENT - may close spontaneously

TREATMENT:
NON- OPERATIVE - followed for 4-5 weeks to allow spontaneous healing.
OPERATIVE - if fistula fails to heal in 4-5 weeks with maximal nutritional support.
- early operative intervention for sepsis or progressive organ dysfunction

NUTRITIONAL SUPPORT
Electrolyte disturbances - disturbances in potassium, sodium, magnesium and if TPN has
been used phosphate.

Malnutrition - more common with high output fistulas and in the presence of sepsis. If
the malnutrition is realated to sepsis, the source of the sepsis i.e. abscess must be dealt

29
with first. CT scan can be used to localize abscess and then be percutaneously drained.
If it cannot be localized the patient must be explored and the abscess drained and the
fistula closed and bowel resected.

TPN - because of the likelihood that the patient has lost 300-500g of lean body mass per
day, the dTPN requirements will be greater than normal.
Protein = 1.7 gr protein per kg per day (corresponds to 250 mg nitrogen per day)
Carbohydrate + Lipid - should be given as 75% glucose and 25% lipid emulsion

Trace elements should be given and attention given specifically to Mg, Ca and PO4

Scenario #15 - Acute Mesenteric Ischemia


55 y.o. male 5 mo. S/P MI with sudden onset midepigastric abdominal pain and nausea.
Normal vitals except for rapid, irregular pulse

Issues:
a.) Differential diagnosis (including MI)
b.) ABC’s and initial resuscitation.
c.) Initial evaluation including labs, flat and upright, ultrasound and possible arteriogram.
What are the indications for a mesenteric arteriogram prior to OR?
d.) Assuming the angio shows cutoff of the proximal SMA near the takeoff of the middle
colic artery, what do you do?
e.) Assuming that this patient has had an MI 5 months ago, how would reduce the
cardiac morbidity during operative procedure?
f.) Scen. #15a. 5 feet of gangrenous ileum? Ileum plus embolectomy?
g.) Scen. #15b. Majority of small bowel is dark and dusky, how do you deal with this?
h.) Describe the operative technique of an SMA embolectomy and a SMA bypass.
i.) Describe how you would assess the small bowel? What would you do if you could not
tell the viability?
j.) What is the appropriate post-op workup for embolus - cardiac source, hypercoaguable
state? Appropriate treatment long term?

Answer:

Background - Acute mesenteric ischemia describes a spectrum of bowel injuries from


thrombosis of mesenteric vessels ranging from reversible alterations in bowel function to
transmural necrosis of the bowel wall.
Causes:
- SMA embolus
- nonocclusive mesenteric ischemia
- SMA thrombosis
- SMV venous thrombosis

30
SMA EMBOLUS - usually originate from LV or left atrial mural thrombi . May be
dislodged after disrhythmia or cardiac cath. Emboli usually found at point of narrowing
(branching point). Pathophys. - collateral circulation is sufficient initally , but
vasoconstriction proximal and distal to the point of occlusion reduces blood flow thru
collaterals and worsens ischemia.

NONOCCLUSIVE MESENTERIC ISCHEMIA - thought to result from mesenteric


vasoconstriction initiated by vasoactive medications or by a period of hypotension from
dysrhythmia, MI with myocardial depression or hypovolemia. Vasoconstriction can
persist after the initiating cause has been corrected. Precipitating factors - MI, CHF,
aortic insufficiency, hepatic insufficiency and renal failure. Frequently mesenteric
ischemia can show up hours to days after initiating event.

SMA THROMBOSIS - occurs at the area of severe atherosclerotic narrowing, usually at


the origin of the SMA. Acute mesenteric ischemia is frequently superimposed on chronic
mesenteric ischemia and patient will often have had abdominal pain , weight loss and
malabsorption in the weeks and months before the acute episode.

Clinical Response to Acute Mesenteric Ischemia - within hours of ischemia the bowel
becomes hemorrhagic and edematous, increasing intramural pressure. This can further
compromise already marginal blood flow thru the capillary system. the flow of fluid into
the bowel wall can result in hemoconcenteration and hypovolemic shock. These effects
can lead to death of a patient before full scale bowel wall necrosis appears.

Presetntation - pain out of proportion to physical findings. Passage of blood y maroon


stools canprecede abdominal pain. Nausea, vomiting back pain are late sign soften
indicating compromise of bowel.

Lab Tests -
cbc - leukocytosis > 15,000 occurs in 75% of patients with ischemia.
- Hct elevated may in dicate hemoconcentration, but this is nonspecific.
ABG - 50% will have metabolic acidosis, but this is a late sign usually indicating bowel
necrosis.
*early in course, labs may be unremearkable.

RADIOGRAPHIC EVALUATION
Flat and Upright -
Early /Before Infarction - Normal
Progression - gasless abdomen, adynamic ileus and pseudoobstruction
Late - small intestinal thumbprinting/”Pinkyprinting”
- pneumatosis or air in mesenteric or portal veins is a late sign.

DECISION - SURGERY OR ARTERIOGRAM

31
SURGERY - evidence of peritonitis. Angiogram may assist OR planning, but should
only be done if the pt is not stable.
ARTERIOGRAM - equivocal physical findings, hemodynamically stable.
- used to diagnose major arterial occlusion by embolus and thrombosis as well as
diagnosing nonocclusive mesenteric ischemia by seeing
- narrowing of branches of SMA
- alternating narrowing and dilationof mesenteric branches
- spasm of mesenteric arcades
- impaired filling of mesenteric vessels.
-> these are found in absence of pancreatitis and use of vasoactive drugs for
proper diagnosis.

General Principles of Treatment


- initial treatment must be aimed at correcting the cause of ischemia in case of
nonocclusive disease. Swan Ganz catheter should be placed to evaluate filling pressure
and cardiac output as well as the presence of sepsis.
- plain films should be performed to rule out other causes of abdominal pain.
- if no other diagnosis is made based on abdominal films, the patient should be taken for
an SMA arteriogram.
-even if decision has been made to take the patient to the operating room , arteriogram
should be done to manage the patient properly at celiotomy.

MANAGEMENT AT LAPAROTOMY -> RESTORE ARTERIAL FLOW+RESECT


BOWEL

RESTORE ARTERIAL FLOW - first

SMA EMBOLUS -
Minor Emboli - emboli branches of SMA or distal to the takeoff of the ileocolic.
Can be treated expectantly if pain is relieved by vasodilator therapy.
Major Emboli - at or proximal to the takeoff of the ileocolic. Can attempt vasodilator
therapy if the patient has significant contraindications to surgery (recent MI) , no
peritoneal signs and imporvement of flow distal after instituting vasodilator
therapy.

Embolectomy - SMA is approached thru the transverse mesocolon. The SMA is


dissected proximally between the pancreas and the fourth portion of the duodenum. The
SMA is dissected proximally 2-3 cm and distally to the origin of the middle colic.
Embolus is located by palpation and Doppler exam. Proximal and distal control
obtained.
Longitudinal arteriotomy made and embolus and debris removed. Balloon catheter
embolectomy is performed. Arteriotomy is closed with vein patch or not.

ACUTE SMA THROMBOSIS-

32
- diagnosed on arteriogram by SMA that is out and distal filling thruy collaterals. I f
confused between thrombosis and embolus - treat as embolus. Important to differentiate
acute thrombosis from old thrombosis - old thrombosis is seen with large celiac and
SMA collaterals. Lack of collaterals dictates intervention.

Options;
-Reimplantation
- thrombectomy and endarterectomy
- bypass

NONOCCLUSIVE MESENTERIC ISCHEMIA


ANGIOGRAPHIC SIGNS:
- narrowing of branches of SMA
- alternating narrowing and dilationof mesenteric branches
- spasm of mesenteric arcades
- impaired filling of mesenteric vessels.

- PAPVERINE INFUSION BEGUN AND CONTINUED INTO OR


- at celiotomy, manipulation of the SMA is minimized. Overtly necrotic bowel is
resected and primary anastomosis is performed if no second look operation is planned.
Papaverine infusion continued after OR and angiograms repeated daily until symptoms
resolved.

TESTS OF BOWEL VIABILITY


1.) EYEBALL - bowel placed in warm lap pads for 10-20 minutes and observed for
peristalsis, pulsation, color.
2.) Doppler of antimesenteric side of bowel
3.) Surface floresence with flouroscein dye using ultraviolet light/Wood’s lamp -
Patterns:
- fine, granular pattern of viable bowel
- patchy, perivascular and nonfluorescent pattern of nonviable bowel.

DECISION TO RESECT:
- for short segments clearly necrotic and questionably viable after revascularization are
resected.
- for larger amounts of bowel, only clearly necrotic bowel is resected and plan is made
for a “second look laparotomy”. This also allows further supportive therapy for
improvement of bowel. Primary anastomosis should not be done.
All or most of small bowel dusky - question whether to do any thing.

Scenario #16 - Ulcerative Colitis


40 y.o. male with history of U.C. presents to ER with history of bloody diarrhea and
approx. 8 bowel movements per day.
Issues:

33
a.) Inital resuscitation and treatment .
b.) If pt. does not improve , are steroids indicated? If so, what dose and for how long?
c.) On the 5th day of hospitalization, patient develops obdominal distention and x-rays
show a 10 cm. transverse colon and pt. becomes toxic, what do you do?
d.) Describe the operative procedure and options of choice including possible pouch
procedure, and colostomy.

Answer:
Part 1 - fulminant Ulcerative Colitis
Part 2 - toxic Megacolon

Initial resuscititation -
IV Fluids -
Electrolyte resuscitation-
NG suction
Broad spectrum antibiotics to cover anaerobic and aerobic gram neg.
TPN - improve nutritional status
Steroids- most are on steroids and will need stress doses. Should be given trial of bolus
steroids at a dose of Hydrocortisone 100 mg IV q 8 hrs..

Duration of Nonoperative therapy - Fulminant UC - > 6 bm per day, should continue for
5 days and operate if there is no improvement. Surgery is indicated if patients
condition worsens .
Toxic megacolon - indication for surgery.

Morbidity and Mortality Associated with Operation of Toxic Megacolon -


Overall mortality - 8.7%
After total abdominal colectomy with Hartmann’s = 6.1%
After total proctocolectomy = 14.7%
- conservative therapy better in this seeting, total abdominal colectomy and colostomy.
Allows for later mucosal proctectomy and ileoanal/pouch procedure.

Scenario #17 - UGI/Small Bowel Bleed


32 y.o. male with Fe deficiency anemia and maroon stools intermittently. Negative
UGI+BaE+enteroclysis+Meckel’s scan.
Issues:
a.) when should you operate?
b.) if arteriogram shows small bowel source, should you operate? What do you do if no
gross lesion is seen intraop?

Possible souces of Occult ( Upper ) GI hemorrhage -


1.) Colonic diverticula
2.) Vascular ectasia
3.) AV malformation

34
4.) Rupture peudoaneurysm
5.) Bleeding into pancreatic pseudocyst
6.) Hemobilia
7.) Hemosuccus pancreaticum
8.) Small bowel diverticula
9.) Small bowel neoplasm

ANGIOGRAPHY - not only important for diagnosis but also for treatment of upper and
lower GI hemorrhage. In some cases angiographic control is more effective and
expeditious than operative .

Ruptured mesenteric aneurysm, hemobilia, hemosuccus pancreaticum - situations in


which operation has poor chance of controlling the site of bleeding. Diagnostic and
therapeutic angiography is procedure of first choice.

Bleeding - after nondiagnostic endoscopy, should be taken to angio suite if bleeding is


at a rate of > 1mL/min. If bleeding localized, start vasopressin at 0.2-.4 U/min for 20
minutes, followed by a second arteriogram. If still bleeding, increase to 1U/min for
10 min. If unsuccessful, can try embolization with autologous clot, gelfoam.
- temporary control is often successful. Emergent surgery is indicated for continued
bleeding or recurrent bleeding.

- bleeding of less than 0.5mL/min. has low diagnostic yield.


- if no bleeding site is found, Priscoline 25 mg or papaverine 10 mg IV bolus can be used
to assist in delineating site prior to repeat angiogram. Either embolization or
infusion of vasoconstrictors can have complications: mucrosal necrosis, perforation
and late stenosis.

Radionuclide scanning - no place in patient with hemodynamically significan t bleeding.


Role is in patient who demonstrates episodic, recurrent , occult upper GI bleeding or
in patients suspected of having hemobilia, bleeding into pancreatic duct, or a
pseudocyst.

Scenario #18 - Appendicitis vs. PID


20 y.o. female presents with RLQ pain and nausea. RLQ tenderness and guarding on
exam. Normal rectal and pelvic exams.

Issues:
a.) differential diagnosis and initial workup including cbc, UA, urine pregnancy and
possible abdominal X-rays and ultrasound.
b.) What operative approach would you use?
c.) What do you do if the appendix is normal?

35
d.) what do you do assuming there is some serosanguinous fluid in the pelvis and a red
purulent mass in the right fallopian tube - do you do a right salpingooophorectomy?
e.) What would you do if there is a perforated DU? a cecal mass showing a cecal
adenoCA?

Differential Diagosis:
-appendicitis
-PID
- ruptured/unruptured ovarian cyst
- ectopic pregnancy
- ovarian tumor
- diverticulitis
- UTI/cystitis
- Crohn’s disease
-endometriosis
- torsion of adnexae

Labs:
- cbc
- UA
- urine pregnancy
- pelvic US to rule out ovarian or tubal pathology.
- plain films of abdomen.

Possible operative approaches:


1.) laparoscopy
2.) RLQ incision

- normal appendix = if no other pathology - take out.

- red purulent mass in the fallopian tube and serosanguinous fluid in the pelvis.
PID
ACUTE SALPINGITIS - can be treated as outpatient if nontoxic with cefoxitin (2g IM)
or ceftriaxone (250 mg IM) plus doxycycline (100 mg po bid for 10-14 days).

TUBO-OVARIAN ABSCESS THAT IS NEITHER RUPTURED OR LEAKING -


- admit to hospital, keep NPO, treat aggressively with combination antibiotic therapy.
Frequent examination, vitals, urine output. Any evidence of small leakage or perforation,
then-> life threatening situtation

LEAKING OR RUPTURED TUBO-OVARIAN ABSCESS OR THOSE NOT


RESPONDING TO MAXIMAL MEDICAL THERAPY -

36
surgery usually consists of total abdominal hysterectomy and bilateral salpingo-
oophorectomy, antibiotics and pelvic drainage as the infection involves both tubes
and ovaries.
exception - young woman of low parity who has apparent unilateral disease (normal
appearing tube and oveary on theother side) may perform a unilateral salpingo-
oophorectomy in attempt to preserve reproductive function.

Scenario # 19 - Bowel Obstruction - Large vs. Small bowel


78 y.o. female from Nursing Home with nausea, vomiting and abdominal distention and
old midline incision.

Issues:
a.) ABC’s, fluid resuscitation, tubes, labs, X-rays, physical examination.
b.) Assuming that no hernia is found, how would you evaluate this patient
radiographically? How would you treat this patient?
c.) If studies show a colon obstruction, what is the differential Dx. How should each of
these forms of colon obstruction be treated?
d.) If studies show a small bowel source, how do you decide if it is complete or
incomplete? Would a smal bowel study help? How do you manage this based on the
results?
e.) If hernia (internal or groin) is noted and loop of small bowel is strangulated and
necrotic, how should this be dealt with?
f.) What if on exploration, the pt. has tumor in the pelvis causingthe obstruction?
Bypass or ostomy?

Tubes: NGT, Foley


Labs: cbc, lytes, chem panel. CEA if concerned about a colon CA.
X- rays: flat and upright abdominal films.
Physical Exam: Ab dominal Exam for presence of ven tral hernias, peritoneal signs,
presence of groin hernias.

Radiographic Exam - #1 - rule out colon obstruction with Gastrograffin enema


- #2 - once colon obstruction is ruled out and plain films suggest
SBO, can proceed with SBFT

Complete obstruction - OR
Incomplete obstruction - as long as there are no peritoneal signs and no evidence of a
complete obstructiion, may follow nonoperative therapy for 48 hrs, then opt for
surgery.

Causes of SBO:
Cause Percentage
Adhesion 75%
Malignant tumor 8%

37
Hernia 8%
Internal 20%
External 80%
Volvulus 3%

Causes of Large Bowel Obstruction:


Cause Percentage
Cancer 65%
Volvulus 15%
Diverticulitis 10%
Hernia
Carinomatosis
Pelvic Recurrence of Rectal CA.

SURGICAL MANAGEMENT OF SMALL BOWEL OBSTRUCTION:

Bowel Viability:
Flouroscein - hyperemic , normal and fine granular patterns of viable intestines.
- patchy, perivascular or nonfluoresecent patterns of nonviable bowel.

Doppler : apply to antimesenteric border. Doppler signal within 1 cm of proposed


anastomosis predicts success.

Adhesionolysis.

Special Considerations:

Benign Stricture - can be treated with stricturoplasty. Make enterotomy, pass balloon
tipped catheter into stricture to insure luminal diameter of 1.5 to 2 cm.
Short strictures- 3-4 cm. Open stricture lengthwise 1 cm beyond its ends and close
transversely in a Heineke - Mikulicz fashion in double or single layer closure.
Long stricture - up to 12 cm. - can be used in a side to side enteroenterostomy
fashionusing a Finney technique with sutures or staples.
> 15 cm - resection.

Incarcerated Hernia -
Sites - femoral, internal inguinal, stomal, obturator or umbilical ring.

Close reduction - only performed if strangulation is not suspected.

Approach
1.) Groin incision - risks of premature reductionand loss of control of potentially
strangulated bowel.
2.) Midline
3.) Preperitoneal

38
Intusseception -
Adults - 30% of enteric intusseception and 65% of colocolonic intussecetion in adults is
secondary to a malignant process, so resection is performed over reduction in adults
with intusseception. The exception is intusseception that occurs in the early postop
period.

Malignant Obstruction
- tempting to pursue conservative approach , but aggressibve approah better as the patient
will eventually require an operation. As many as 33% of obstructions will be due to
adhesions, and 60-90% will be palliated. Definitve procedure should be
performed(bypass, resection or ostomy) whenever possible.

Postoperative Obstruction ( within first 2 weeks of surgery)


- observation is performed for first 48-72 hrs. As long as the patient is improving after
this inital period, conservative treatment is continued . 10-14 days of tube
decompression for partial obstruction is indicated before considering surgery.
SBFT can help differentiate ileus from mechanical obstruction.

SURGICAL MANAGEMENT OF LARGE BOWEL OBSTRUCTION

COLONOSCOPY - ESSENTIAL IN INITAL EVALUATION BECAUSE IT CAN


DIAGNOSE PARTIAL OBSTRUCTION AND TREAT SOME COMPLETE
OBSTRUCTION.

PARTIAL OBSTRUCTION - from CA or diveriticulitis can be diagnosed with


endoscopy and wire/ catheter can be placed thru site of obstruction. DIAGNOSE
VIA COLONOSCOPY OR CONTRAST ENEMA AND PREPARE FOR
SURGICAL RESECTION.

COMPLETE OBSTRUCTION - from CA or volvulus. Can differentiate cecal volvulus


from colonic pseudoobstruction. Can treat sigmoid volvulus prior to definitive
surgery.

Specific situations:

Cecal Volvulus - emergent surgical treatment is indicated because 30% of time


nonviable bowel is present and resection is indicated. If nonviable bowel is present,
resection with or without anastomsis is indicated. I f viable bowel present, options
include cocostomy or cecopexy or both.

Sigmoid Volvulus - initial decompression is performed with rigid colonoscope . Prep


and perform resection ASAP.

39
COMPLETE OBSTRUCTION FROM BENIGN (ISCHEMIC STRICTURE OR
DIVERTICULITIS) OR MALIGNANCY - initial decompression can be performed
with transverse loop colostomy or fully diverting colostomy.

MALIGNANT OBSTRUCTION
High risk patient, large fecal load, massive dilation --> decompressive colostomy /staged
procedure.
Dilation, ischemia,perforation, peritontis, large fecal load --> resection with delayed
anast.
Moderate dilation and fecal load, no ischemia, stable patient --> resect with anastomosis

BILIARY

Scenario # 20 - Gallstone Ileus


60 y.o. female previously healthey who presents with crampy abdominal pain, vomiting
and jaundice.

Issues:
a.) Initial workup and differential diagnosis.
b.) How do you deal with the obstructive aspect of a gallstone ileus.
c.) how do you deal with the gallbladder apect of a gallstone ileus.
d.) If the patient has an inflammatory mass making dissection of the gallbladder and
duodenum difficult, how do you deal with the jaundice?

ANSWER:

Differential diagnosis:
Acute cholecystitis
Chronic cholecystitis
Pancreatitis
Biliary stricture

Diagnosis: accounts for 15% of simple complete small bowel obstruction for those over
over 70.
Plain Abdominal Films:
Air in biliary tree - 55-60% of patients.
Show pattern of bowel obstruction
Stone seen 20% of time.

Removal of Stone - attempt to compress the mass proximally and then remove it
through a proximal enterotomy. Since stones often cause irritation and ulceration,
they may not be amenable to dislodgement, in which a limited small bowel resection
of the segment containing the stone is necessary. Check the remainder of the small

40
bowelfor additional stones. If stone is removed thru an enterotomy, the enterotomy
is closed in two layers.

Closure of Cholecystoenteric fistula - 10% risk of recurrent gallstone ileus after


enterotomy alone. Removal of gallbladder and closure of enteric portion of fistula is
peroformed.
1.) cholecystectomy
2.) Cholangiogram - to see if CBD exploration is needed.
3.) closure of small bowel component of cholecystoenteric fistula - 2 layer closure
with 3.0 vicryl Connell suture running and interrupted 3.0 silk seromuscular. Avoid
narrowing the lumen.

Unable to close cholecystoenteric fistula - If this is too hazardous or the patient is not
doing well, clean out the gallbladder of stones and place a no. 30 mushroom catheter.
In this case, leave for 3 months before tube cholangiogram and pulling catheter. Given
the advanced age of patients, the risk of another gallbaldder complication may be
worth it.

Scenario #21 - Painless jaundice S/P cholecystectomy


64 y.o. female with 1-2 week history of progressive jaundice . PMHx of
cholecystectomy.

Issues:
a.) What is the appropriate workup for this patient with painless jaundice including inital
labs and US? How would you attempt to get a detailed anatomical map of the biliary
tree?
b.) If distalCBD obstruction is noted, how would you proceed? Describe the exploration
in detail? What should you do if you are unable to get cholangiogram dye into the
duodenum
c.) If on CBDE a stone easily removed with clean cholangiogram, how should one
proceed?

# 22 - Ascending Cholangitis S/P cholecystectomy


Part #1 - Infection/Critical Care/Stabilization - 65 y.o. male S/P cholecystectomy in
E.R. complaining of dyspnea, severe epigastric and RUQ pain fever and chills.
Hypotension, jaundice tender RUQ, guarding.

Issues:
a.) What is the initial evaluation and intervention (ABC’s)
b.) What interventions are needed if the patient persists with hypotension and respiratory
distress?

41
c.) What testing is necessary (labs,US)? How can biliary duct be accessed if there is no
gallbladder for cholecystostomy? How should the patient be stabilized for the OR?
d.) If the biliary duct can not be accessed for drainage, and patient continues to
deteriorate clinically, what should be done?
e.) What shoud be done if a stable patient on the way to the OR becomes unstable
intraop?

Part #2 - Distal CBD obstruction.


a.) In a stable patient, what are options for stone that cannot be retrived from the distal
CBD?
b.) How should CBD drainage be performed?
c.) Describe in detail how to perform a CBD exploration + choledochoduodenostomy as
well as a transduodenal sphincteroplasty.

ANSWER:

PART #1 - EVALUATION AND TREATMENT OF CHOLANGITIS

ETIOLOGY OF BILIARY TREE OBSTRUCTION:


1.) common duct stones - most common etiology =70%
2.) benign biliary strictures
3.) malignancy
4.) obstructed biliary -enteric anastomosis.

A = if in respiratory distress or unresponsive/lethargic because of sepsis, intubate.


B = if stable airway, start on 02, 4-10L by facemask. Titrate up with 02 sat.
C= IV’s - 2 peripheral IV’s ASAP.
- NS at 125cc/hr.
- Start IV antibiotics
- Foley - titrate fluid to keep UO = 30cc/hr.
Persistent hypotension refractory to luid resuscitation - admission to ICU for invasive
monitoring and pressor support . Dopamine is preferable at at least low dose because
of the high risk of renal faliure in these patients.

Antibiotics - traditionally amp, gent and flagyl have been used as traditional triple
therapy until identification and sensitivity is back. Piperacillin and mezlocillin can be
used in place o f combination of aminoglycoside and ampicillin and is less
nephrotoxic. Guiding therapy specifically at antibiotics that have high biliary
concentrations has not been shown to be of use in this situation. If an
aminoglycoside is chosen, serum creatinine and peaks and troughs must be followed
closely because of the high risk of renal failure in these patients.

Laboratory -
cbc - leukocytosis usually present , but leukopenia in se vere sepsis.

42
LFT’s - serum bilirubin and alk phos are elevated in patients with cholangitis and levels
are higher in those with malignant obstruction.
- transaminases are elevated in most patients with higher levels being seen in
benign disease.
Amylase - elevated in 33% of patients, in these cases usually benign cause.
PT/PTT - may be elevated inpatients with liver dysfunction.
CA 19-9 - elevated in cholangitis , normal when resolved.
Blood Cultures - identifying causative organisms.

Radiographic Evaluation - US and CT scan. Both good for showing intra and extra
hepatic bile duct dilatation. CT scan is better for showing CBD stones and
periampullary tumors.
Cholangiography .

Biliary Decompression -

Cholangiography - if defervesces, cholangiography can be performed after afebrile for


24 hrs.

Emergent Cholangiography and Urgent Biliary Decompression - for those who do not
demonstrate clinical and lab evidence of improvment within 12-24 hrs and those
with septic shock.

How? - studies hav e shown that the complication rate, rate of retained stones , length of
dependency on a ventilator, and mortality are all higher in those who have septic
shock and have surgery compared to those who have early endoscopic drainage.

Patients with indwelling catheters - place to external drainage, treat with supportive and
antibiotic therapy.

Patients with contraindications to endoscopic biliary drainage - such as those with a BII
gastrectomy or those in whom the ampulla of Vater cannot be cannulated or those
with malignant obstruction or benign biliary stricture - PTC and external drainage are
preferable.

Patients in whom PTC is contraindicated - those with ascites, those with uncorrected
coagulopathy or those with nondilated intrahepaic bile ducts.
Patients in whom endoscopic biliary drainage cannot be performed and percutaneous
biliary drainage is contraindicated or cannot be performed - stabilization and OR for
CBD exploration.

Unstable patient intraop who had been stable preop - T-tube and get out.

PART #2 - EVALUATION AND TREATMENT OF CBD OBSTRUCTION

43
ABC’S - fluid resuscitation as indicated for hypovolemia secondary to emesis, etc.
Antibiotics - enteric bacteria should be preseumed to be present in the bile of patients
with primary common duct stones. Instrumentation of CBD during diagnosis,
elevated CBD pressures from obstruction all necessitate strting prophylactic
antibiotics. Piperacillin /mezlocillin + Flagyl as above.

Laboratory -
Alk Phos - alk phos most sensitive indicator.
Bilirubin - chance of finding stone rises as bilirubin rises.
Amylase - associated pancreatitis, usually normal
PT/PTT - may be elevated in patients with liver dysfunction
cbc- leukocytosis usually seen with cholangitis, usually normal with CBD stone.
Transaminases - along with wbc, amylase are usually normal.

RADIOLOGIC EVALUATION:
US - can show CBD dilatation, but is less accurate at demonstrating CBD stones( 15-
30%)
CT scan - also excellent for demonstrating intra and extrahepatic ductal dilatation. More
sensitive than US at detecting CBD stones. (75-90%). Also more better for
det3ecting the level of obstruction and presence or absence of pancreatic mass.

CHOLANGIOGRAPHY -
ERCP-
complications - severe bleeding and duodenal perforation occur most often when there is
a sphincterotomy peroformed.
previous BII gastrectomy , biliary stricture, malignant obstruction may make ERCP
difficult.

PTC - contraindications - ascites, uncorrected coagulopathy and narrow caliber biliary


ducts.

TREATMENT OPTIONS:
ENDOSCOPIC SPHINCTEROTOMY - diagnostic ERCP performed first. One or
two small stones can be removed thru an intact papilla. However, for more
extensive stone extraction an endoscopic sphincterotomy must be performed. This
techni que is difficult with large stones, large tortuous ducts and stones secondary
to multiple strictures
Contraindications - coagulopathy and long biliary strictures

LAPAROTOMY WITH COMMON DUCT EXPLORATION-


- conmmon duct opened after a Kocher maneuver and palpation of the intrapancreatic
duct.

44
8F or 10 F soft rubber catheter is passed and used to irrigate the duct. A fogarty catheter
is passed proximally and distally until all the stones are removed.

- choledochoscopy - can reduce the incidence of retained common duct stones to 2-4 %
without increasing mortality. Any residual stones can be removed with Fogarty
catheters.

- T tube - used in order to perform postop cholangiogram as well as to extract stones


thru the tract. if this is to be done, at least a 14 Fr. tube should be used.

- Completion Cholangiogram - has been shown to decrease incidence of retained stones


by 75%.
No passsage of contrast into the duodenum - due to spasm of the sphincter of Oddi or the
effects of narcotics. Spasm can be relieved by glucagon , the effects of narcotics on
the sphincter can be reversed by naloxone.

Laparoscopic CBDE - relative contraindication is numerous stones in the CBD. Elect


either an open CBDE or postoperative endoscopic spincterotomy.

DRAINAGE PROCEDURE :

Open Sphincteroplasty
Indications:
- sphincter stenosis or dysfunction
- possible ampullary tumor (represents an opportunity to do a biopsy)
- recurrent pancreatitis
- multiple stones in a nondilated system
- Choledochocoele

Perform kocherization of duodenum, CBDE and palpation of intrapancreatic portion of


the duct first. Identify the ampulla by passing a Fogarty catheter thru the ampulla into
the duodenum. Transverse duodenotomy made over the ampulla. Two stay sutures
(3.0 silk) are placed on either side of the ampulla and lifted forward. IF pancreatic
duct is identified at the 4 o’clock position, place a lacrimal duct probe in for
identification.
Make a small (1-1.5 cm ) incision at 11 o’clock position. Imbricate edges with
interupted 5.0 PDS. Progressively extend incision with imbrication until a Bake’s
Dilator the size of the CBD can be passed. Be sure to place an apex suture to prevent
duodenal leak. Close transverse duodenotomy transversely to prevent narrowing.

Choledochoduodenostomy
Indications:
**- primary common duct stones
- multiple stones

45
- long distal inflammatory stricture
- Perivaterian duodenal diverticulum

Can be performed in a side to side or end to end fashion:


SIDE TO SIDE: Key - place choledochotomy low on the common bile duct for
CBDE.Make incison 2-3 cm longitudinally. Place stay sutures of 4.0 silk.
Make longitudinal duodenotomy (small becasue it will stretch) over the CBD over the
first portion of the duodenum generally 7-8 cm from the pylorus. Close with single
layer of 4.0 prolene such that all sutures are placed on the outside. Place the first
stitch at the duodenal apex farthest away and then thru the inferior portion of the
choledochotomy. Complete the back row first, tying all of the sutures at the end,
starting with apex and proceeding with the order placed. Place T-tube above the
anastomosis.

Potential Problem - “Sump Syndrome” - food from the distal duodenum is caught in the
distal CBD and serves as a nidus for infection/bacterial growth. Eventually the
anastomosis or pancreatic duct can become obstructed leading to pancreatitis or
cholangitis. This can be prevented by tying off the distal duct and performing :

END TO SIDE: transection of distal CBD and anastomosing end into the side of the
duodenum using single layer 5.0 PDS and placing a T-tube above the anastomosis.
** place closed suction drains near but not at the anastomosis.

Choledochojejunostomy - can be done with limb of jejeunum in continuity or Roux en


Y , though a Roux is preferable. In either case the CBD should be transected and the
end anstomosed to the side of the jejunum to prevent sump syndrome.
Roux - brought retrocolic to provide an isoperistaltic, defunctionalized drainage of the
CBD with low tension anastomosis and low risk of reflux of gastric contents and
cholangitis. Place a T- tube above the anastomosis. If the anastomosis is too close
the hepatic bifurcation, place stents.
- jejunojejunostomy is performed 60 cm downstream.

TECHNICAL DESCRIPTION = COMMON BILE DUCT EXPLORATION:


1.) KOCHER MANEUVER - incise the lateral peritoneal band s of the duodenum.
2.) CHOLEDOCHOTOMY - rememeber = CBD lateral, hepatic art medial and portal
vein posterior. 2-3 cm incision anterior lower mid CBD distal to CBD and cystic duct
junction (make sure that the cystic duct isn’t opened) . 4.0 silk stay sutures over
incision.
3.) PALPATION OF CBD - milk stones proximally and distally thru the
choledochotomy.
4.) RED RUBBER ROBINSON CATHETER - 8-10 FR. - irrigate duct for stones.
5.) FOGARTY - if stones are not removed, pass biliary Fogarty into duct proximally and
distally and gently remove with balloon inflated.

46
6.) CHOLEDOCHOSCOPY - flexibel choledochoscope is passed ot visualize the distal
CBD to the ampulla of Vater. Passed proximally into the liver to visualize the right
and left biliary radicals. Biliary Fogarty can be passed into the CBD thru the scope to
remove stones.
7.) Stone forceps - use only if all else fails. Tend to damage duct wall.
8.) T-TUBE - use 14 FR. or large to allow postop stone retrieval.

TECHNICAL DESCRIPTION = TRANSDUODENAL SPHINCTEROPLASTY


INDICATED FOR STONE IMPACTED IN THE AMPULLA THAT CANNOT BE
REMOVED BY ANY OTHER MEANS
1.) KOCHER MANEUVER - for better exposure and to allow tension free duodenal
closure.
2.) DUODENOTOMY - longitudinal incision 3-4 cm is made over the medial aspect of
the duodenum. Palpate the stone to make sure the ampulla is in the center of the
incison.
Place duodenal traction sutures.
3.) IDENTIFICATION OF THE AMPULLA - in case of CBDE where the CBD is now
clean use a biliary Fogarty. Otherwise, feel junction of the third and fourth portions
of the duodenum and milk bile from distal CBD to locate ampulla.
The main pancreatic duct is located inferior border of the ampulla, slightly to the right
and may be deep or superficial.
4.) INCISION - after the ampulla is located, inser t a probe into the distal CBD. Incise
along probe for 2-4 cm at the 11oclock position. Should see pancreatic duct
inferiorly within first 1 cm. 4.0 or 5.0 interrupted PDS are used to apprximate the
duodenal and ductal mucosa. Place apical suture to prevent duodenal leak.
5.) IDENTIFICATION OF PANCREATIC DUCT - should be seen after stone is
removed by drainage of clear pancreatic secretions. If not use secretin, 1 mg/kg IV.
6.) CLOSURE - longitudinal incision is closed transversely like a Heineke - Mikulicz
pyloroplasty in 2 layers.

TECHNICAL DESCRIPTION = CONSTRUCTION OF ROUX EN Y

Scenario #23 - Lap Chole


Pt. with symptomatic cholelithiasis.
Issues:
a.) Describe a lap chole in detail.
b.) Describe what you would do if 6 stones were spilled?

ANSWER: Yeah , right.

HEPATIC/PORTAL HYPERTENSION

Scenario #24 - Hepatoma/Liver Mass.

47
45 y.o. female presents with RUQ mass which moves below sthe ribs on inspiration.

Issues:
a.) What is the differenital diagnosis.
b.) What is the workup (CT scan, AFP, CEA, CXR, labs). How would you rule out
hemangioma? How would you evaluate patient’s hepatic function.
c.) Assuming the CT scan shows an 8 cm clacified mass on the right lobe of the liver,
what would you do?
d.) What incision would you use? How would you evaluate the patient for resec tability?
Would you biopsy this mass?
e.) Describe in detail the performance of fa right hepati c lobectomy.
f.) Assuming bile comes from your drain on the 5th post-op day, what would you do for
diagnosis and treatment?

ANSWER:

BACKGROUND: Study done on patients with asymptomatic liver masses showed that
51% were benign hemangiomas and 17% were malignant. Unable to identify
clinical , lab or radiographic test which could differentiate a benign from a
malignant lesion.
Suggests malignancy - age > 55yrs., palpable mass, elevated alkaline phos.

DIFFERENTIAL DIAGNOSIS OF LIVER MASS:


BENIGN:
CYSTS
CYSTADENOMA
ABSCESS
ADENOMA
FNH
HEMANGIOMA
HEMATOMA
AVM OR ANEURYSM

MALIGNANCY:
HEPATOMA
METASTASIS
CHOLANGIOCA
CYSTADENOCA
SARCOMA

Labs:
cbc - thrombocytopenia and neutropenia may reflect hypersplenism.
Coags - elevated PT after administration of vit. K reflects decreased liver synthetic
function.

48
lytes
liver function tests - to assess for undrlying liver disease or measure compromise from
mass.
Transaminases - elevations from ischemia or hepatitic processes.
Alk. Phos. - elevations due to biliary obstruction
Bilirubin - elevation in absence of biliary obstruction speaks toward impairment of
hepatic function from acute or chronic disease.
Albumin - decrease may be due to decreased synthesis , malnutrition, sepsis.
hepatitis screen
AFP - elevated in those with cirrhosis or hepatitis , but rarely above 400 in absence of
HCC or germ cell tumors. Levels greater than 400 highly suggestive of HCC and
mandate investigation.
CEA - not usualy elevated in HCC, so elevations may indicate new metastatic lesion.
CXR
CT scan

RADIOGRAPHIC EVALUATION:

US - accurate at detecting focal lesions but less accurate at detecting diffuse liver disease.
Can be used to see splenomegaly, flow reversal in portal vein, ascites or other signs of
chronic liver disease. Use to detect size and cystic or solid.

CT scan - dynamic CT involves the injection of contrast and serial CT imaging.


Incremental CT - serial imaging 20 sec after injection of contrast to allow contrast to be
at its highest density as it passes thru the hepatic arteries. Delayed scans done 2-3
minutes later.
Arteriographic CT - images obtained as contrast is injected into the SMA /hepatic
artery.
** though there are patterns that can strongly suggest a malignant or benign lesions, there
are no absolute criteria. The accuracy of these techniques is often outweighed by
the complication rate from either CT arteriogram or portography.

Radionuclide - 99mTC labeled rbc scan can be used to diagnose hepatic hemangioma.

Arteriogram - can determine tyhe proximity of the lesion to major vascular structures,
anatomay of lesion itself, and the vascularity of the lesion. Gold standard for
hemangioma though can usually be ruled out by US and tagged red cell scan.
Role is less prominent than in past.

Sequence of Radiographic Exams -


#1 - ULTRASOUND - determines whether mass is cystic or solid and whether it is
single or multiple.
Cystic + Single - likely benign
Cystic + Multiple - benign cystic disease/abscess

49
Solid + Multiple - likely metastasis.
Solid + Single - needs careful further evaluation to determine if malignant.

#2 - Tagged RBC scan - if US suggests possible hemangioma, this can be ruled in or out
by tagged rbc scan.
- Sulfur colloid scan - should be used in place of tagged rbc scan if central scar noted
on US. Used to rule in or out FNH. If positive , no further workup is necessary.

#3 - Enhanced/Dynamic CT scan - perform if hemangioma screen via tagged rbc scan


is negative . CT scan is performed with CT angio or CT portography to help
delineate relationship of lesions to major vascular structures.

#4- Percutaneous Biopsy - performed if result will change therapy.


US guided or blind - performed in those in whom nonsurgical therapy is being considered
(as in the patient with cirrhosis or to confirm diagnosis of mets to liver which would
not be amenable to surgical resection.) Should not be performed if patient has a
lesion which is suspicious for HCC and is seen as reesctable by preoperative
screening.
RESECTABLE HCC, ADENOMA AND METS SHOULD BE RESECTED.

RESECTION OF ADENOMA - SURGICAL RESECTION IS THE PRIMARY


THERAPY FOR ADENOMA. Resection eliminates risk of bleeding or hemorrhage
which occurs in 20-25% of patients, and rules out HCC.

Resection for FNH - asymptomatic FNH can be treated conservatively if HCC is ruled
out as these lesions never rupture or bleed. Resection performed in large
symptomatic lesions and in those whom HCC cannot be definitively ruled out. Try
to rule out this lesion with sulfur colloid scan.

RESECTION OF HEPATOMA -
Prognosis - dependent on tumor size, tumor growth rate, functional hepatic reserve,
presence of extrahepatic spread.
CRITERIA FOR RESECTABILITY: (1) LESION MUST BE LOCALIZED TO A
PORTION OF THE LIVER SUCH THAT RESECTION WILL LEAVE WITH 20%
OF WELL VASCULARIZED LIVER (2) NO EXTRAHEPATIC DISEASE

5 YEAR SURVIVAL IN PATIENTS RESECTED WITH CIRRHOSIS = ZERO.


RESECT FOR PALLIATION - RISK OF BLEEDING, ETC. = NO. EMBOLIZE FOR
BLEEDING.

TECHNICAL DESCRIPTION = RIGHT HEPATIC LOBECTOMY


1.) INCISION = bilateral subcostal with midline extension for full mobilization.
2.) EXPOSURE = use Omni Retractor for retraction of costal margins.

50
3.) MOBILIZATION OF RIGHT LOBE - right triangular ligament is taken down and the
falciform ligament is incised, and bare area of the liver is mobilized off the
diaphragm.
The right branch of the hepatic artery is found and dissected out behind the CBD and
isolated. The right branch of the portal vein is isolated (usually on right side of
CBD). For vascular tumors, these should be clamped now.
The liver is retracted to the left and anteriorly by assistant on left (help the doctor) to
expose the IVC. The IVC is cleared off tto the level of the right hepatic vein. The
right adrenal vein is identified, ligated and divided. Small veins from the right lobe to
the IVC are ligated.
4.) DISSECTION - if the hepatic artery and portal vein branches have not been clamped,
do it now to demarcate the right liver. Incise the liver peritoneum withc autery,
divide parenchyma by finger dissection , clamping and dividing vascular structures
(vascular structures with Prolene and biliary structures with vicryl) . Dividing the
major vessels within the liver prevents inadvertant ligation of aberrant vascular
anatomy. Finally, the right hepatic vein is divided and ligated.

Scenario #25 - UGI Bleed/ Portal HTN with GE Varices.


40 y.o. female with alcoholic cirrhosis and GE junction varices referred by
gastroenterologist.

Issues:
a.) How would you assess this patient’s functional status?
b.) When patient develops hematemesis, how would you evaluate this?
Discuss resuscitation and ABC’s.
c.) How would you treat this patient? What is the appropriate dosing of Pitressin? What
are contraindications to its use? describe in detail the placement of a Sengstaken
Blakemore tube. How should the patient be further worked up once the bleeding is
under control?
f.) What are the indications for a shunt to control bleeding?
g.) What is necessary anatomically to perform a Warren shunt? What other shunts are
possibe if the splenic vein is clotted or too small for a Warren shunt?

FUNCTIONAL STATUS = CHILD’S CRITERIA FOR HEPATIC RESERVE

CRITERIA CLASS A CLASS B CLASS C


(minimal) (moderate) (advanced)
Bilirubin <2 2-3 >3
Albumin >3.5 3-3.5 <3
Ascites None Easily controlled Poor control
Neurologic Disorder None Minimal Poor control
Nutrition Excellent Good “wasting”

51
Labs:
cbc - anemia from bleeding. Leukopenia and thrombocytopenia from splenomegaly.
Coags - elevated PT after administration of vit. K reflects decreased liver synthetic
function.
lytes - abnormalities seen with cirrhosis and ascites include hyponatremia, hypokalemia,
metabolic acidosis.
BUN, Creatinine - elevated Cr . may reflect prerenal azotemia, recent GI bleed or
hepatorenal syndrome.
liver function tests - to assess for undrlying liver disease or measure compromise from
mass.
Transaminases - elevations from ischemia or hepatitic processes.
Alk. Phos. - elevations due to biliary obstruction
Bilirubin - elevation in absence of biliary obstruction speaks toward impairment of
hepatic function from acute or chronic disease.
Albumin - decrease may be due to decreased synthesis , malnutrition, sepsis.
hepatitis screen.
T&CX6U prbc.

Airway - if unstable from massive hematemesis, shock, or decreased level of


consciousness from worsening encephalopathy.
Breathing - if airway stable, start 02 at 4-10 L by facemask. Titrate by O2 saturation.
Circulation - peripheral IV’s (14-16 gauge ) X2. Resuscitate with .D51/2 NS and blood
as needed . Transfuse with FFP. Place Foley and titrate fluid to UO of at least
30cc/hr. Place art line.

UPPER ENDOSCOPY - crucial in initial evaluation of the site and cause of GI bleeding
in patient with GI bleed and portal hypertension. Bleeding usually from esophageal
varices , gastric varices or hypertensive portal gastropathy, but can occur from
Mallory Weiss or peptic ulcer as these occur more often in this population than in the
normal population.

TREATMENT OF BLEEDING VARICES


- mortality in bleeding episode is directly related to the hepatic functional reserve as those
with extrahepatic portal vein thrombosis with portal HTN do not die of bleeding
while those with decompensated cirrhosis die at a rate of 50%.

MUST CONSIDER 3 SITUATIONS: ACUTE BLEEDING, DEFINITIVE


THERAPY TO PREVENT RECURRENT BLEEDING AND
PROPHYLACTIC THERAPY FOR THOSE THAT HAVE NOT YET BLED.

ACUTE BLEEDING:
ABC’s and resusctitation, then upper endoscopy. Visualization of medium and
large varicies, even if not bleeding confirm varices as the cause of hemorrhage.
Endoscopy is crucial because it localizes bleeding site to esophagus or stomach

52
( from gastric varices or portal gastropathy) and will dictate treatment. Because of
the comorbid factors associated with cirrhosis and portal HTN, the risk of surgery
for acute bleeding is very high. This is why scleerotherapy and ligation of varices is
the mainstay of treatment if applicable. However, sclerotherapy and ligation of
varices cannot be used for gastric varices and portal gastropathy, so these patients
need early surgical therapy.

ENDOSCOPIC SCLEROTHERAPY - sclerose with sodium morrhuate or sodium


tetradecyl sulfate or ligate with rubber bands. Early therapy should be directed at
bleeding varicesand followed up at 1-2 weeks until all varices are ligated.

- endoscopic therapy = 65-90%. Standard of therapy.

When are pharmacologic or balloon tamponade used instead of endoscopic therapy?


When acute bleeding is so brisk that endoscopic therapy can not be performed.

Complications of Endoscopic Therapy - recurrent bleeding ( bleeding episode after 2


sessions) is most common. Others: esophageal stenosis and perforation, fever.

PHARMACOTHERAPY - GOAL IS TO LOWER PORTAL PRESSURE BELOW


12MMHG- THE CRUCIAL LEVEL. This is performed by splanchnic
vasoconstrictors which decrease portal infow.

Vasopressin - Bolus = 20U IV


Infusion = 0.2-0.8 U/min.
Nitroglycerin - 40-400 micrograms /minute depending on blood pressure.

Problem with vasopressin - also causes vasoconstriction of other beds causing coronary
vasoconstriction , decreased cardiac output, hypertnesion, bradycardia. Many of
these side effects are eliminated by using nitroglycerin.

BALLOON TAMPONADE - 3 SITUATIONS:


1. THOSE WITH MASSIVE HEMORRHAGE
PREVENTING ENDOSCOPIC THERAPY.
2. PERSISTENT BLEEDING AFTER ENDOSCOPIC
THERAPY.
3. TEMPORIZING PATIENTS IN TRANSIT.

Advantage - cessation of bleeding in most cases.


Disadvantage - persistent bleeding.
** because of bleeding after deflation, need to plan for definitive therapy while tube in
place.

TECHNICAL DESCRIPTION: SENGSTAKEN BLAKEMORE TUBE

53
BEFORE: Consider nasotracheal /orotracheal intubation
Attach NGT to tube above esophageal balloon
Evacuate blood from stomach with large tube.
Insert thru nose.

AFTER: Low , intermittent suction to stomach balloon


Continuous suction to NGT.
Inflate stomach balloon by 25cc increments to total of 100cc.
Pull inflated stomach balloon to GE junction and fix to nose with slight
tension using foam tape.
Add additional 150 cc to gastric balloon.
Clamp on tube to prevent loss of air from gastric balloon.
Inflate esophageal balloon to 25-45 mmHg.
X-Ray - confirm position of balloon
check Hcts q 6 hrs.
Deflate balloon after 24 hrs.
Leave in place for 24 hrs. , remove if no further hemorrhage.

EMERGENCY SURGERY FOR BLEEDING:


rare in the day of TIPPs, but present in 2 situations:
- failure to control bleeding after 48 hrs. maximal therapy.
- recurrent hemorrhage after 2 sessions of sclerotherapy.
( remember that gastric varices and portal gastropathy cannot be treated by sclerotherapy
or SB tube, only by medical treatment)
**approach - aggreesive , as mortality climbs as long as the patient continues to bleed.

OPERATIONS;
1.) ESOPHAGEAL TRANSECTION - easier for those not familiar with shunt surgery.
No clear benefit based on mortality. Vagus nerves dissected away from GE
junction.
Anterior gastrotomy, EEA stapler placed in GE junction thru gastrotomy. Silk suturew
used to tie esophagus to stapler in open position. Stapler fired. Confirm 2 doughnuts.
Close gastrotomy.
2.) PORTOCAVAL SHUNT
3.) INTERPOSITION MESOCAVAL SHUNT.
4.) DSRS - only if preop angio showing adequate anatomy and either no or medically
controlled ascites.

Bleeding Algorithm - Rikkers, p. 668.

DEFINITIVE THERAPY FOR PREVENTION OF RECURRENT


HEMORRHAGE.
PHARMACOTHERAPY - beta blockers can reduce bleeding rate by 33%, but works
best in those who have varices but have not yet bled. Compliance problematic.

54
LONG TERM SCLEROTHERAPY - benefit for long term not as clear as acute
bleeding, but because of noninvasive nature has become more popular than shunting.
Failure rates of sclerotherapy is 20-40%, so patient must be near a center where
definitive therapy can be delivered.
TIPPS - long term results for prevention are not yet known, though TIPPS tend to have
failure both early and late from graft thrombosis. Ideal for the transplant candidate.
NONSELCTIVE SHUNTS - problem - deprive liver of nutrients and hepatotrophic
hormones and shunt cerebral toxins away from liver to the brain.
TYPES: -end to side portocaval shunt.
-side to side portocaval shunt
-**interposition mesocaval shunt - good for the transplant pt.
- (CENTRAL) splenorenal shunt with splenctomy - proximal splenic vein
to left renal vein.
** use 10-14 mm graft.
use in emergency setting when TIPss not an option and with bleeding from portal
hypertnesion in patient with intractable ascites.
SELECTIVE SHUNTS - distal splenorenal shunt
Contraindications - intractable ascites - operation can worsen ascites.
- incompatible anatomy - vein smaller than 7 mm
- previous splenectomy
- clotted splenic vein
- splenic vein and left renal vein
separated by more than one vertebra on
angio.

? no portal flow to liver - not a contraindication, can be reestablished with ligation of


collaterals.

DISTAL SPLENORENAL SHUNT - ligation of coronary vein


- ligation f IMV
- removal of splenic vein from portal vein and
oversewing of splenic vein stump. The splenic vein is brought down to the left renal
vein and anastomosis performed.
PARTIAL SHUNT - portocaval shunt using a 8-10 mm interposition graft. Only recently
has this form of therapy been used consistently , so studies for efficacy are not done.

PREVENTION OF INITIAL EPISODE OF VARICEAL BLEEDING


- therapy must be minimally invasive and have low morbidity as only 33% of those with
varices will bleed.
1.) propranolol
2.) ?surgery - not effective as prophylaxis.

Scenario #26 - Pancreatitis/ Pancreatic Abscess

55
Woman with idiopathic pancreatitis develops worsening abdominal pain, fever,
tachycardia and tachypnea.

a.) ABC’s.
b.) What is the appropriate workup for this pt. with worsening AIP?
c.) What are the Ranson criteria?
d.) What should be done if fluid/nonperfused pancreas is seen on scan?
e.) What if pus is demonstrated on CT scan?
f.) If debriding necrotic pancreas and end up with dark blood oozing fromn pancreas,
what should you do?

Airway = if unstable from decreased level of consciousness from sepsis, intubate


Breathing = if airway stable, start 02 at 4-8 L by facemask and titrate by 02 saturation.
This resolves but should be treated with diuresis.

Hypoxemia - often CXR is clear with early hypoxemia. Pulmonary edema on CXR
often shows up 2-4 days after initial episode.
Circulation = treat aggressively , because if patient becomes hypovolemic and the
splanchnic circulation is compromised the pancreas may be ischemic. May need to
place Swan Ganz catheter and monitor resuscitation closely. High rate of renal
failure in this group of patients.

RANSON CRITERIA:
AT ADMISSION:
1.) Age > 55 yrs.
2.) Blood Glucose > 200
3.) SERUM LDH >300
4.) SEUM SGOT > 250
5.) WBC > 16,000

AFTER 24 HRS.
6.) HCT FALL >10
7.) BUN rise > 5
8.) Ca++ < 8
9.) Arterial p02 < 60
10.) Base Deficit >4
11.) Fluid Sequestration > 6L

DIAGNOSIS OF ACUTE PANCREATITIS:


1.) BIOCHEMICAL MARKERS - AMYLASE - single most important lab test, but still
has a significant false pos. and false negative rate. Though elevated amylase and
lipase can be used to predict the presence of pancreatitis , they cannot predict the
severity of the disease.

56
2.) RANSON CRITERIA - 4 OR MORE POSITVE SIGNS PREDICT COMPLICATED
COURSE.

RANSON CRITERIA:
AT ADMISSION:
1.) Age > 55 yrs.
2.) Blood Glucose > 200
3.) SERUM LDH >300
4.) SEUM SGOT > 250
5.) WBC > 16,000

AFTER 24 HRS.
6.) HCT FALL >10
7.) BUN rise > 5
8.) Ca++ < 8
9.) Arterial p02 < 60
10.) Base Deficit >4
11.) Fluid Sequestration > 6L

3.) DYNAMIC CT SCAN - used to visualize areas of the pancreas which are
nonperfused and likely to undergo necrosis subsequently. Early CT scan can redict
the likelihood of subsequent complications. The amount of unperfused pancreas
correlates with the number of positive Ransons signs. Should be used early to
identify areas of the pancreas that will necrose and guide surgery if it is necessary
and to predict those who will have complications based on the degree of unperfused
pancreas. Dye is given and then 5mm sections are taken at 5 sec intervals thru the
pancreas. Should be about 2/3 as bright as the aorta.

MANAGEMENT OF ACUTE INTERSTITAL PANCREATITIS:

1.) FLUID REPLACEMENT - ABC’S. Use D51/2 NS and invasive monitoring -Swan
Ganz and art line. Degree of fluid sequestration will give prediction of how severe
the pancreatitis will be.
2.) HYPOXEMIA - ABC’S. Check serial ABG’s . 75% mortality for those who require
ventilatory support.
3.) RENAL STATUS - urine output and serial BUN/CR. must be followed.
4.) MINIMIZING PANCREATIC SECRETIONS -
-NPO
-Somatostatin - no effect on course of disease.
-NGT - only with ileus for aspiration prevention, but prolonged NGT does not have
beneficial effect vs pancreas for long term.
- H2 blockers - no effect on disease.

57
5.) ANTIBIOTICS - should not be used in mild or moderate cases of pnacreatitis , but
Imipenim or Mezlocillin should be used when indicated for severe cases of
pancreatitis.
6.) NUTRITION - no evidence that using fat in TPN will ncrease the pancreatic
secretions and worsen pancreatitis disease course. Mainstay of therapy.
- enteral nutrition can be difficult because of the ileus. If given, is best in the
jejunum.
7.) HEMORRHAGE / COAGULOPATHY - transfusion with red blood cells and FFP.
8.) METABOLIC/ELECTROLYTES - usually hypokalemia, hypocalcemia. Treat with
replacement.

TREATMENT - ALGORITHM, RIKKERS, p. 824.

** almost all of those with AIP will recover with conservative therapy in 2-3 weeks.
Many of those with necrotizing pancreatitis will also recover without surgery, so
initially all are treated with conservative management and surgery based on
response.

ALGORITHM - CT SCAN - if no necrosis, observe. If clinically worse, repeat CT


scan .
- if necrosis present but pt. stable , observe.
- if necrosis present, questionable stability - percutaneous
needle aspiration. If infected, operate. If sterile, observe.
- if necrosis present and the patient not stable, operate.
TREATMENT
EARLY (first 4 days) - lavage thru a percutaneously placed catheter. 2 liters of fluid per
run is used, and fluid is evacuated, removing the toxins. The effects of this treatment
should be noted within several hours.

MID PHASE ( 4 days to 2 weeks) - percutaneous aspiration / drainage is for diagnosis


only, not for treatment. The treatment of sterile necrosis is controversial, but if
patient is unstable, should have operation.

Procedure - midline incision to allow for placement of stomas if necessary. Obtain


cultures from any free fluid. Access to the lesser sac is obtained thru the transverse
mesocolon in the avascular areas on either side of the middle colic artery. Bleeding
vessels in the mesocolon should be ligated. Dissection thru the gastrocolic omentum
can be difficult and can be associated with injury to the mesocolon, stomach or colon.
The necrosis is gently dissected away down to the viable pancreas. Strings of tissue are
ligated as these represent vessels.

BLEEDING FROM PORTAL AND SPLENIC VEIN - make initial effort to ligate
bleeders. However, as tissue is friable, this may make bleeding worse. Pack with
laparotomy sponges and close if necessary.

58
- place several large drainagel catheters that can be used for irrigation and drainage.
Options include irrigation and drainage of the pancreas and packing of lesser sac with
return to OR for repeat performance.
LATE PHASE ( > 2 WEEKS) - abscesses the most common cause of death, result from
the infection of necrosis. Diagnose via CT scan.
Percutaneous drainge - major draw back is the inability to evacuate thick , tenacious pus
and break up all the loculations . Despite these, percutaneous route can be sucessful ,
though it may take several drains. Those no responding to percutaneous drainage
should have surgery.

Scenario #27- Chronic Pancreatitis


45 y.o. male alcoholic presents with epigastric pain, new onset diabetes and diarrhea.
Pain radiates to back and pt. presents for IV Demerol.

Issues:
a.) Initial workup of pain including labs, and radiographs.
b.) What is the diagnosis if the plain X-rays show calcifications of the pancreas and stool
that is positive for fecal fat.
c.) Is ERCP indicated and why? What do you do if there is a ductal stricture in the head
of the pancreas?
d.) What do you do it there is s stricture in the mid body of the pancreas?
e.) what do you do if there are multiple strictures in the pancreas?

ANSWER:

DIFFERENTIAL DIAGNOSIS: though it may seem straightforward, it is difficult to


differentiate from cancer of the pancreas, which may also present with pain and
weight loss. CT scan and ERCP may be needed but sometimes is not enough to
differentiate.

WORKUP AND LABS/X-RAYS -


Pancreatic Insufficiency - hallmarks are steathorrhea, diarrhea and azotorrhea. Do not
occur until 90% of pancreatic function is gone. Can occur with gradual parenchymal
loss or ductal obstruction from stricture or tumor.

Diabetes - 70% of those with chronic pancreatitis will have glucose intolerance and 35%
will have frank diabetes.

LABS: OF MINIMAL VALUE. Most will have relatively normal LFT’s and amylase.
Nothing will differentiate chronic pancreatitis from CA.

RADIOGRAPHIC EVALUATION:

59
Plain X-Rays - speckled calcifications on plain x-ray seen in 30-50% of those with
chronic pancreatitis.
US - sensitive for detecting pseudocysts, dilated pancreatic duct and calcifications.
CT scan - same as US, but more sensitive. Should be done for those being considered
for surgery.
ERCP - procedure of choice for chronic pancreatitis. 92% accuracy. Will delineate
areas of stricture, dilatations, filling defects, stones, pseudocysts.
If preop ERCP cannot be done, intraop pancreatography should be done.

INDICATIONS FOR SURGERY:


PAIN - most common indication
COMMON BILE DUCT OBSTRUCTION - 10% will have persistent CBD obstruction
after acute episode has subsided from fibrotic stricture of CBD. Stricture is long,
tapered and incomplete as opposed to CA in which it is short, assymetric and
complete. If this is the only symptom, can be treated with bypass.

DUODENAL OBSTRUCTION - true diagnosis may not be made until surgery when
biopsy can rule out CA. Can be treated with bypass./gastrojejunostomy.
COLON OBSTRUCTION - rule out CA with colonoscopy, perform colon resection.
SPLENIC VEIN THROMBOSIS - can cause left sided portal HTN with varices.
Perform splenectomy.

SURGERY FOR PAIN - GOALS:


1.) Relieve pain
2.) Preserve as much pancreas as possible.

PEUSTOW PROCEDURE/DRAINAGE PROCEDURE -


Normal ductal anatomy = 4-5 mm in head, 3-4 mm in body and 2-3 mm in tail
once ductal size reaches 7-8 mm or more, a Peustow can be performed with good
chance of success. Closed suction drains are placed near the anastomosis to control
leakage and fistula. Generall y has good results for pain. Recurrent pain is an
indication for ERCP to visualize anastomosis.

PANCREATIC RESECTION - indicated for treatment of patients with ducts of normal


or decreased diameter. Resection is directed at the area of the gland that is most
effected.

Whipple - pylorus preserving for benign disease, for disease in the head.
Distal Resection - for disease in the body or tail of pancreas
Subtotal ( 80-95%) - diffuse disease with small ducts or for recureent pain after smaller
resections.

WHIPPLE - for extensive disease of the head. Indicated in situations in which there is
combined duodenal and CBD obstruction and when cancer cannot be ruled out.

60
DISTAL RESECTION - for disease localized to the tail, for those with primary disease
of the body and tail with ductal obstruction of the neck and for those with ductal
stenosis after trauma. High incidence of postop diabetes.

Scenario # 28 - Subphrenic Abscess/Pancreatic Duct Leak.


30 y.o. male with history of ITP S/P splenctomy now with fever and LUQ fluid
collection, fever, abdo pain, nausea, LUQ pain, wbc=18,000.

Issues:
a.) without the LUQ fluid collection, what is the differential dx. o f a septic patient after a
splenectomy.
b.) Appropriate workup for the LUQ fluid collection?(labs, radiographic studies)
c.) Is it appropriate to percutaneously drain this mass? (yes, to sample the amylase
content of the fluid).

Answer:

Differential Diagnosis:
1.) Subphrenic Abscess
2.) Gastric Leak ( necrosis at site of short gastric ligation)
3.) Pneumonia
4.) Empyema
5.) Pancreatic duct leak
6.) UTI
7.) OPSS
8.) Addisonian crisis

Workup:
Labs:
-cbc - elevated wbc may indicate infection
- platelet - need to evaluate platelet number before intervention
- lytes - may be indication of adrenocortical insufficiency
- amylase - pancreatic duct leak or pancreatitis of the tail of the pancreas may show
elevated amylase in serum.
- UA
- urine lytes
- urine cultures
- blood cultures
- sputum cultures

CXR - look for pneumonia , L pleural effusion, or haziness of the left hemi diaphragm.
US - look for fluid in LUQ

61
UGI with Gastrograffin - rule out leak of stomach cardia.
CT scan - fluid in LUQ, inflammation of distal pancreas.

Percutaneous drain via CT or US guidance - test for amylase content.If


pancreaticocutaneous fistula, leave drain in and treat conservatively with TPN for 4-
6 weeks. If the fistula does not dry up, obtain ERCP to evaluate for stenosis of
pancreatic duct.
Surgery - persistent sepsis and failure of the percutaneous drain to adequate drain a
(loculated ) abscess located inthe LUQ. Should be approached thru a left lateral
subcostal incision. Alternatively, abscesses in theleft subphrenic space may be
drained posteriorly through the bed of the twelfth rib. The abscess is generally
deeper than thought . Make sure the area is cleared of infected material by palpating
the aorta, esophagus at the esophageal hiatus, the caudate lobe of the liver and the
anterior margin of the left lobe of the liver.

Scenario #29 - Mass in Tail of Pancreas


74 y.o. male with mass in tail of pancreas.

Issues:
a.) Differential diagnosis and initial owrkup.
b.) Describe resection in detail.

Scenario #30 - Colon CA


70 y.o. make presents with complaint of occasional blood in his stool.

issues:
a.) Differential diagnosis and inital workup.
b.) Assuming a mid-sigmoid CA is found, what type of operation must be performed
(include vessels taken, type of anastomosis and margins of resection).
c.) Transect L ureter 5 cm from ureteropelvic junction, what do you do?
d.) Palpate a nonvisible 3 cm mass on the left lateral segment of the liver. What do you
do? What do you do if the mass is deep medial segment of the left lobe of the liver?
e.) What are recommendations for the patient with a CII lesion but nomets? BII lesion?
etc?
f.) What is the appropriate preop preparation?

Answer:

Differential Diagnosis:
1.) Colorectal CA/ Polyp/Neoplasia
2.) Hemorrhoids
3.) Rectal trauma.
4. ) Diverticulitis

62
5.) Meckel’s Diverticulum
6.) Angiodysplasia
7.) Inflammatory Bowel Disease
8.) Vasculitis
9.) Colonic or rectal varices from portal hypertnesion
10.) Ischemic colitis
11.) Amebiasis - diarrhea
12.) Clostridium Difficile - diarrhea

WORKUP:

AGE/COMORBID FACTORS - careful history and physical exam as well as preop


EKG and PFT’s if indicated. If EKG or history of chest pain/SOB or recent MI, start
with stress thallium testing.

LABS:
cbc - present hematocrit, anemia may point toward prolonged occult hemorrhage.
wbc - if infectious etiology such as diverticulitis , amebiasis or C. Diff.
LFT’s, PT/PTT, platelets - if history of alcoholism or liver failure are possible cause of
varices. Abnormal testing is not predictive of met. disease to liver.
Stool Cx - if diarrhea and infectious etiology possible
C. Diff - if diarrhea and C. diff. possible.
CEA - if colon or rectal CA suspected. may be elevated in diverticulitis, chronic or
active hepatitis, pancreatitis or renal failure.

BaE
Colonoscopy - indicated for preop visualization of lesion and biopsy. Once histologic
diagnosis has been made , the remainder of the colon should be surveyed for
synchronous CA (5%) or synchronous adenomatous polyps. Either BaE or
colonoscopy is acceptable, but colonoscopy has the benefit of allowing biopsy of
other lesions to rule out synchronous CA and perform polypectomy for polyps
outside the desired resection area.

CT scan - colon CA - not indicated as surgery will be performed for pallitation in event
of metastatic disease.
- Rectal CA - indicated if there is a question of resectability or involvement of
contiguous structures such as the bladder, prostate and bony pelvis.

STAGING - DUKES CLASSIFICATION


Stage Description
A Limited to mucosa
B1 Limited to muscularis propria, nodes negaive
B2 Through muscularis propria, nodes negative

63
B3 Through bowel wall, contiguous organ involvement, nodes
negative
C1 Limited to muscularis propria, nodes postive
C2 Through muscularis propria, nodes positive
C3 Adjacent organ involvement, nodes positive
D Mets

Preop staging - much more important for rectal CA than colon CA.

SURGICAL THERAPY AND CONSIDERATIONS.


ANATOMY -
Cecum, Ascending Colon , Proximal Transverse Colon - midgut
Arterial Supply - SMA-> ileocolic, right, middle colic.
Venous drainage - SMV - > drains into portal vein.
Distal transverse colon, descending colon, sigmoid - hindgut
Arterial Supply - IMA-> middle, left colic , sup. hemm.
Venous drainage - IMV -> drains into splenic/portal vein
Rectum
Arterial Supply - middle rectal arteries (hypogastric br.)
- lower rectal arteries ( pudendal br.)
Venous drainage - middle + inferior rectal veins - > IVC.

Lymphatic Drainage - parallels vascular anatomy with 4 levels of drainage - > epicolic,
paracolic, intermediate and principal nodes --> cysterna chyli.

ONCOLOGIC PRINCIPLES OF COLON RESECTION:


RIGHT HEMICOLECTOMY - for lesions of the cecum and ascending colon. Ligate
the ileocolic and right colic arteries. Divide the bowel at the distal ileum and at the
mid- tranvese colon.
EXTENDED RIGHT HEMICOLECTOMY - for lesions of the hepatic flexure or
proximal transverse colon. Ligate the ileocolic and right and distal left
colic/marginal artery of Drummond. Divide the bowel at the distal ileum and at
theproximal/ mid-descending colon.
LEFT HEMICOLECTOMY - for lesions of descending colon. Ligate the marginal
artery beneath the distal transverse colon and left colic artery. Divide the bowel at
the mid/distal tranverse colon and at the proximal sigmoid colon.
EXTENDED LEFT HEMICOLECTOMY - for lesions of the distal tranverse colon and
the splenic flexure. Ligate the marginal artery beneath the mid/distal tranverse colon
and the left colic , sigmoid and upper rectal branches. Divide the bowel at the
mid/distal transverse colon and at the distal sigmoid /upper rectum.
TRANSVERSE COLECTOMY - for lesions of the mid tranverse colon. Ligate the
marginal artery at the hepatic flexure, distal middle colic and the marginal artery at
the splenic flexure. Divide the bowel at the hepatic and splenic flexure.
SIGMOID COLECTOMY - Ligate the proximal branches of the left colic artery ( left
colic, sigmoid and upper rectal branches). Divide the bowel at the distal desceding

64
colon and at the upper rectum and mobilize the splenic flexure as needed for a
tension free anastomosis.

No touch - no data.
Intraluminal spread - no data.

SPECIAL CIRCUMSTANCES:
1.) SYNCHRONOUS CA OR POLYPS - should be considered for subtotal colectomy
depending on the distance between lesions and the number of lesions. If patient has
multiple polyps or CA’s should have subtotal colectomy to facilitate surveillance of
remaining colon.

2.) OBSTRUCTING CANCERS:


OBSTRUCTIING RIGHT COLON CANCERS - can have right hemicolectomy and
ileocolic anastomosis.
OBSTRUCTING LEFT COLON CANCER - OPTIONS:
- if tumor can be intubated endoscopically, a wire can be passed thru the lumen and a
tube be passed over the wire into the proximal obstructed bowel and allow the bowel
to be decompressed and mechanically prepped for surgery. This should be used for a
partial obstruction or near complete obstruction to ensure adequate mechanical
preparation of the colon for safe resections and anastomosis.
- resection with primary anastomosis and colonic cleanout with or without protective
colostomy.
- primary resection and colostomy with Hartman’s pouch for second stage
anastomosis.
- subtotal colectomy with primary anastomosis.
- decompressive left colon loop colostomy followed by formal resection of tumor
and
primary anastomosis.

3.) ADJACENT ORGAN INVOLVEMENT - appears to be a variant of colon CA


which grows to large size and is locally invasive that is less aggressive from a
metastatic potential point of view. Because of this, these lesionsare potnetially
curable even when they involve adjacent structures.

LIVER METS - those which can be resected with clear margins by wedge resection
should be resected and those lesions that will require a formal hepatic resection
should be postponed because fothe magnitude of the operation. Also allows use of
intraop ultrasound.

4.) OOPHORECTOMY - indicated for mets to the ovary and for direct extension of the
tumor to the ovary. Can also offer prophylactic oophorectomy for postmenopausal
women.

65
ONCOLOGIC PRINCIPLES OF RECTAL RESECTION:

APR - standard of therapy for rectal CA. En bloc resection of rectum, mesorectum,
perineal skin and spincters. Downside is the permanent ostomy and the bladder and
sexual dysfunction.

LAR - “ 5 cm distal margin” rule has proved wrong since tumor rarely spread further
than 2 cm. This has allowed for some distal mid third and proximal lower third
tumors to be resected by LAR as opposed to APR.

COLOANAL ANASTOMOSIS - can be use for very low, invasive rectal cancers (a very
low LAR which utilizes a colonic J pouch for improved continence). Often
recommended a protective colostomy.

LOCAL EXCISION - GOAL IS TO REMOVE FULL THICKNESS DISK OF


RECTUM WITH 1-2 CM MARGIN OF NORMAL RECTAL TISSUE BEYOND
THE TUMOR.
Anterior Wall Lesions - prone or jacknife prone
Posterior Wall Lesions - high lithotomy position.
Advantage over laser /thermal ablation or radiation- entire specimen is available for
histlogic analysis.
Criteria for local excision:
1.) distance of less than 8 cm from anal verge.
2.) Size of less than 3-4 cm.
3.) mobile, nonulcerated, noncircumferential
4.) well to moderately differentiated
5.) T1 or T2 lesions
6.) Diploid by flow cytometry.

ADJUVANT THERAPY -
RADIOTHERAPY -
COLON - indicated for tumors that extend thru wall posteriorly or laterally.
RECTUM - NEOADJUVANT - should be sued for tumors that are fixed or unresectable
at the time of evaluation for surgery. Can convert marginally resectable to resectable
lesions and can permit sphincter sparing operations for tumors that would have
required
APR.
- ADJUVANT - since pelvic recurrence of CA is a catastrophic event, therapy
which can decrease local recurrence is desirable - and radiotherapy has been shown to
decrease chances of local recurrence. Used for B2 and B3 (thru muscularis propria or
thru wall) and for C2 and C3 (thru muscularis propria or thru muscel wall) .

CHEMOTHERAPY -

66
COLORECTAL - should be offered 5-FU + levamisole based chemotherapy in
patients with Dukes stage C colorectal cancers based on improved survival in these
patients vs. controls.

FOLLOWUP - not only for recurrences but also for metachronous polyps and cancers.
COLONOSCOPY-
Preop perforation or obstruction- soon after surgery to evaluate the rest of the colon.
REST - CXR and colonoscopy at 1 year. If negative, every 2-3 years. If polyps present,
remove and repeat annually until negative.
CEA - at 6 weeks and then every 3 months for 2 years.

Scenario #32 - Rectal Villous Adenoma with High Grade


dysplasia.
45 y.o. male who is found on screening sigmoidoscopy to have a villous adenoma with
high grade dysplasia 5 cm in size 6 cm from the dentate line.

Issues:
a.) Appropriate workup of patient with villous adenoma?
b. ) How would you remove this lesion if the remainder of colon is evaluated as normal.
c.) How often should patient be followed for this lesion? How often should you follow up
a patient who has had colon CA?

ANSWER - LAR OR PIECEMEAL COLONOSCOPIC POLYPECTOMY WITH


FOLLOW UP COLONOSCOPY AND BIOPSY IN 2-3 MO.

Scenario #33 - Cecal Adenoma S/P Sigmoid Colectomy


50 y.o. S/P sigmoid colectomy for Duke’s BII CA. On F/U colonoscopy found to have
cecal tubulovillous adenoma without evidence of dysplasia.

Issues:
a.) What sort of resection should be entertained in this patient?
b.) Follow Up?

ANSWER - ENDOSCOPIC POLYPECTOMY IF POSSIBLE. IF NOT,


CELIOTOMY AND LIMITED RESECTION.

NEOPLASTIC COLORECTAL POLYPS -

CARCINOMA IN SITU = SEVERE DYSPLASIA= ADENOMA WITH ATYPIA =


FOCAL CARCINOMA = INTRAMUCOSAL CARCINOMA.

67
IN SITU - DO NOT METASTASIZE
INVASIVE - DO METASTASIZE

POLYPS
**thelarger the polyp and the wider the base (sessile>>pedunculated), the greater
the chance of malignant potential.
- small polyps (< 1 cm) don’t present with malignancy often, but can present with
bleeding.
-CANCER CAN OCCUR IN ANY POLYP, BUT MOST OFTEN IN VILLOUS.

TUBULAR ADENOMA - more frequent than villous and increase in age. Tend to be
multiple in contrast to malignant polyps which are more often singular. May
occasionally present with bleeding, but generally are asymptomatic and are found on
contrast studies of the colon or colonoscopy.

VILLOUS ADENOMA - like cancerous polyps, tend to be single as opposed to multiple.


Have a greater propensity to harbor an in situ or invasive carcinoma. In one series,
over half of villous adenomas over 2 cm were histologically malignant.
Represent almost 10 fold increase in frequency of CA over tubular adenomas
less than 1 cm in diameter.
- because of large surface area may initiate bleeding , cause obstruction, cramping and
obstipation. May occasionally produce a diffuse watery diarrhea that can lead to
hypokalemia.
- TEND TO OCCUR IN THE CECUM, SIGMOID AND RECTUM.
- invasive malignancy occurs in 20-30% for polyps > 2cm diameter.

TREATMENT OF COLORECTAL POLYPS


- simple polypectomy is considered appropriate therapy for tubular or villous
adenomas
that contain foci of in situ carcinomas (absence of penetration of the muscularis mucosa
and invasion of the submucosa). SHOULD BE COMPLETELY REMOVED AT
COLONOSCOPY OR SIGMOIDOSCOPY. DO NOT METASTASIZE.

- if invasion of CA/malignant cells is deep to the muscularis mucosa, these patients


should be treated as invasive carcinoma of the colon. The risk of invasion deep to the
muscularis mucosa is greater with sessile than pedunculated polyps. Because of
chance of incompletely excising tumor endoscopically and not treating potential
lymph node mets, the failure rate of treating sessile polyps with invasive CA by
endoscopic polypectomy is 21%. With pedunculated polyps , the risk of lymph node
mets is lower and the risk of incomplete excision is lower, but the rate of failure is
still 10%. For pedunculated polyps in which the malignancy has reached the neck of
the stalk, a standard resection should be peroformed. If CA is limited to the head of
a pedunculated polyp, risk of mets is only 3%.

68
VILLOUS ADENOMAS - SHOULD BE COMPLETELY EXCISED BY
COLONOSCOPY OR TRANSANAL EXCISION IF POSSIBLE.
- often these tumors are large and pedunculated and cannot safely be excised via a
colonoscopic polypectomy. If the polyp has been incompletely excised or biopsied,
and lie above the peritoneal reflection, a celiotomy and excision of the entire lesion
(segmental resection ) should be performed.
- tubulovillous adenomas lie between tubular and villous in malignant potential.

TREATMENT OF POLYPS:

TUBULAR
DYSPLASIA/CARCINOMA IN SITU - COLONOSCOPIC/OPEN COMPLETE
SIMPLE POLYPECTOMY

TUBULAR PEDUNCULATED WITH INVASIVE CA AT HEAD -


COLONOSCOPIC/OPEN COMPLETE SIMPLE POLYPECTOMY IF:
1.) CA is not highly undifferentiated.
2.) blood vessels or lymphatics do not contain CA.
3.) stalk is long enough to allow histologic evaluation and malignancy does not
extend to neck of adenoma.
4.) CA does not extend to the base of the stalk
5.) entire polyp is not CA.

TUBULAR SESSILE WITH INVASIVE CA - RESECTION OF COLON+


MESENTERY

VILLOUS
- all villous adenomas should be removed.

DYSPLASIA/CARCINOMA IN SITU - COLONOSCOPIC/OPEN COMPLETE


SIMPLE POLYPECTOMY

VILLOUS PEDUNCULATED WITH INVASIVE CA AT HEAD -


COLONOSCOPIC/OPEN COMPLETE SIMPLE POLYPECTOMY IF:
1.) CA is not highly undifferentiated.
2.) blood vessels or lymphatics do not contain CA.
3.) stalk is long enough to allow histologic evaluation and malignancy does not
extend to neck of adenoma.
4.) CA does not extend to the base of the stalk
5.) entire polyp is not CA.

VILLOUS SESSILE WITH INVASIVE CA - RESECTION OF COLON AND


MESENTERY

69
KEY - biopsy or partial polypectomy does not reliably assess the presence or
absence of malignancy, as well as the extent of the spread. Need the whole polyp
to rule in or out CA, and to tell the level of invasion if CA.

Pathologic diagnosis : invasion to muscularis mucosa, predominant adenoma type,


presence of lymphatic or vascular invasion, adequacy of margin of resection.

FOLLOW UP - EVERY 2 YEARS IF MORE THAN ONE ADENOMA REMOVED


- EVERY 3 YEARS IF THE PATIENT HAS HAD ONE INDEX
LESION.

SUBTOTAL COLECTOMY - aggressive approach may be indicated in high risk patients


with previous complete endoscopic polypectomy for CA and found to have villous or
tubulovillous adenoma.

Scenario # 34 - Low Rectal CA


80 y.o. male with rectal mass at 3 cm from anal verge, about 3 cm in diameter. Bx is
positive for adenoCA.

Issues:
a.) How else would you evaluate the colon?
b.) Describe the technique of a transanal excision of a mass? What are options for the
anesthetic?
c.) What if further workup revealed CAD, angioplasty performed - would this change
your mind concerning options for treatment? What if the patient had had an MI 2
months ago? What if the pt. had had a recent MI and now has a near obstructing
rectal CA?
Role for preop radiation to shrink tumor?
d.) APR/LAR performed and positive nodes found, what chemo should be offered?
e.) LAR is performed for 2x4cm mass at 5 cm, upon stapling the anastomosis, an
incomplete ring is found and posterior defect is confirmed by insuffaltion - what do
you do?Do you place a colostomy?

LOCAL TREATMENT FOR DISTAL/LOW RECTAL CA.

RATIONALE FOR LOCAL TREATMENT OF RECTAL CA -


1.) Local treatment is confined to that group of patients with CA limited to rectal and
perirectal tissues - it is this group of patients that will not benefit from a more radical
resection. Since APR only benefits some of those with early Duke’s C disease , if all
patients are treated with this therapy, only 10-15% will benefit from resection over
local treatment.

70
2.) Local treatment is also benficial for those who are symptomatic from a rectal tumor
but are known to have distant disease and are candidates for palliation but not cure.
When symptoms can be eradicated by local treatment an d the potential morbidity,
mortality and inconvenience of a more extensive procedure is avoided. However,
local treatment for disease that has spread to the nodes is inadequate therapy.

3.) Local treatment also avoids unnecessary surgery in patients with significant comorbid
disease , who might die from general anesthesia or celiotomy.

PROBLEM - in patient without distant disease, how do you select patients with Duke’s A
or B disease who would benefit from local treatment prior to surgery.

ANSWER - accurate preop staging. Problem is staging for presence of nodal disease.
Methods - digital rectal exam.
- CT scan is no better than digital rectal exam
- Endorectal US - negative predictive value ( ability to predict tumor that are not
thru wall or into nodes) . 70-95% for penetration of wall ( vs. 40-60% for CT).
80-90% for nodes ( 50-70% for CT).

LOCAL TREATMENT OPTIONS:

LOCAL EXCISION: should be limited to tumors that are 3 cm or less in diameter.


- should not be used for poorly differentiated histology.
- should not be used for tumors which are more than 1/3
circumference.
Technique - full thickness excison of rectal wall including 1-2 cm beyond the tumor. to
minimize bleeding, shoudl use cautery for excision. Proctoscopic examination
should be performed at the end of the procedure to assume the lumen has not been
compromised.
Results - poor prognostic signs include positive margins, poorly differentiated histology
and depth of invasion.

INTRACAVITARY RADIATION:
Limited tolerance of tissues for external beam radiation does not make this viable for
curative treatment of rectal CA. Use a short probe to deliver high intensity dose of
radiation with limited tissue penetration.
Criteria for treatment - well differentiated or moderately well differentiated.
- limited to bowel wall on preclinical assessment.
- maximal diameter of 4.5 cm.
- ulcerated lesions within 9 cm of verge, polypoi d lesions within
12 cm of verge.
Results - local control of the tumor with 92.5% of patients.

71
ELECTROCAUTERY -
Criteria - similar for histology, circumference and size for
transmural excision.
- should be limited to tumors at or below 10 cm. to avoid
peritoneal soiling for transmural cauterization. Tumors below 10 cm can be
cauterized thru he rectal wall and attempts should not be made to close a transmural
defect or protect it with a colostomy.
Results - 5 yr. survival rates of 50-65% when treated with cautery alone. Bleeding is a
common complication (7%).

LOCAL RECURRENCE -
can occur in 6-44% of patients receiving local treatment for low rectal CA. Though a
second trial of local treatment can be tried for recurrence, LAR or APR are the option
most chose. Salvage rate is 21-87% for these procedures.

ADJUVANT THERAPY - promising for those who receive adjuvant radiotherapy after
either local excision or electrocautery.

ROLE OF PREOP AND POST OP RADIOTHERAPY

PREOPERATIVE THERAPY - studies show that pelvic recurrence rates are lower
than those treated with surgery alone . There is still questions as to whether there is
imporved 5 year survival with this treatment. Should be focused on T3 and T3N1
lesions as well as T4(circumferential, near obstructing ) lesions which will have the
highest rate of locoregional failure and the best chance of shrinkage.

SELECTION - rectal lutrasound is best for determining depth of tumor invasion but not
quite as good for detecting local nodes. T3 lesions ( full thickness of wall) and T4
lesions ( involving adjacent structures) , near obstructing lesions and circumferential
lesions should be selected for preoperative radiotherapy.

POSTOPERATIVE RADIOTHERAPY- incidence of locoregional failure is reduced


when radiotherapy is used with surgery as compared to surgery alone. Recomended
for T3N0 lesions ( thru submucosa into subserosa) or for any TN1lesion. When
accompanied with 5-FU and levamisole lower locoregional recurrence to 10-14%.

FOLLOW - UP AND POSTOPERATIVE SURVEILLANCE -


RATIONALE - goal is to find tumor when it is still resectable as most tumor recurrences
are found when unresectable leading to patients death.
- almost all recurrences occur in the first 5 years after rescetion

Scenario -#31 - H/O Colon CA with Rising CEA.

72
50 y.o. male is S/P sigmoid resection for BII CA. Found on routine F/U screening
examination to have rising CEA.

Issues:
a.) What is your initial workup?
b.) What do you do if metastatic workup is negative and patient is found to have a cecal
CA on colonoscopy?
c.) What do you do if the metastatic workup is positive for liver and lung mets and there
is a cecal CA on colonoscopy?
d.) What do you tell the patient if the metastatic workup is negative but the path comes
back positive for lymph nodes after a total colectomy and ileo-proctostomy is
performed?
e.) What do you tell the patient regarding the frequency of bowel movements?

ANSWER:

CAUSES OF ELEVATED CEA: cigarette smoking, emphysema, bronchitis, peptic ulcer


disease, gastritis, hepatitis, obstructive jaundice, pancreatitis, diverticultis, renal
failure.
- because of the number of causes of elevated CEA there is no role for using CEA as
a screening test for large populations. Cannot distinguish between benign,
noncolorectal cancer and colorectal conditions.

PROGNOSTIC IMPLICATIONS - 25-75% of patients with colon cancer will have


elevated CEA. There is a positive correlation of the incidence of positivity and the
magnitude of the elevation of CEA with advancing Duke’s staging.

- CEA levels should normalize 4-6 weeks after resection. Persistent levels implicate an
incomplete resection and ominous prognosis. In order for CEA level to have use as
a prognostic factor , it must return to normal after surgey.

WHEN TO MEASURE:
1.) PREOP
2.) POSTOP - 4-6 WEEKS
3.) EVERY 3 MONTHS FOR 2 YEARS
4.) EVERY 6 MONTHS FOR NEXT 3 YEARS.
**vast majority of recurrences will recur within 5 years. Most will recur in 2 years.

STANDARD FOLLOWUP :
1.) CXR, LFT’S - EVERY 6 MONTHS
2.) FOLLOW- UP VISITS - history, abdominal exam, hemoccult, CEA.
3.) COLONOSCOPY - ANNUALLY FOR AT LEAST 3 YEARS

?ABNORMALITIES -
- hemoccult positive - colonoscopy

73
- CXR nodule- chest CT
- LFT’s - abdominal CT.
- elevated CEA
upper limit of normal - 2.5 in nonsmokers - consider recurrence until
- 5.0 in smokers - proven otherwise.
>10 - recurrent colorectal CA.

CASE #1 - ASYMPOTMATIC WITH ELEVATED CEA


first - CT scan of chest , abdomen and pelvis
- colonocopy
- bone scan

second - if colonsocopy positive and others negative. - laparotomy


- if bone scan positive or CT shows distant spread - ?palliation
- if none of the above positive - second look laparotomy

CASE #2 - SYMPTOMATIC PATIENT WITH ELEVATED CEA


first - stage disease with CT scan of chest, abdomen and pelvis, bone scan and
colonoscopy.

second - based on symptoms and preop staging ,options are palliation via surgery,
radiation , chemotherapy and any combination of these.
- other options are re-resection for cure with or without the addition of
chemotherapy /radiotherapy.

EXPLORATORY LAP FOR ELEVATED CEA - midline incision . Inspect visceral and
parietal peritoneum for tumor implants. Bimanually palpate the liver. Use intra-op
ultrasound to more thoroughly examine the liver. Examine the retroperitoneum in
clockwise fashion fromthe GE junction. Run the GI tract.
- can have patients undergo preop testing with radiolabeled murine monoclonal
antibody after pretest sensitivity to Ab. Allows localization of recurrent tumor with
exception of liver which takes up radioactivity nonspecifically. Also allows scanning
intraop with gamma probe, biopsying areas with increased counts.

COLONIC RECURRENCE - requires resection of anastomosis with primary


anastomosis. Should take 5 cm margin (ideally 10 cm on either side) to prevent
further recurrence. Wide resection of mesocolon should be taken to prevent further
recurrence. Any adjacent structures are taken en bloc.

LIVER METASTASIS - wedge resection can be performed if possible over lobectomy,


bilaterally if necessary. Minimum of 1-2 cm of normal tissue as margin. Prior to
proceeding with lobectomy, should make certain that there are no implants in pelvis,
omentum or peritoneum.

74
PELVIC RECURRENCE - if pain is present as symptom the tumor is locally advanced
and usually incurable. If tumor extend s into sacrum or bladder, en bloc resection is
performed. If recurrence has positive margins by gross or microscopic criteria,
brachytherapy needles are inserted.

Scenario #35 - Simultaneous Colon and Liver Masses.


70 y.o.male with sigmoid CA undergoing resection; noted intraoperatively to have a 4 cm
mass in the left lobe of the liver?

Issues:
a.) How would you evaluate the liver lesion? Would you perform a biopsy?
b.) If the biopsy came back positive for CA, how would you proceed?
c.) Explain the rationale for treating colorectal mets to the liver including indications,
contraindications and technique.
d.) Discuss the anatomic limits, technique and vascular anatomy for major liver
resection.

ANSWER:

- of all patients with disseminated colorectal cancer, 15-20% will have lesions which are
localized to the liver, and a smaller percentage will be surgical candidates.
Extremely highly selected group of patients.
Rationale for treatment = 2 yr. survival of patient with unresected mets is 45%. 5 yr.
survival is zero. Those that do not undergo resection will die of disease.

SITUATION #1 - LIVER METS DISCOVERED AT TIME OF COLON


RESECTION.

MUST EVALUATE; size and location of mets, status of portal and celiac nodes,
presence or absence of extrahepatic disease.

IF RESECTABLE - resect the colon /rectal primary and then decide whether to resect
with this opeation or wait.
CRITERIA FOR SIMULTANEOUS RESECTION OF LIVER TUMOR WITH
COLON RESECTION:
1.) single met which can be removed by wedge resection.
2.) colon/rectal resection has gone flawlessly .
3.) adequate incision.
4.) surgeon comfortable with resection under circimstances.

SITUATION #2 - LIVER METS DISCOVERED ON FOLLOWUP;


3 GROUPS -
1.) LIVER METS + PORTAL/CELIAC INVOLVEMENT -
mets in celiac and portal nodes are result of spread from liver. Dismal 5 yr. survival .

75
Do not resect.
2.) LIVER METS + EXTRAHEPATIC DISEASE - 2 situations in which
extrahepatic disease does not rule out liver resection - simultaneous lung mets and
anastomostic recurrence. In the case of anastomotic recurrence, re-resect colon/rectal
tumor and wait 4-6 weeks for liver resection. If other extrahepatic disease is found,
do not resect.
3.) LIVER ONLY DISEASE - DO NOT RESECT MORE THAN 4 METS.
The survival after resection decreases with increasing number of mets. The upper
number
of mets is arbitrary. In resecting mass, obtain a 1-2 cm margin of normal tissue if
possible. Inability to obtain a 1 cm margin does not contraindicate resection.

EVALUATION OF LIVER MASS - either synchronous mass that will be resected at a


later date or a metachronous mass( recurrence) found on followup needs to have
workup for extent of metastatic disease.

Sites of Recurrence - anastomotic recurrence, liver and lungs


Tests for recurrence - colonoscopyBaE, CEA, abdo/pelvic CT, LFT’s, chest CT.

Colonoscopy /BaE- evaluate the anastomotic site as well as the remainder of colon for a
metachronous primary.

Abdo/pelvic CT - should be used to evaluate the number and location of liver mets as
well as portal and celiac nodes and the presence of extrahepatic disease within the
abdomen.

LFT’S - elevations of alk phos and bili may indicate more mets than seen on CT or
presence f cirrhosis. Unusual for mets and cirrhosis to present at the same time- can
remove 75% of normal functioning liver.

Chest nodule - does not rule out liver resection. May either be a benign granuloma or
met, and one or two colorectal mets to the lung can be resected either at the same time
or in a staged manner.

BIOPSY OF LIVER MASS - INTRAOP AT TIME OF COLON RESECTION --> YES


- METACHRONOUS LESION -----> N O. ( positive biopsy
does not change plan and negative biopsy is seen as inadequate biopsy)

Preop Prep - usual bowel prep - if anastomotic recurrence is found during exploration of
for liver resection, the anastomotic recurrence is resected and the liver resection is
delayed.

SURGICAL RESECTION -

76
INCISION - plan for a bilateral subcostal, but start with a right subcostal incision for
exploration.
EXPLORATION - explore the abdomen for extrahepatic disease. If none present,
proceed with taking down of the falciform ligament for intraop ultrasound.
INTRAOP ULTRASOUND - will change operative plan 20% of time , either from
finding additional mets or thru defining the relationship of the mass to the hepatic
veins.
RESECTION - if resectable, extend incision to bilateral subcostal

TECHNICAL DESCRIPTION = RIGHT HEPATIC LOBECTOMY


1.) INCISION = bilateral subcostal with midline extension for full mobilization.
2.) EXPOSURE = use Omni Retractor for retraction of costal margins.
3.) MOBILIZATION OF RIGHT LOBE - right triangular ligament is taken down and the
falciform ligament is incised, and bare area of the liver is mobilized off the
diaphragm.
The right branch of the hepatic artery is found and dissected out behind the CBD and
isolated. The right branch of the portal vein is isolated (usually on right side of
CBD). For vascular tumors, these should be clamped now.
The liver is retracted to the left and anteriorly by assistant on left (help the doctor) to
expose the IVC. The IVC is cleared off tto the level of the right hepatic vein. The
right adrenal vein is identified, ligated and divided. Small veins from the right lobe to
the IVC are ligated.
4.) DISSECTION - if the hepatic artery and portal vein branches have not been clamped,
do it now to demarcate the right liver. Incise the liver peritoneum withc autery,
divide parenchyma by finger dissection , clamping and dividing vascular structures
(vascular structures with Prolene and biliary structures with vicryl) . Dividing the
major vessels within the liver prevents inadvertant ligation of aberrant vascular
anatomy. Finally, the right hepatic vein is divided and ligated.

TECHNICAL DESCRIPTION = LEFT HEPATIC LOBECTOMY


1.) INCISION = bilateral subcostal with midline extension for full mobilization.
2.) EXPOSRE = use Omni retractor for retraction of costal margins.
3.) MOBILIZATION OF LEFT LOBE - take down the left triangular ligament by
opening the gastrohepatic ligament and mobilizing the left triangular ligament to the
IVC. Left phrenic vein is identified and divided. (Careful for theleft phrenic nerve
which lies across the vein to be divided.) This mobilization will leave you at the
IVC/left hepatic vein junction. The falciform ligament is divided to the IVC(done
already for intraop US). Retract the left lobe to the right (help the doctor). The IVC
is seen by retracting yhre caudate lobe anteriorly and to the right. The adventitia is
opened up over the IVC, opening up the posterior suprahepatic cava for full
mobilization of the IVC.
4.) HILAR VASCULAR DISSECTION - hilar dissection is done by dentifying the
bifurcation of the hepatic artery, portal vein and bile duct. Left portal vein is divided
by clamping and oversewing using a running prolene. The left bile duct and left

77
hepatic artery are divided. The liver substance is divided from the gallbladder fossa
toward the junction of the middle hepatic vein/IVC.

Scenario #36 - Lower GI Bleed


70 y.o male in ER who 2 hrs earlier had passed 100 cc of bright red blood per rectum.

a.) ABC’s including type and cross.


b.) Initial evaluation and exam (including flex sigmoidoscopy). NGT?
c.) Appropriate workup for localization of a lower GI bleed (know rates of bleeding for
detection by arteriogram and bleeding scan). When is colonoscopy indicated?
Are there any blood tests that may be helpful?
d.) What are the surgical options if the bleeding source cannot be localized?

ANSWER:

A= if airway unstable becasue of hypotension, intubate.


B = in absence of unstabel airway, place on 02 4-6L by facemask to increase 02
carrying capacity.
C= peripheral IV’s - 16 gauge or larger. Resucitate with LR. Place Foley catheter and
titrate fluid for a UO of 30cc/hr. Place art line. Admit to ICU.

LABS:
cbc - Hct and platelet level. Cirrhotic may be thrombocytopenic.
Coags - PT may be elevated in patient with cirrhosis.
LFT’s - may indicate chronic liver disease and cirrhosis.
T&C X 4U prbc’s.

Differential Diagnosis of Massive Hematochezia


Cause FREQUENCY
Angiodysplasia 30%
Diverticulosis 17%
Neoplasm 11%
Colitis, ulcers 9%
Small bowel lesions 9%
Mixed Upper GI source 11%

EVALUATION OF LOWER GI BLEEDING


NGT - to rule out 11% chance f UGI bleed. If positive, proceed to upper endoscopy. If
negative, leave in in case needed for colonscopy.
ANOSCOPY/SIGMOIDOSCOPY - should be performed immediately after digital
rectal exam to rule out hemorrhoids, anal fissure, rectal ulcers, trauma, or low rectal
CA. Anal lesions can be controlled with locl lidocaine/epi. Hemorrhoids can be

78
banded. Decison made to go to OR for bleeding low rectal CA. If site not located
and patient hemodynamically stable:

DIAGNOSTIC/THERAPEUTIC COLONOSCOPY - problem - unprepped colon will


not yield a good exam.. Go-Lytely thru NGT at a rate of 1L/hr until bowel effluent
clear. Sedate. If bleeding is located and cannot be cauterized, use flouro to localize
the lesion. Examine the terminal ileum for evidence of blood or Crohn’s disease.

If hemodynamically unstable: after anoscopy , proceed to upper endoscopy quickly to


rule out upper GI source. If the upper endoscopy is negative, proceed to mesenteric
angio.

MESENTERIC ANGIO: for bleeding at 1cc/min. locates lesion, can be therapeutic


and directs surgical intervention. 75% of all lower GI bleeding can permanently or
temporary controlled angiographically. Particularly useful in patient with massive
bleeding, no known site and can’t prep the colon for colonoscopy.
? Embolize - NO. There is not the rich collateral circulation in the lower tract that is
present in the upper tract, so embolization likely to cause ischemia. IF bleeding
slower and more intermittent :

SCINTIGRAPHY - for bleeding of 0.1 cc/min. Accuracy depends on rate of bleeding


and the transit of blood thru the small bowel/colon. As the sensitivity of the bleeding
increases, the localization decreases. Performing surgery solely based on a positive
scintigraphy has undesired results in 40%.

SURGERY -- for 6%, no lesion can be localized. Position in the Sims position to assist
with intraop colonoscopy . Midline incision. Palpate the stomach , duodenum , small
bowel and large bowel to determine the proximal extent of the bleeding. Bleeding in
the stomach or duodenum should prompt intraop upper endoscopy.
- if massive colonoscopic bleeding, place colotomy in cecum via the appendix and
lavage colon. Administer at 0.3L/min. Place large rectal tube in to catch effluent.
Once colon is clean, advance scope into rectum and allow surgeon to guide the scope
thru the colon. If no site is found, blind resection or ostomy for trans-stomal
endoscopy. Blind resection should be based on the localization of bleeding to the
right or left side . This has better results and less morbidity than subtotal colectomy.
Subtotal colectomy is done as last option.

Scenario #37 - Acute Diverticulitis with Phlegmon/Abscess


60 y.o. female presents to the emergency room with the complaints of LLQ tenderness
and fever.

Issues:
a.) What is the differential diagnosis and the initial workup?

79
b.) Assuming the CT scan shows thickening of sthe sigmoid with phlegmon, what is you
initial treatment. If there is no air fluid level or fluid collections; what if there is a
fluid collection? What would you do if a CT scanner was not available?
c.) Assuming the patient is not better in 24 hrs., what do you do? What about the
possibility of Crohn’s disease in this patient?
d.) Assuming that the patient defervesces and improves, what would be the length of
treatment and discharge advice to her? What would you tell the patient regardingthe
possibility of a colostomy?
e.) Assuming she does not get better, what would your operative treatment be -i.e. - how
would you decide how much colon to remove? Describe in detail how you would
remove the colon including identification of important anatomic structures?
f.) Upon removal of colon, there is a rush of urine - how would you evaluate this ? (ureter
and bladder)
g.) What would you do if the sigmoid colon is densely adherent to the uterus?
Are sections necessary for CA?

ANSWER:
DDX OF LEFT LOWER QUADRANT PAIN AND FEVER:
1.) Diverticulitis
2.) Crohn’s Disease of the colon.
3.) Perforated Colon Cancer.
4.) Perforated DU with LLQ abscess.
5.) Pyelonephritis.
6.) Mesenteric Infarction
7.) Ischemic Colitis
8.) Stercoral Ulcer
9.) Tuboovarian Abscess (unlikely here)
10.) Foreign body perforation
11.) Pseudomembranous colitis
12.) Lymphoma.

WORKUP:
CBC- assess for leukocytosis and/or anemia.
Bun/Creat.- evaluate renal status and guide selection and dosing of antibiotics.
LFT’s - evaluate transaminases , alk phos and bili. for survey for liver complications of
diverticulitis.
UA - look fr UTI or evidence of fecaluria.

Radiologic Tests:
Upright chest film - look for free air from perforation.
Abdo flat and upright - assess mechanical obstruction or ileus.

CT Scan - question is whether to pursue a CT scan right away or give a presumptive


diagnosis of diverticulitis and start medical therapy and get CT scan when it appears

80
that medical therapy is not succeeding (2-3 days). At least as sensitive as contrast
enema for diagnosing diverticulitis and abscess and better at diagnosing pathology
that isn’t diverticulitis and can be used for placement of percutaneous catheter for
abscess drainage.
How? - enteric contrast is given by mouth or NGT. Whenever possible, 150-200 cc of
contrast is given by enema. If colovesical fistula is suspected, do not give IV contrast
to see if GI contrast fill the bladder.

TREATMENT OPTIONS FOR DIVERITCULITIS:


UNCOMPLICATED (does not need an operation)
Outpatient - mild pain, minimal tenderness, low-grade fever, insignificant leukocytosis.
Clear liquid diet and oral antibiotics
Inpatient - comorbid disease, severe manifestations of inflammation, altered immune
status. Hospital ization , bowel rest, IV antibiotics, IV fluids. NGT if emesis or
evidence of obstruction. Hold oral intake until signs of inflammation has resolved.
Lack of improvement of diverticulitis in 72 hrs. means this is likely complicated
diverticultis and should be investigated with a CT scan.

ABSCESS - CT guided percutaneous drainage - successful in 75% of patients. Failure is


associated with feculent drainage. Treat concurrently with antibiotics aimed at
aerobes and B. Fragilis. Obtain sinogram at 4-7 day intervals for development of
fistula to colon and monitor collapse of abscess. Leave drain in place. After
leukocytosis and pain resolve,patient can be started on low residue diet and
discharged with drain in place. Leave drain in place until surgery which can be
performed 10-14 days after discharge. If septic picture re-develops, CT should be
redone to see if there are other collections to drain.

NO ABSCESS - no immediate diagnostic tests are necessary. Wait 4-6 weeks for
resolution of inflammation. Should get sigmoidoscopy to rule out CA or Crohn’s
disease and BaE to see diveritculi. Hospitalization rarely requires more than 7-
10days.
?Operation - those who have been treated as above (successful medical therapy) , 1/3
will have a second attack and 25% of those will need hospitalization, usually within
the first year. Mortality climbs with each successive episode of diverticulitis.
WARN ABOUT THE POSSIBILITY OF A COLOSTOMY (ENCOUNTERING
UNDRAINED ABSCESS, DILATED BOWEL PROXIMAL TO SIGMOID,
ETC)

COMPLICATED DIVERTICULITIS (REQUIRING OPERATION).


INDICATIONS:
ABSOLUTE:
*Perforation./Peritonitis/Acute Abdomen
*Obstruction
*Abscess (not drainable percutaneously)

81
Abscess - those with successful percutaneous drainage should have resection.
Fistula
Inability to exclude CA.
Stricture.
Immunocompromised patient.(should have operation with initialadmission)
*indications for emergency surgery.

RELATIVE:
Two or more attacks
Persistent mass
Persistent urinary tract symptoms

EQUIVOCAL
One attack under 40 yrs. (data for age of 50 as cutoff is controversial)

Abscess - undrained abscesses should be drained operatively. Patients who undergo


percutaneous drainage should have resection as these often lead to fistulas, strictures,
persistent attacks and other indications. 70% of percutaneous ly drained mesocolic
abscesses will not require surgery.

SURGERY FOR DIVERTICULITIS


1.) 3 STAGE - historical interest.
2.) 2 STAGE - resection with Hartmans+colostomy takedown.
- resection with anastomosis and proximal colostomy+colostomy takedown
3.) 1 STAGE - resection with anastomosis. Only in those who have recieved colon prep,
so can’t be used in those with perforation or pelvic abscess.

SITUATIONS:

1.) FECULENT OR PURULENT PERITONITIS - 2 stage procedure with resection of


involved colon and colostomy.
2.) ABSCESS THAT HAS BEEN PERCUTANEOUSLY DRAINED - single stage with
preop bowel prep
3.) UNDRAINED PELVIC ABSCESS - usually a 2 staged procedure. If resection and
anastomosis with protective proximal colostomy are to be performed, an intraop
bowel cleansing will be necessary. No difference in the morbidity /mortality for the
two 2 stage procedures.
4.) UNDRAINED PERICOLIC/MESOCOLIC ABSCESS - can be dealt with in a 1 stage
procedure. Because of the limited inflammatory process with a mesenteric abscess, a
prep bowel prep can be performed allowing a resection and anastomosis.
5.) COLOVESICAL FISTULA - single stage procedure. Resect colon. If there is an
identifiable defect in the bladder, close in two layers. If no defect is seen, distend the
bladder with 0.5% iodine thru Foley to look for leak. If no leak, do not look further -
will need to leave Foley for 7 days anyway. Cystogram prior to d/c’ing the foley.

82
6.) COLOVAGINAL FISTULA - single stage procedure. No need to repair the vaginal
defect after the sigmoid has been resected.
7.) COLOUTERINE FISTULA - standard has been one stage procedure with resection of
the uterus, though the study group is not large enough for adequate analysis.
8.) CHRONIC STRICTURE - single stage.

TECHNICAL ASPECTS -
PREP - place in lithotomy position in case rectal irrigation or stapler may be needed.
Place Foley for bladder drainage and irrigation if necessary. Ureteral stents may help.
INCISION - MIDLINE.
MOBILIZATION - look for perforation or purulence. Note structures that the sigmoid is
stuck to. GET FROZENS TO RULE OUT CANCER. If the sigmoid is stuck to
the bladder, it should be free inferiorly. Gently peel the colon off the posterior
bladder, the trigone is unlikely involved. If adhesions are dense, probably a fistula.
If defect is seen , close in 2 layers.
- if hysterectomy has been preformed previously, colon may be stuck to vaginal cuff.
This is unlikely a fistula. Easier to get at once upper rectum has been taken down.
MOBILIZATION OF COLON - start with the descending colon and take down the
lateral peritoneal attachments up the the level of the inferior pole of the left kidney.
Dissect the colon from retroperitoneal attachments to the level of the gonadal vessels.
This will allow identification of the ureter, which can be traced inferiorly. Distal
descending colon is divided at the level where there is no inflammatory reaction. Do
not worry about diverticuli in proximal specimen, diverticuli in descending(or
transverse colon) do not cause disease.
- continue dissection inferiorly with both ureters identified . Dissection often
becomes tougher near the iliac bifurcation. (at this time if the colon is stuck to the
vaginal cuff, it can be mobilized )
- distal bowel is divided and the specimen removed with ureter in sight. Inspect the
ureter for leak or devascularization.
ON TABLE PREP - clamp terminal ileum with bowel clamp. Insert catheter via
appendicostomy and irrigate with saline followed by 0.5% iodine. Irrigate rectum
with sigmoidoscope.
ANASTOMOSIS- performed to upper rectum

BREAST

Scenario #38 - Inflammatory Breast CA


40 y.o. female presents with a 6 cm erythematous left breast which she has ignored for 1
year. She has palpable axillary nodes.

Issues:
a.) What is the appropriate workup of this patient?

83
b.) Assuming that the bone scan lights up in several places and the FNA shows CA, what
should you do? Mammograms necessary?
c.) What is the stage of this cancer and what are your recommendations to her?
d.) Is there a role for surgery in this patient?

ANSWER:
DIAGNOSIS - BIOPSY + FOR CA AND 3 CLINICAL FINDINGS
1.) peau d’orange/edema
2.) erythema (heat)
3.) ridging/wheals of skin
BIOPSY = cytologic aspiration, needle core, or incisional - need not demonstrate dermal
invasion of lymphatics if al l of the clinical criteria are present. Send for ER and
PR.
** > 1/3 HAVE METASTATIC DISEASE AT PRESENTATION. SYSTEMIC
DISEASE.

WORKUP:
1.) CEA, CA 15-3 - can use for serial monitoring if found elevated.
2.) CXR
3.) Bone Scan - follow up with bone X-rays if positive.
4.) CT of abdomen - rule out live mets.
5.) BILATERAL MAMMOGRAMS - rule out contralateral disease and serve as a
baseline for treatment. Many will develop contralateral disease during the course of
treatment.
6.) ER and PR - small subset of tumors may respond to hormonal manipulation.

TREATMENT:
CHEMOTHERAPY - INITIAL FORM OF TREATMENT FOR IBC, A SYSTEMIC
DISEASE.
F= 5-FU
A= Doxorubicin
C = Cyclophosphamide
V= Vincristine
P = Prednisone.

- 3 cycles of chemotherapy are given and the response to therapy is gauged, the next
step of therapy is based on the response to chemo. Those with clinical response or
partial response should have mastectomy to measure the residual disease in the breast.
Prognosis and 5 year survival correlate with the amount of residual disease in the
breast. Those who have surgery are followed up with more chemotherapy and XRT.

SURGERY - does not affect survival, as the difference in disease free survival is no
different for those who have surgery from those who don’t. However, surgery can
provide important information about the response to chemo. Those that respond to

84
chemo can have long term survival and can be spared the highest dose of XRT and its
late complications. For those that don’t respond to chemo, surgery is only a
debulking procedure.

Scenario # 39 - DCIS/LCIS
55 y.o. female referred for mammography finding and stereotactic biopsy giving
diagnosis of DCIS.

Issues:
a.) What do you do? What questions would you ask the previous surgeon?
b.) With path of DCIS, what would you recommend?
c.) If the path is LCIS, what would you recommend?
d.) how would you follow this patient?
e.) what would you tellthe person who asks about the risk of CA with fibrocystic breast
disease?
f.) what if the biopsy reveals sclerosing adenosis and a small focus of LCIS - is any
further workup needed ? Any further therapy needed?

ANSWER:
a.) Path slides for own pathologist, presence of positive margins, mammograms to
determine other sites of microcalcifications.
b.) If no contraindications, local therapy and radiation.

DCIS - DEFINITION = transformation of normal ductal epithelial cells into malignant


cells that remeain in anatomic position and have not yet broken thru the basement
membrane where lymphatic and vascular structures are. DCIS by defintion cannot
metastasize.
- it is thought that DCIS may be the precursor to invasive cancer if the malignant cells
break thru the basement membrane and are capable of metastasis. Therefore, surgery
should aim to use minimal therapy to remove cancerous cells and cure chance of
cancer while preserving the breast.

AKA = intraductal carcinoma which is confined to the ducts ( as opposed to the terminal
lobules in LCIS).

Paget’s disease of the Nipple - represents extension of intraductal carcinoma to involve


the nipple and histologically consists of permeation of the nipple epidermis by
carcinoma cells. Almost always is associated with DCIS of the immediate
subaureoloar ducts, and sometimes with coexisting invasive carcinaoma.

MICROSCOPIC PATTERNS OF DCIS:

85
COMEDO DCIS - marked by cellular atypia and central necrotic tissue, like a zit, hence
its name. Believed to be a (single) unique subtype and likely a precursor of invasive
carcinoma. BAD.

NONCOMEDO DCIS - several subtypes.(cribiform, micropapillary, solid, papillary)


displaying various amounts of atypia and microscopic patterns.

GRADING OF DCIS:
HIGH - cytologic atypia and necrosis(zit) - COMEDO DCIS
INTERMEDIATE - show either central necrosis or atypia but not both (as in HIGH) -
CRIBIFORM, MICROPAPILLARY, SOLID.
LOW - proiferating cells without atypia or necrosis - CRIBIFORM OR
MICROPAPILLARY WITHOUT ATYPIA OR NECROSIS.

PRESENTATION - can present as mass , nipple discharge, Paget’s disease, or


asymptomatic (like invasive CA). Seen most often now asymptomatically as part of
a screening mammogram.

MAMMOGRAPHIC FINDINGS: calcifications (75%), soft tissue abnormality (10%),


both (12%).

TREATMENT OF DCIS:
CONTROVERSY - while total mastectomy is curative in 100% of cases, mastectomy for
a benign lesion is more radical than accepted local treatment of a breast malignancy.
Problem with local therapy - high incidence of multicentricity ( lesions outside index
quadrant) and patients who develop recurrence may not be detected by
mammography and may develop systemic breast CA disease and die.

Radiation - those that receive local resection without radiation have 18% recurrence rate
while those treated with resection and radiation only recurred 9% of time.
Therefore, benefit of radiation is to lower recurrence rate. Most effective way of
lowering recurrence and preserving breast.

INDICATIONS FOR LOCAL THERAPY - DCIS in patients whom the entire tumor can
be surgically removed with negative histologic margins and in whom the remaining
breast tissue can be reliably assessed clinically and radiographically.

CONTRAINDICATIONS TO LOCAL THERAPY -


1.) Extensive multifocality ( another lesions within the index quadrant) may be present
and make it impossible to excise tissue to obtain a negative histologic margin and is a
contraindication to local therapy.( CANNOT EXCISE TO A NEGATIVE MARGIN).

86
2.) Presence of extensive microcalcifications throughout the breast. These may represent
other areas of DCIS but regardless (CANNOT RELIABLY ASSESS THE BREAST
RADIOGRAPHICALLY)

FOLLOW-UP - CLOSE

LCIS DEFINITION = transformation of normal ductal epithelial cells into malignant


cells that remeain in anatomic position and have not yet broken thru the basement
membrane where lymphatic and vascular structures are.
= LCIS is confined to the terminal lobules.

PRESENTATION: always an incidental finding, unlike DCIS which can present with
mass, Paget’s or nipple discharge. INCIDENTAL FINDING ON BIOPSY.
Clustered microcalcifications of DCIS are not seen with LCIS, few mammographic
clues of LCIS. Often, a benign lesion will appear suspicious and biopsy will show a
benign lesion and incidental LCIS. 30-50% chance of bilateral LCIS if
contralateral biopsy done. 1/3 of patients with LCIS will develop invasive CA,
50% in index breast, 50% in contralteral breast.
Hence, LCIS should have 100% multicentricity ( arising outside the index quadrant) if
looked for diligently enough.
-subsequent cancer can be lobular or ductal.

TREATMENT OF LCIS - since the risk is the same for both breasts, the treatment must
be the same for both breasts.
OPTIONS: BILATERAL TOTAL MASTECTOMY OR OBSERVATION.
BILATERAL TOTAL MASTECTOMY - for those who cannot or will not follow up,
for those who have a strong family history of breast CA, those who are difficult to
follow clinically or radiographically (extensive fibrocystic disease, previous radiation
therapy).

e.) what would you tellthe person who asks about the risk of CA with fibrocystic breast
disease?
f.) what if the biopsy reveals sclerosing adenosis and a small focus of LCIS - is any
further workup needed ? Any further therapy needed?

FIBROCYSTIC DISEASE - not so much a disease as changes in breast tissue related to


hormonal cycling typified by fibrosis, cysts and adenosis. may be asymptomatic ( as
in women with tender breasts) or symptomatic ( as in women with nodular , painful
breasts). symptomatic women need to be treated ( as with Danazol).

- changes do not confer an increased risk of CA. However, breasts are more difficult to
follow clinically and mammographically be cuase of the nodularity of the breasts.
Patients need excellent monthly self BSE, biannual breast exams by MD and annual

87
mammograms. New masses need to be pursued aggressively even if mammogram is
negative.

Scenario # 40 - Nipple Discharge


35 y.o. female who is otherwise healthy who is otherwise healthy comes in with the
complaint of a 2 month history of intermittent left nipple discharge.

Issues:
a.) What is the initial differential diagnosis?
b.) What do you do if you cannot find the offending duct on the first examination?
c.) What do you do if you cannot find the offending duct on the second examination?
d.) Is a mammogram indicated?
e.) Is cytology useful as a positive predictor of the presence of cancer?
f.) Is the diagnosis of carcinoma likely in this setting and is it reasonable for this patient
to be followed if she elects this option?

Differential Diagnosis:
1.) CA - bloody or serous/bloody discharge. 20-30% of those with bloody nipple
discharge will have CA.
2.) Intraductal Papilloma - bloody or serous bloody.
3.) Fibrocystic disease/change - green, yellow or brown. Brown can look like old blood
and can be differentiated from blood as in CA or papilloma using a guaiac card.
4.) Subaureoloar infection - discharge is purulent and nipple is erythematous. Treat with
antibiotics.
5.) Galactorrhea - bilateral - may be due to elevated prolactin from a pituitary
prolactinoma or from medications such as phenothiazines, Tricyclic antidepressants,
oral contraceptives. should check prolactin levels and thyroid function tests.

WORKUP;
1.) GUAIAC DISCHARGE
2.) MAMMOGRAMS
3.) CONSTRAST DUCTOGRAM - perform only in high likelihood, i.e. those with
guaiac positive discharge. If a subaureolar defect is found, a fine lacrimal duct probe
is inserted into the offending duct .Incision made around the edge of the aureola one
half the circumference of the nipple. Nipple / aureola complex is elevated and
dissected from the surrounding fat. duct containing probe is isolated and subaureolar
tissue is taken. Same approach to discharge from fibrocystic disease that is
unrelenting.

Discharge cytology - high false positive and negative rate .

Scenario #41 - Breast Mass


52 y.o. postmenopausal female presents to your clinic with the complaint that she has
discovered a 1.5 cm. new mass in the upper outer quadrant of her left breast.

88
Issues:
a.) What is your workup of this lesions?
b.) Assuming that she comes in with a nonpalpable lesion with microcalcififcations on
mammography in the same location that is not palpable, what is your workup?
c.) What would you tell this patient regarding your recommended course of action to
establish a diagnosis and treatment? Is a contralateral biopsy needed?
d.) What would you tell this patient if an FNA showed malignant cells?
e.) What advice would you give about breast reconstruction?
f.) Will treatment (surgical and/or chemo) vary depending on whether or not the patient is
pre- or post menopausal?
g.) What would you recommend about adjuvant chemotherapy if this were a
premenopausal woman?
h.) In this post menopausal woman, what chemotherapy would you recommend?
Tamoxifen?

45 y.o. female sent in by LMD because of a stelllate lesion seen on the mammgram. No
calcifications. Her mother and sister both have had breast CA, and she has had lumps
removed from her breast before - all of which were benign.

Issues:
a.) What would you recommend of her mammogram?
b.) Biopsy comes back as invasive lobular carcinoma, what would you recommend given
her strong family history?
d.) What advice would give for this premenopausal woman if the path shows negative
nodes? positive nodes?
e.) Would you recommend Tamoxifen?

ANSWER:

WORKUP OF DOMINANT MASS:

KEY - FOR WOMAN OVER 30, START WITH BILATERAL MAMMOGRAM.


FOR WOMAN UNDER 30, SELECTIVE USE OF MAMMOGRAPHY WHEN
INDICATED.

WOMAN UNDER 30 :
1.) BREAST ULTRASOUND: used to rule out cyst/adenoma and direct mammography.
2.) US= cyst or adenoma - Observe. If changes in cyst or adenoma, BIOPSY.
3.) US= not cyst or adenoma - BIOPSY.

WOMAN OVER 30 (PREMENOPAUSAL) :

89
1.) BILATERAL MAMMOGRAM - sensitivity and specificity increases with age. Risk
of exposure to radiation decreases with increasing age. Utility of mammograms
minimal for women under 30 because of density of breast tissue.
2.) NEEDLE ASPIRATION CYTOLOGY -
3 options:
- cyst which resolves upon aspiration. Nonbloody aspiration. Can be observed - if it
does not recur, it is benign. If it recurs or enlarges it should be biopsied.
- yields diagnostic cytology benign - in order for observation, the aspirate must be
diagnostic and benign. If the aspirate does not contain appropriate cells, a formal
biopsy should be done. If this is to be obseved, it should be corroborated by other
evidence, such as a breast ultrasound in women under 30 or either a normal
mammogram or second benign aspirate in women over 30.
- nondiagnostic cytology - such as no cells. formal biopsy.

WOMAN OVER 30 (POST MENOPAUSAL)


1.) BILATERAL MAMMMOGRAM - all post menopausal woman should go
immediately to mammogram.
2.) BIOPSY - because of the high risk of CA in this age group , biopsy should be done
for any mammographic abnormality.

ASPIRATION CYTOLOGY - 10cc syringe and 21 gauge needle. No local . Insert


needle into mass. Withdraw/suction on the needle making several passes thru the
mass without withdrawing the needle from the skin. If cystic and disappears on
aspiration, return in 4-6 weeks for re-eval. If bloody, send for cytology.
- if solid, material in barrel of needle should be placed onslide and fixed.
Cytology - 10-15% FALSE NEGATIVE. 5% FALSE POSTIVE.
- if benign, should have normal mammogram to corroborate for observation . return in 6-
8 weeks.

BIOPSY -
INCISIONAL - removes a portion of the mass. Should be reserved for those with lesions
3 cm or larger /too large to allow complete excision without unacceptable cosmetic
sequelae.
EXCISIONAL - removal of the entire mass, allows optimal diagnostic.
- incsion shoould be over the mass and oriented circum aureolarly such that it could be
encompassed in mastectomy incision.

SURGICAL OPTIONS:

PREOP ASSESSMENT -
BILATERAL MAMMOGRAMS - already done. Used to detect the size of the lesion in
the ipsilateral breast and the presence of multicentric cnacers or microcalcifications.
Screen contralateral breast for malignancy.
CXR - rule out met disease to the lungs.

90
Alkaline Phosphatase - with normal alk phos and normal physical exam, chance of
boney mets or intrabdo mets is minimal.

BREAST CONSERVING THERAPY VS. MRM -

- BREAST CONSERVATION = size of breast vs size of lesion.

FACTORS FAVORING BREAST CONSERVATION -


1.) Patient prefernece.
2.) Tumor size vs. breast size favorable and in god position.
3.) Unifocal tumor
4.) Anticipate ability to follow with physical exam and mammography.

FACTORS CONTRAINDICATING BREAST CONSERVATION/IRRADIATION;


1.) Total excision of mass causes significan t breast deformity.
2.) Multiple microcalcifications - inability to exclude multicentric tumors will amke
radiographic followup difficult and should have MRM.
3.) extensive intraductal carcinoma in infiltrating ductal CA.
4.) lobular CA
5.) Patient preference.
6.) contraindication to radiation - like pregancy.
7.) Inability to achieve negative margins on excision (either invasive or in situ ).
8.) Inability to folllow the patient closely by physical exam or radiographically.

Importance of Radiotherapy to Breast conservation - lowers recurrence rates from 40% to


10%.

TECHNIQUE OF BREAST CONSERVATION - incise over mass (ellipse skin if old


incision en bloc). If previous incision and biopsy, do not enter cavity - simply
reexcise around old site. Orientation sutures are beneficial . Perform ALND.

BREAST RECONSTRUCTION:
TIMING - while reconstruction can be at the same time or delayed, ti is generallly better
delayed if consideration is being given to radiation therapy. (unusual in case of
MRM).
No problem with timing of chemotherapy when considering reconstruction. Trend now
is toward immediate reconstruction.

OPTIONS;
1.) TRAM - autogenous reconstruction of choice. Rectus abdom inus muscle and island
of skin are mobilized and rotated . Can also use latissimus dorsi flap or omental flap.

91
2.) Prosthesis - usually an implant into subpectoral region,need t o preserve pectoralis
fascia and median pectoral nerve during mastectomy. May need tissue expander
placed at the time of mastectomy to expand the skin .
Complications - inbfection, tissue loss and poor cosmetic results.

ADJUVANT CHEMOTHERAPY

RATIONALE FOR THERAPY -


- large number of prospective trials have confirmed improvement in disease free and
overall survival with adjuvant chemotherapy in premenopausal women and a lack
of benefit in those who have gone thru menopause or who are over 50.
- have also shown a major survival advantage after adjuvant chemotherapy in breast
cancer patients under 50 years with positive nodes.
CONSENSUS - TREAT PREMENOPAUSAL WOMEN WITH NODE POSITIVE
BREAST CA WITH ADJUVANT CHEMOTHERAPY AND TREAT
POSTMENOPAUSAL WOMEN WITH NODE POSITIVE BREAST CA WITH
TAMOXIFEN.
MILAN TRIAL - median survival of premenopausal women withnode positive disease
who were not treated is 8 years. Median survival of premenopausal women treated
with CMF (cyclophosphamide, methotrexate and 5-FU).

Why? - uncertain. May be that chemo causes a “chemical oophorectomy” by killing the
ovaries early in premenopausal patients giving antiestrogen result .
- may be that premenopausal women can tolerate higher doses of chemo and
subsequently get more therapy which is effective in killing tumor cells.
- may be that premenopausal malignant cells are biologically different and respond
differently to therapy.

Chemotherapy + Tamoxifen - would think a natural synergistic effect of the two but
studies are not that clear. Some have actually shown decreased survival with the
combination.

POSTMENOPAUSAL WOMEN WITH NODE POSITIVE DISEASE - not the benefit


of adjuvant chemotherapy seen in those premenopausal /under 50 years. What to
do?
- those with ER positive tumors and postive nodes should be treated with tamoxifen..

TREATMENT OF NODE NEGATIVE PATIENTS - several studies have shown a slight


but not statistically signficant improvement in overall survival in patients with node
negative disease. I t seems that some node negative patients would benefit from
postop chemo but it is not yet known which ones. Patients with larger tumors may
benefit from chemo even with negative nodes.

TUMOR SIZE PREMENOPAUSAL POSTMENOPAUSAL

92
ER (+) ER(-) ER(+) ER(-)
< 1 CM, NEG.NODE NT NT NT NT
> 1 CM, NEG.NODE TAM. CHEMO TAM CHEMO
POS. NODES CHEMO CHEMO TAM* CHEMO

* - same results as chemo, but tamoxifen less toxic


- those post menopausal , node positive woman have improved disease free survival but
not overall survival. Premenopausal, node positive women have improved disease
free and overall survival.
- appears to be increased disease free survival , but probably not overall with adjuvant
chemotherapy. Results of trials are needed.

HEAD AND NECK

Scenari o #42 - Parotid Mass


40 y.o with 2 cm mass at the angle of the mandible that has been present for at least one
month and may be slowly increasing in size.

Issue:
a.) Initial work-up and differential diagnosis of this mass.
b.) Operative approach to this lesion, including what is it most likely to be?
c.) What are the types of parotid tumors and how do they dictate your initial
management?
d.) Operative approach if the facial nerve is going thru the center of the mass? After
excision of the facial nerve, what are the possible sequelae and possible remedies to
these?
e.) What are the indications for sacrificing the facial nerve?
f.) What are the indications for an accompanying neck dissection?

ANSWER:

DIFFERENTIAL DIAGNOSIS:
1.) Suppurative parotitis
2.) Chronic parotitis
3.) Recurrent chronic sialadenitis
4.) Chronic sialolithiasis
5.) Trauma
6.) Mucocoele
7.) Cyst
8.) Fistula
9.) Tumor - benign or malignant.

DIAGNOSIS AND WORKUP:

93
CLINICAL FACTORS - need to know whether the facial nerve is paralyzed preop as it
is unlikely that it’s function will return after a parotid resection.

RADIOLOGY - should be used for mass lesions and for localization in either a deep or
superficial lobe.

BIOPSY - is not appropriate for differentiating malignant from benign (test is not that
sensitive ) All masses of the parotid with normal facial nerve function will be treated
with superficial parotidectomy. Open biopsy is discouraged for fear of injuring the
facial nerve. Since surgery will be utilized for removal of benign lesions and frozen
section can be obtained at that time, distinguishing benign from malignant
preoperatively will not save the patient an operation.

STAGING:
1.) Tumor Size
2.) Local extension of the tumor.
3.) Spread to regional nodes.
4.) Distant Metastasis.

TYPES OF BENIGN PAROTID TUMORS

PLEOMORPHIC ADENOMAS - 90% of all benign lesions, are slow growing and may
have a thin capsule. Unless they have undergone malignant degeneration, these are
benign encpasulated lesions. do not invade facial nerve. 100% curable if the lesion
is removed with entire capsule, but if small pseudopods are left behind it will recur.
Re-resection is difficult because of multifocality of recurrence and high likelihood of
injuring facial nerve.

ADENOLYMPHOMA (WARTHIN’S TUMOR) - SECOND MOST COMMON - 10%


bilateral. Superficial , located in the tail of the gland. do not become malignant, do
not recur. Do not enucleate and require partial parotidectomy.

ONCOCYTOMA - rare, tend to be bilateral. do not recur.

MONOMORPHIC ADENOMA - thought to be a precursor lesion to either pleiomorphic


adenoma or adenoid cystic carcinoma. Importance is avoiding confusion of this
lesion with malignant adenoid cystic carcinoma.

Treatment of Benign Lesions - Superficial Parotidectomy - implies sparing of the


facial nerve. The exception to this is a tumor that lies in the deep lobe. Re-resection
for recurrence of pleiomorphic adenoma is also done superficiall y to spare the facial
nerve.

94
- serves as a diagnostic procedure or a “grand biopsy” to remove the entire superficial
lobe intact without capsular disruption. Frozen section is obtained at this point to
determine whether the lesion is malignant or benign. Shoul d have frozen section
diagnosis before sacrificing any vital structures.

TYPES OF MALIGNANT PAROTID TUMORS:

MUCOEPIDERMOID - MOST COMMON PAROTID MALIGNANCY. Can be low


or high grade. Low grade have excellent prognosis. High grade tend to recur
and metastasize.

ADENOID CYSTIC CARCINOMA - A.K.A. “CYLINDROMA” - marked by


perineural invasion and mets to lymph nodes and lungs. Tend to be slow growing
and tend to recur, so usually a long course that results in death. Good short term
prognosis is deceptive, because long term prognosis is really shitty - dying.

MALIGNANT MIXED TUMOR - tend to occur in the presence of pleiomorphic


adenoma, leading some to believe it is malignant degeneration of pleiomorphic
adenoma. Tend to present as a sudden enlargement or facial nerve paralysis/pain in
a longstanding parotid tumor. worse prognosis than the other “low grade”
malignancies (adenoid cystic and mucoepidermoid).

ACINOUS CELL CARCINOMA - another “ low grade” malignancy that tends to grow
slowly and recur. Lymph node mets about 15%.

ADENOCARCINOMA - these are all glandular tumors that do not fit into the above
categories. Fall into 3 grades which range from low grade to high grade with
prognosis that range from good to shitty.

SQUAMOUS CELL CA - sometimes difficult to distinguish primary involving skin


from a skin primary involving/metastatic to the parotid. Aggressive tumor with
shitty prognosis.

Treatment of Malignant Parotid Tumors - complete conservative parotidectomy -


implies preservation of main trunk of facial nerve and its branches.

SPECIFIC SITUATIONS IN PAR0TID RESECTION -


1.) Tumor adherent to nerve - can be dissected off and treated with tumoricidal doses of
radiation (5000 -6000 Gy).
2.) Low grade malignancy involving portion of facial nerve - should be treated with
subtotal parotidectomy with preservation of uninvolved branches of the facial nerve.
3.) Massive tumors involving facial nerve with facial nerve paralysis, high grade tumors
and malignant recurrences should be treated with sacrifice of the facial nerve. Nerve
grafting is attempted if the facial nerve worked preop.

95
4.) Malignancy involving facial nerve - must take along with involved structures.
Should attempt grafting with sural or greater auricular nerve graft.

Indications for acompanying MRND -


1.) High grade lesion that tend to metastasize.
2.) Clin ically suspicious nodes.

Scenario # 43 - Oral Mucosal Cancer


70 y.o. male, pipe smoker, presents with a 0.5 cm ulcer on his lower lip, slightly to the
right side and midline. First noted approximately 8 months ago and has never
entirely healed. It is moderately painful with some induration at the margins. There
are palpable, mildly tender nodes on the upper neck on the right submandibular
triangle.

Issue:
a.) Differential diagnosis and initial treatment ( initial treatment should include the use of
oral antibiotics for 2 weeks to see if the lympn nodes decrease in size and tenderness.)
Is incisional or excsional biopsy appropriate?
b.) Operative approach for this lesion, including initial closure or rotational flap closure
should the defect be too large. What is the appropriate margin?
c.) Is MOHS surgery appropriate for this type lesion
d.) is radical neck dissection indicated if the nodes do not disappear after treatment with
oral antibiotics?
e. ) What is the stage of this lesion?
f.) What are 5 yr. sruvivals with and without positive nodes?
g.) How is treatment and survival changed by a tumor located in
- anterior 2/3 tongue
- posterior 1/3 tongue
- floor of mouth

ANSWER:

PATHOLOGY - head and neck cancers are either sqaumous cell CA of the mucosal
lining of the upper aerodigestive tract. Remainder are salivary gland tumors (see
question # 42).

Premalignant lesions - several types (leukoplakia, acanthosis, hyperkeratosis,


parakeratosis) are all benign in contrast with premalignant leukoplakia which shows

96
atypical cells and is dificult to differentiate from carcinoma in situ. Rarely have
malignancy and very small progress to cancer.
Erythroplasia - much higher potential for malignant generation (30%).

Level of Lymph Node Metastasis Primary Site


I - above the digastric(submandibular) Lip, oral cavity, skin
II - superior SCM (post to gr. auricular n) Oral cavity,
oropharynx, nasophar
hypopharynx, larynx
III - mid SCM (ant to fr. auricular n) Oral cavity, orophar.
hypopharynx, larynx,
thyroid
IV - inf SCM ( below omohyoid) oropharynx, hypophar.
larynx, cervical esoph.
thyroid
V - posterior triangle nasopharynx, scalp,
breast, lung, GI tract,
scalp.

- importance is not to memorize but to know that there is an orderly progression of tumor
cells thru lymphatic drainage. Involvement at multiple layers, it is usually represents a
progression from superior to inferior echelon nodes and is considered BAD. Presence of
confirmed lymph node met cuts survival rate in half compared to disease localized
to primary site. Those with mets at multiple levels in the lymph node chain will not
be cured regardless of therapy.

PREOP EVALUATION:
CT SCAN - useful for deciding the amount of bony involvement of the mandible or
maxilla in oral cavity tumors.
- assessment of neck lymph node mets is also enhanced by CT scan.

PRINCIPLES OF TREATMENT
- for small tumors such as T1 ( 2 cm or less in largest diameter) or T2 (tumor larger
than 2 or less than or equal to 4 cm) surgery and radiotherapy give equivalent
results. Factors in selecting one over the other depend on morbidity of treatment ,
complexity of treatment and other shit, you get the picture.
Early laryngeal tumors - best treated by radiotherapy first, reserving surgery for salvage
in the case of failure.
Early oral lesions - those that can be easily excised perorally in a single setting with
primary reconstruction are best treated by single modalitysurgical therapy and avoiding
prolonged radiation treatments and their radiation related complications.

97
- for larger tumors such as T3 (larger than 4 cm ) or T4(invading adjacent
structures) multimodality therapy - surgery + postoperative radiotherapy is the
standard of care. Selective regional neck dissection is used in cases where early lesions
have a question of lymph node mets.
Radiation - external beam - used for bulky primary tumors and extensive nodal
metastasis.
Other situations in which radiation is used is in cases where there are positive margins
and where ther has been extensive soft tissue involvement by the primary or mets.

PRIMARY TUMOR + CLINICALLY POSITIVE NODES - open biopsy of node is of


little value in diagnosis and detrimental to treatment. If a primary tumor is identified and
proven by biopsy , any clinically suspicious nodes have traditionally been treated as
metastasis for the purposes of staging and treatment planning. Can use FNA if it will
change treatment ( as in a questionable node in the presence of a small =T1 or T2 tumor
which would add radiotherapy to the treatment plan of an oral lesion.)

STAGING 5 YR. SURVIVAL


I T1NOM0 70-90%
II T2NOMO 60-80%
III T1 N1MO 30-50%
T2 N1MO “ “
T3 N1MO “ “
T3 NOMO “ “
IV T4 NO-1MO 25% OR LESS
T1-3N2-3 MO
T1-3 N0-3M1

SURGICAL TREATMENT - “BLOCK” resection of tumor with grossly and


microscopically clear margins in continuity with lymph node shed.

MRND- indicated for large tumor (T2 or greater) or in association with clinically
positive nodes.

LIP - early lesions are easily treated with surgery or radiotherapy, ease of surgery
makes a wedge excision of the lip lesion with reconstruction in a single setting.
- lip shave procedure - good for areas adjacent to tumor with severe actinic changes and
dysplasia. MOHS surgery is an effective alternative.
- palpable nodal mets indicate MRND.

Reconstruction - most lip cancers are 2 cm or smaller at diagnosis. For these patients,
resection with negative margins will be less than 1/3 of the lip.
Shield excision - shield shaped incision is made around the tumor and vermillion
border marked with scalpel.Full thickness lip is incised and closed in three layers.

98
W- shaped incision- W shaped incision is made around tumor withpoint of W
near tumor. If more tissue needs advancing the incision can be extended laterally along
labiomental folds.

- Excision of 1/3-2/3 of lip - can use stair step excision which is a form of a
complex Z-plasty.

TONGUE -
ANTERIOR 2/3 - chronic non-healing ulcer. At presentation, 1/3 will have cerivical
lymph node mets at presentation. During the course of disease, 2/3 will develop cervical
nodal mets. T1 cancers have 5 yr. survival of 80%, nodal mets drop this in half.
- early lesions should be excised thru oral approach with transverse excision.
Margin should be 1.5 cm for small tumors.
- MRND for T2 or greater lesions, also for locally invasive tumors.

POSTERIOR 1/3 OF TONGUE - 2/3 will present with lymph node mets.

FLOOR OR MOUTH - small, early tumors can be treated by surgery or radiation, but
radiation can be complicated by osteonecrosis of the mandible. Enlarged salivary
glands may represent occlusion of the Wharton glands and may be benign. Smaller
lesions should be resected transorally. Larger lesions approximating mandible must be
treated with partial mandibulectomy. Extension into the mandible requires a
“commando “ type of procedure where they rip off half of your face. As always, stage I
and II lesions have a 5 year survival of 70-90% and nodal disease (stage III) reduces this
by one half.

Scenario #44 - Solitary Neck Mass


30 y.o. female with solitary neck mass/lymph node.

Issue:
a.) What is the appropriate workup of this neck lesion? What test would you obtain to
image areas outside the head and neck?
b.) what is the best initial means of obtaining tissue?
c.) What operation would you perform if this came back SCCA? AdenoCA? MCT?
Hodgkin’s Disease?
d.) What are sites of origin of SCCA above the clavicle? Appropriate workup?
“ “ “ “ ADENOCA “ “ Appropriate work- up?

99
ANSWER:

-excluding benign primary thyroid tumors, 80% of neck masses in adults are
maignant, of which 80 % will be metastatic. 80% of metastatic neck masses arise
from above the clavicle.

INITIAL EVALUATION AND EXAMINATION:

EXAM - if the mass moves on swallowing, it is likely unde the strap muscles and is
likely a thyroid primary.
- use otoscope, tongue depressor, nasal speculum, glove and mirrors to examine the
oral cavity (glove and tongue depressor and otoscope), oropharynx , hypopharynx ,
larynx and nasopharynx (nasal spectulum and mirrors for indirect laryngoscopy).
- also examine groin and axilla (adenopathy associated with Hodgkin’s disease),
breasts (breast CA) , abdomen and rectal exam (GI tract CA), respiratory tract (lung
CA).

BIOPSY - FNA
Positive Results:
Metastatic thyroid - definitive surgery.
Lymphoma - remove entire node to confirm and establish cell type.
Branchial Cleft cyst - definitive surgery.

SCCa with unknown primary - search for primary before providing definitive
treatment.
- CT scan
- CXR - squamous cell lung CA can go to neck.
- Upper GI - squamous cell of esophagus.
- Head and Neck Exam under anesthesia.

AdenoCA with unknown primary - search for primary before providing definitive
treatment.
- CT scan of chest and abdomen - rule out adenoCA from lung or
GI primary. Also rules out renal cell/upper Gusystem tumor as primary.
- UGI/Upper endoscopy + BaE/Colonoscopy - rule out adenoCA
of GI primary.
- Mammogram - rule out breast CA primary.
- Cystoscopy - rule out lower GU primary

Indeterminate Results - proceed to open biopsy.


What kind ? - parotid - None. Leads to too high recurrence rate and could injure
facial nerve. Do superficial
parotidectomy.

100
- small - excsional biopsy.
- large - incisonal biopsy.

Head and Neck Exam under anesthesia - indicated for diagnosed SCCA by FNA
without known primary site by initial examination. Examine the oral cavity,
nasopharynx, tonsil, base of tongue, hypopharynx and larynx. Biopsy any
suspicious lesions. These should be done as frozens. If nothing is seen, proceed to
Triple endoscopy.
Triple endoscopy = esophagoscopy, laryngoscopy, bronchoscopy .
Biopsies - nasopharynx, base of tongue, tonsil, pyriform sinus. Do as frozen to allow
repeat biopsy for questionable lesions.

TREATMENT :
1.) NECK - if it can be surgically removed, it should be removed along with a
MRND.Need to determine th e presence of tumor , the number of affected lymph
nodes and the presence of extracapsular spread. If multiple nodes are involved or
there is extracapsular spread, postop radiotherapy is necessary.

2.) PRIMARY -
Nasopharynx, tonsil, base of tongue, hypopharynx and larynx - treated with
radiotherapy.
Oral Mucosal - lip, anterior 2/3 of tongue, floor of mouth - surgical excision.

Metastatic SCCa without known primary = 3% - results are the same after MRND if
you irradiate the head and neck or whether you wait for the primary to show up.
When it does show up, you can always treat with surgery or radiation. If you decide
to observe, check every month for first year, every 2 months for next year and every
three months for third year.

TRAUMA

#45 - Polytrauma with Pelvic Trauma


25 y.o. female who fell three stories to the pavement and brought to ER.
Initial vitals BP=100/60. RR = 22.

Issues:
a.) How would you manage this patient? ABC’S and initial evaluation (primary survey)
and intervention.
b.) Secondary survey shows blood at the meatus and pelvic film showing inf/sup ramus
disruption and widened SI joint? What intervention is necessary befor e placing a
foley? What if an extraperitoneal bladder injury is found?

101
c.) How would you treat this pelvic fracture if the patient becomes unstable? How would
you rule out other injury? how is this performed? If the other injury is ruled out, and
the patient becomes more unstable ( from pelvic injury) what is appropriate
intervention? Discuss which types of pelvic fractures do and do not benefit from an
external fixator?
d.) CXR shows widening of the mediastinum. What does this suggest? What is the
mechanism of this injury and how is it surgically repaired?
e.) CXR ahows haziness on right side. CT is placed and puts out 500cc?
What is the initial output and hourly output that would prompt you to operate on this
patient?

ANSWER:

PELVIC FRACTURES:

-many patients with severe pelvic trauma have multiple injuries , such that the
source of hemorrhage may bw the pelvic fracture itself, or some combinationof
associated intraperitoneal, thoracic or long bone injuries.Only after the major
sources of hemorrhage have been found and treated and the patient is stabilized
should the associated structures (rectum, blaader and nervous system ) be evaluated.

CLASSIFICATION - multiple classification systems, revolve around the stability of the


fracture which is dictated by the number and position of the fracture sites.

UNSTABLE - generally involves more than one fracture site and disruption of the pelvic
ring.
“Open Book” - pelvic ring disruption - with AP force, the pelvis separates
anteriorly and hinges open on the posterior sacroiliac joints. Pubic diasthasis.
“Vertical Shear” - pelvic ring disruption in front and back with force from below
resulting in dispalacement of one hemipelvis superior to another.
“ Lateral Force” - force comes from lateral side disrupting the pelvic ring in front and
in back.

Crush Injury - involves several breaks in the pelvis


** direct correlation between the number of fractures and the number of blood
transfusions necessary.

STABLE - involve single breaks in ring or iliac crest or symphysis.

MANAGEMENT - hemorrhage is the leading cause of keath insevere pelvic fractures


and survival depends on rapid identification and control. 2 situations in which there
is a delay in treating bleeding (with increased morbidity and mortality) is a delay in
diagnosis and putting the patient thru a negaitve celiotomy.

102
UNSTABLE PATIENT - ongoing bleeding from pelvic hemorrhage - need to control
with external fixator or MAST trousers.
Approach #1- place MAST (legs to 50 mmHg and abdo to 40 mmHg) trousers in ER
and then do DPL. If positive, celiotomy. If negative , arteriography and
embolization.
Approach #2 - take to OR for DPL. If positive, celiotomy. If negative, place external
fixator in OR. If bleeding continues, go for arteriogram.

DPL - supraumbilical tap yields a false positive rate of 10-18%.

CELIOTOMY - midline incision is used. Pelvic hematomas are not to be opened


unless rapidly expanding or pulsatile. That is , never.
Bilateral internal iliac artery ligation is not successful be cause of the rich collateral
network.

HEMODYNAMIC INSTABILITY AFTER CELIOTOMY - patient should be taken


immediately to arteriography for emobilization. Embolization will stop arterial
bleeding but not venous ooze, so if arteriogram shows no arterial bleeding , an
external fixator (or MAST trousers) should be placed (if not already placed) to
decrease the venous oozing. If arteriogram shows leakage , embolize. Then to ICU
for observation.

UROLOGIC INJURIES - URETHRAL INJURY SHOULD BE SUSPECTED IN ANY


MALE WITH A PELVIC FRACTURE. Anyone with blood at meatus, high prostate,
scrotal hematoma should have retrograde urethrogram prior to placing Foley.
Treatment of Urethral Injuries - place suprapubic tube . Most partial tears will heal
spontaneously.

BLADDER INJURY - If urethrogram is negative, place Foley catheter. Indications for


cystogram are gross or microscopic hematuria with pelvic fracture.

TREATMENT -
EXTRAPERITONEAL - treat with drainage (foley or suprapubic).
INTRAPERITONEAL - operative drainage of bladder with 3 layer closure.

BLUNT THORACIC VASCULAR INJURY -


falls from great heights with deceleration when hitting the ground, deceleration injuries
from MVA and direct blows to the precordium can cause blunt injury to the aorta or
the innominate artery.
= 15% will survive the initial insult. Of those that make it to the ER, 50% will rupture in
the first 24-48 hrs.
= strong history of mechanism and widening of the superior mediastinum more than 8
cm, loss of aortic knob,multiple left rib fractures, pleural cap or left pleural effusion

103
indicates retrograde femoral arteriography to detect site of aortic tear. Site of injury
at the ligamentum arteriosum distal to the origin of the left subclavian artery.

REPAIR - left posterolateral thoracotomy in fourth interspace after placing double


lumen tube. Isolate the aortic arch, descending aorta and lsft subcalvian artery with
umbilical tapes.. Clamp above and below and sew in a Dacron interposition graft.
Rate of paraplegia is the same regardless of type of repair used.

#46 - Blunt Trauma with Hypotension


18 y.o. male S/P MCA comes into E.R. with normal GCS and BP =60. C-spine, pelvis,
CXR are clear. DPL is grossly positive. Urine is pink.

Issues:
a.) ABC’s.
b.) How would evaluate the urine/kidneys?
c.) How would you deal with patient with hypotension/+ DPL.
d.) Intraoperatively, you encounter a large midline retroperitoneal hematoma. How
will you get apporpriate control?
e.) Once you have appropriate vascular control, how would you expose and explore this
hematoma?
f.) You find that the pancreas is transected over the spine. How do you deal with this?
How do you deal with the spleen?
g.) You find the left renal avulsed at the origin, what do you do? What if there is a blow-
out and reimplantation cannot be performed?

ANSWER;
a.)
b.) One shot IVP
c.) SRA or IRA clamp. Iliac clampp distally.
d.) SRA/ Right - Kocher and right colon mobilization.
e.) SRA /Left - mobilize the spleen and left colon to midline.
IRA - clamp below renals , clamp at iliacs.Open midline.
f.) see below.
g.) Reimplantation /repair difficult in blunt trauma -> opt for nephrectomy.

RETROPERITONEAL INJURY:

Hemodynamically unstable patient with pelvic fracture - to OR for supraumbilical DPL.


If positive, celiotomy. If negative, external fixator or arteriogram.
Hemodynamically unstabel patient without pelvic fracture - to OR for celiotomy.
Hemodynamically stable with or without pelvic fracture - CT scan.

104
ZONE I INJURY - superior from aortoesophageal hiatus to sacral promontory (aortic
bifurcation). Lateral to the proximal renal vessels. Contains the aorta, cava,
duodenum, pancreas, proximal renal vessels. ALL MUST BE EXPLORED.

EXPOSURE -
AORTA CLAMPING SUPRARENAL - in lesser sac , divide the right crural
fibers over the aorta. Clamp
here.
- alternatively, thoracotomy may be performed in OR.
INFRARENAL - below renals as in AAA.

- DON’T FORGET TO GET DISTAL CONTROL AT THE ILIACS BEFORE


MOBILIZATION.

-EXPOSURE OF THE SUPRARENAL AORTA =

SMA, IMA, LEFT ILIAC AND LEFT RENAL ORIGIN - reflect the spleen and left
colon toward the midline.
CAVA, RIGHT RENAL HILUM, RIGHT ILIAC ARTERY - perform a Kocher
maneuver combined with mobilization of the right colon.

Arterial injuries - suprarenal aorta, celiac axis, SMA and renal artery.
Venous Injuries - renal vein, peripancreatic veins.

-EXPOSURE OF THE INFRARENAL AORTA - can expose the infrarenal aorta as


one would for an infrarenal AAA in midline

Arterial injuries - infrarenal aorta or IMA .


Venous injuries - IVC

--EXPOSURE OF THE IVC - whole infrarenal IVC can be exposed by mobilizing the
right colon and performing a Kocher maneuver. Cava can be controlled by clamping
with a Satinsky and compressing with sponge sticks. Posterior holes may need to be
closed thru the anterior defect (transluminal repair).
Ligation of IVC - only for desparate circumstances. Ligation of renal veins is fatal.

ZONE II - from diaphragm to iliac crests lateral to psoas muscle. Implies injury to
kidney or colon.
One shot IVP = should be used when patient is taken directly to OR from ER because of
hypotension or + DPL. Shows whether there are two functioning kidneys (rules out
renovascular injury) , but does not show injury to collecting system.
Indications for exploration Intraoperative : (1) Expanding or pulsatile perinephric
hematoma. (2) Suspected renal vascular injury.

105
Control Intraop - proximal control of renal artery and vein prior to opening hematoma by
mobilizing the right or left colon. This may be difficult in patient who is
hemodynamically unstable, in this case, use digital control of the hilum and rapid
exposure of the kidney.
DECISION - whether to perform nephrectomy or not. Must have idea of contralateral
function via IVP. Assuming contralateral kidney functions, nephrectomy if:
- operated on after 6-8 hrs.
- unstable patient
- associated life threatening injuries.
- severely injured kidney
?Revascularization of Renal vascular Injury - because of occlusion or intimal dysruption.
Blunt Trauma - no. Rarely successful.
Penetrating Trauma - possibly.

ZONE III - bladder anteriorly, sacral promontory posteriorly and iliac crests laterally.
Do not explore unless it has ruptured into the peritoneal cavity.

#47 - Trauma with Liver Laceration.


25 y.o. trauma patient is brought to the emergency roon after an MVA. Examination is
negative except for a tender abdomen with guarding. Heart rate 110, BP=80/palp

Issues:
a.) What do you do- CT vs DPL vs immediately to OR and what are the advantages of
each?
b.) Describe preoperative need for lines, positioning, prepping and draping.
c.) Describe the initial exploratory lap in a trauma patient.
d.) The only injury you find is a larghe stellate laceration high on the posterior dome of
the liver, which bleeds massively when the packing is removed, what do you do?
Should the liver be mobilized? How? How should you obtain vascular control of the
liver’s blood supply? How do you differentiate injury from the arterial supply from
the venous supply? How can you directly expose the hepatic veins? If controlling the
arterial supply controls bleeding, what do you do next? What should be done if
suture ligation does not succeed? How would you perform a hepatic resection from
this liver fracture? Would you perform an omentopexy? What should you do if the
control/occlusion of the arterial supply does not controlthe bleeding?

ANSWER -
Airway = for those comatose or obtunded.
Apneic = intubate orally.
Apneic with facial trauma = emergency cricothyroidotomy.
Breathing but need airway control = nasotracheal intubation.
Breathing but combative = rapid sequence induction with cri coid pressure and in-line
traction.

106
Breathing = pulmonary ventilation is necessary. Confirm by physical exam - no
ventilation can occur because of poor EET placement, hemothroax, pneumothorax,
aspiration of teeth and shit, pulmonary contusion.
Circulation= a.k.a. shock - 14 gauge IV X 2
- draw blood for HCT and T&C ( and other labs)
- Adults = 2L of LR.
- Peds = 20cc/kg
Response - thoswe that improve and stabilize are unlikely to have ongoing bleeding
and can be approached systematically. Those that do not improve or improve
transiently and then deteriorate arelikely to have ongoing hemorrhage and should be
taken to OR.

Secondary Survey - detailed physical exam . Rectal exam , Foley, NGT. CXR, pelvic
X-ray.

Laparotomy - for those who remain hemodynamically unstable after 2L of fluid or those
that improve and then deteriorate.

DPL - 95% sensitivity. Positive if : 10ml of gross blood, or after instillationof 1L of fluid
yields an effluent of 100,000 rbc’s. Also postive if effluent seen in NGT, Foley, or
chest tubes or enteric material or stool present. Advantage is 95% specificity for
major organ injury and speed. disadvantages are the lack of ability to diagnose
retroperitoneal injuries.

CT scan - advantages are nonoperative/nonintervention means of diagnosing solid organ


injury (liver, spleen and kidneys) as well as diagnosing retroperitoneal injuries.
Disadvantages are need to prep pt. and time necessary for study, as well as poor
identification of small bowel injuries.
- indicated for the hemodynamically stable patient in whom nonoperative therapy is an
option. also indicated for those in which other CT scan will be necesssary (head
trauma). Needs supplementation with contrast studies of the upper and lower GI
tract.

NONPERATIVE THERAPY - better in children than adults - for those who are
hemodynamically stable blunttrauma patients. Generally seen in patients with large
hematomas and lacerations who are hemodynamically stable. These patients are
placed on bedrest and in ICU. 50% will need transfusions but should not exceed 2
units in 24 hrs.

OPERATIVE THERAPY -
POSITION - SUPINE
PREP AND DRAPE - NECK TO THIGHS

INCISION - LONG MIDLINE (xiphoid to pubis, if no pelvic fracture.)

107
EVACUATE - liquid blood and clots.
PACK - place packs into each quadrant to assess the site of hemorrhage.
EVALUATE - each quadrant, pay particularly close attention to the spleen and mesentery
as sources of ongoing bleeding as a coagulopathy develops. if the liver is injured, the
right costal margin is retracted superiorly and the right and left lobes are palpated for
defects.

MOBILIZATION OF THE LIVER - retract costal margins superiorly with self retaining
retractor. Mobilize the right lobe by dividing the right coronary and triangular
ligaments medially taking care not to injure the lateral wall of the right hepatic vein.
Retrohepatic cava and right adrenal gland lie here.
-- mobilize the left lobe ofthe liver by dividing the triangular ligament toward the
cava taking care not to injure the left hepatic vein or the retrohepatic cava.
- if exposure to the retrohepatic veins is needed, the incision should be directed superiorly
as a median sternotomy. Divide the pericardium and diaphragm radially.

METHODS OF TEMPORARY CONTROL OF LIVER HEMORRHAGE.


MANUAL COMPRESSION AND PERIHEPATIC PACKING -
RIGHT LOBE - difficult to control. Take lap pads, folded, 2-3 at a time and pack
around injury site. Place pads between the liver and diaphragm and the liver and
anteror costal margin. Have assistant exert pressure down on chest wall.
LEFT LOBE - more difficlut to pack because of anatomy. However, this can usually be
controlled by compressing the left lobe by hand.
POTENTIAL COMPLICATIONS - decreased cardiac output from caval compression
and increased airway pressure from upward pressure of packs on diaphragm.

PRINGLE MANEUVER - best to place the clamp frm the left side by opening the
lesseer omentum while guiding the posterior blade thru the foramen of Winslow
with the lft index finger. This maneuver can differentiate hemorrhage from t he
hepatic artery or portal vein from the hepqatic vein or caval injury. Can be placed
for 1 hr.Should be placed along with packs for life threatening hemorrhage on
entering abdomen.

HEPATIC VASCULAR ISOLATION - if hemorrhage is still brisk after placing the


Pringle clamp, injury is likely to the hepatic veins or retrohepatic cava.
1.) HEPATIC ISOLATION WITH CLAMPING - used frequently in elective
situations, but has limited role in trauma because of patient’s unstable condition and
lack of tolerance for clamping of cava and decreased left ventricular filling pressure.
Performed by clamping aorta, suprarenal and suprahepatic cava.
2.) ATRIOCAVAL SHUNT - median sternotomy, take down pericardium and
diaphragm. Place umbilical tapes around cava just above renal and around cava as it
enters the right atrium just abve the diaphragm. 36 Fr. chest tube wityh addtional
hole cut about 17 cm from end (intended to be in the mid-atrim). Pursestring suture
placed in the right atrial appendage. Guide CT down thru atrium and thru cava with

108
care taken not to further damage cava. Pursestring sutures are tied and portion of tube
outside atrium is clamped. Don’t try this at home.
3.) MOORE - PILCHER TUBE - balloon placed thru iliac vein and guided up to defect
and inflated to tamponade bleeding. Same results as atriocaval shunting (piss-poor)
4.) VENOVENO BYPASS - cut down on axillary artery and femoral vein. Avoids
having to open chest.

DEFINITIVE MEASURES -
TOPICAL MEASURES - for minor lacerations and grade I or II injury , manual
compression along with topical agents can be used. Can cauterize raw surfaces.
Fibrin glue is an option but if you use in front of those old fucks they’d probably
laugh to death.

SUTURING PARENCHYMA - should be used for lacerations that are 3 cm or less in


depth or in situations in which instability does not allow hepatotomy or individual
ligation of bleeders. Use 0 or 2-0 chromic on a large blunt tipped needle. Use
mattress sutures for deeper lacerations. Tighten knot until hemorrhage ceases and
liver blanches . This may work for oozing surfaces and hepatic veins but is unlikely
to work for larger hepatic arteries.

HEPATOTOMY WITH SELECTIVE LIGATION - SHOULD BE USED IF


PARENCHYMAL SUTURE DOES NOT WORK. SHOULD BE USED FOR
PENETRATING TRAUMA. Use finger fractureas hepatotomy down to the level of
the bleeding vessel. Major vascular structures (right, left or middle hepatic veins)
should be repaired instead of ligated.

HEPATIC ARTERY LIGATION - used in case of deep hepatic arterial hemorrhage in


which Pringle maneuver works to control hemorrhage. if selective clamping of the
left or right hepatic artery slows hemorrhage, it can be ligated anhd tolerated.

RESECTIONAL DEBRIDEMENT - used for removal of nonviable parenchyma at


periphery. Will incur blood loss with this maneuver, so should be used in those that
can tolerate blood loss.

PACKING - used not necessarily for the laceration but around the liver between the liver
and diaphragm/anterior chest wall to compress the edges of the laceration together.
Take to SICU and remove after 24-48 hrs. Use for grade IV and V and those who
develop coagulopathy.
- should be used for patients who develop the triad of hypothermia, acidosis and
coagulopathy. If patient is sufficiently ill, skin can be closed with towel clip and
patient rewarmed. COMPLICATION - ABDOMINAL CAOMPARTMENT
SYNDROME - when pressure increases to 20 mmHg, caval return decreases. Take
patient back, remove blood and packs and re-pack.

109
HEPATIC RESECTION - HAS LARGELY BEEN REPACED BY OTHER
MODALITIES. Morbidity of 50%.
2 SITUATIONS: Extensive injury to the left lateral segment of liver.
Packing has been successful and necrotic tisse leaves right or left lobe
nonviable.

HEMATOMAS - smaller grade I and II should be observed. larger can be explored,


packed or left6 alone. Those that are expanding should be explored.

DRAINS - USE CLOSED SUCTION DRAINS.

OMENTAL PACKING - fills defects and buttresses sutures.

COMPLICATIONS - INFECTION, BILOMAS (wait for fistula to mature),


HEMOBILIA (treat with arteriogram/emblization).

# 47 - Trauma with RUQ Retroperitoneal Hematoma


25 y.o. maleS/P MVA from referring institution found on exploratory laparortomy to
have a “subhepatic hematoma” the local surgeon could not handle.

Issues:
a.)How would you approach this patient referred after trauma?
b.) What studies would you like to get for initial evaluation assuming the patient is
stable?
c.) A duodenal perforation is noted on preop UGI, what would you do? How would you
assess the common bile duct and the pancreatic duct?
d.) How would you repair the duodenal perforation?

# 48 - Penetrating Trauma to RUQ with Retroperitoneal


Hematoma
18 y.o male with gunshot to RUQ presents in shock to the E.R.

ISSUES:
a.) ABC’s
b.) What do you do?
c.) Intraoperatively, you find a nonbleeding liver laceration, a hole in the second portion
of the duodenum and retroperitoneal hematoma. How do you proceed with midline
retroperitoneal hematoma- how do you get proximal and distal control?
d.) how do you evaluate the pancreaticoduodenal complex? how would you manage
injuries?how would you manage the duodenal perforation?
e.) How do you manage injury to the IVC?

110
ANSWER:
Airway = for those comatose or obtunded.
Apneic = intubate orally.
Apneic with facial trauma = emergency cricothyroidotomy.
Breathing but need airway control = nasotracheal intubation.
Breathing but combative = rapid sequence induction with cri coid pressure and in-line
traction.
Breathing = pulmonary ventilation is necessary. Confirm by physical exam - no
ventilation can occur because of poor EET placement, hemothroax, pneumothorax,
aspiration of teeth and shit, pulmonary contusion.
Circulation= a.k.a. shock - 14 gauge IV X 2
- draw blood for HCT and T&C ( and other labs)
- Adults = 2L of LR.
- Peds = 20cc/kg
Response - thoswe that improve and stabilize are unlikely to have ongoing bleeding
and can be approached systematically. Those that do not improve or improve
transiently and then deteriorate arelikely to have ongoing hemorrhage and should be
taken to OR.

Secondary Survey - detailed physical exam . Rectal exam , Foley, NGT. CXR, pelvic
X-ray.

PENETRATING TRAUMA - EXPLORE


BLUNT TRAUMA - difficult situation is identifying pancreatic injury in the absence of
other indications for exploration ( shock, peritonitis, + DPL). Many ot these will
remain asymptomatic for a long period of time. Generally associated with high
energy impact to the epigastrium.

AMYLASE - not a great indicator of pancreatic injury. Elevation without pain should
prompt repeat measure. Development of pain and failure of the amylase to normalize
should prompt investigation with CT scan or ERCP. Problem with CT is that it has
only subtle signs of injury early. If first scan didn’t show anything and symptoms
persist, repeat.
ERCP- good for hemodynamically stable patients with persistent elevated amylase and
pain. Ideally this could be performed within the first 12 hrs of injury. Transected
duct warrants surgery.
INTRAOP ERCP - when injury to duct cannot be seen on careful examination . Can be
done by duodenotomy, ampulla cannnulation or by transection of the tail and distal
duct cannnulation.

INTRAOP EVALUATION - signs warranting pancreatic evaluation: midline hematoma,


edema around pancreatic gland and lesser sac, retroperitoneal bile staining.

111
EXPOSURE - divide the gastrocolic ligament outside of the gastroepiploic vessels.
Retract the stomach anteriorly and superiorly. To see the head and uncinate process,
Kocherize the duodenum medially to the superior mesenteric vessels. Mobilization of
the hepatic flexure of the colon helps.
- to expose the tail for injury, perform a medial rotation of the colon, spleen and
pancreasfor bimanual palpation of the tail.
PENETRATING TRAUMA - if the injury has not involved the substance of the
pancreas, no ductal study is necessary. Duct study necessary if the substance of
pancreas is injured.
BLUNT TRAUMA - can have transected duct with the peritoneal covering intact or with
the gland not completely transected.

TECHNIQUE OF INTRAOP PANCREATOGRAPHY -


1.) 18 gauge angiocath in to gallbladder - give morphine to induce sphincter contraction
and inject 20-30 cc of water soluble contrast.
2.) Duodenotomy and ampulla cannulation -
3.) Tail of pancreas transection and duct cannulation
4.) Transected gland and duct - cannulate and get dye study of proximal duct to
visualize the duct that will remain. Use 5 Fr. feeding tube and 2-5 cc of
contrastunder low pressure.

TREATMENT OF PANCREATIC INJURY

CONTUSION AND LACERATION WITHOUT DUCT INJURY -


these account for 80% of injuries. These should be treated with hemostasis and drainage
with soft, closed suction drains. Attempt to close laceration are as likely to make the
situation worse as better. If the amylase content of drainage is that of serum, remove
drains at 24-48 hrs. If amylase is high, leave drains in until output is low.
Nutrition - oral diet as soon as possible. If ileus persists because of pancreatic injury
feed via preplaced J-tube with elemental formula.

DISTAL TRANSECTION/DISTAL PARENCHYMAL INJURY WITH DUCT


DISRUPTION
- treat with distal pancreatectomy with or without splenectomy. Pancreaticogram via
transected ductto assess remaining proximal duct.
Normal duct - staple the pancreas with stapler, but individually close the duct with
figure of eight Prolene, butress with omentum and place a drain.
Abnormal proximal duct/stricture - distal resection of gland using either mattress sutures
or stapler and internal drainage with Roux en Y limb of jejunum.
?Splenectomy - concern over OPSS must be balanced with the blood loss and time in
trauma patient when considering splenic conservation.
NUTRITION - PLACE J-TUBE BEFORE CLOSING.

PROXIMAL TRANSECTION OR INJURY WITH PROBABLE

112
DUCT DISRUPTION
- important to perform pancreatography early inb course.
Head and Neck Injury without Duct injury - provide wideexternal drainage.
Head and Neck injury, unstable patient, Duct injury unclear - wide drainage.
Proximal Duct injury, ampulla+duodenum spared - pancreatectomy distal to duct injury
with wide drainage of transected pancreas.
- if the amount of pancreatic tissue remaining with this will leave patient with
inwsufficiency, the tail of the pancreas can be preserved and anastomosed to a Roux
en Y limb of jejunum. This is done less frequently because of complications and the
generally good result with subtotal pancreatectomy.
NUTRITION - PLACE J-TUBE BEFORE CLOSING.

COMBINED PANCREATICODUODENAL INJURY


NEED:
CHOLANGIOGRAM - can be performed thru the gallbladder using 18 gauge needle and
20-30 cc water soluble contrast.
EVALUATION OF AMPULLA -
PANCREATICOGRAM - may be done thru the injury site or via tail of pancreas
transection and duct cannulation.

KEY -> IF CHOLANGIOGRAM SHOW FLOW OF CONTRAST INTO THE


DUODENUM WITHOUT EXTRAVAZATION, THE DISTAL
COMMONBILE DUCT AND AMPULLA ARE INTACT. THE DUODENAL
INJURY AND PANCREATIC INJURY (BASED ON INJURY AND
PANCREATICOGRAM) CAN BE TREATEDSEPARATELY

PANCREATICOGRAM -if it cannot be obtained intraop and duct cannot be assessed,


place multiple drains around the head rather than a total pancreatectomy.

DIVERSION OF GASTRIC CONTENTS IN SEVERE COMNBINED


DUODENAL AND HEAD OF PANCREAS INJURY -
A. ) DUODENAL DIVERTICULIZATION - consists of primary closure of the
duodenal wound, antrectomy , vagotomy, T-tube drainage of CBD, lateral tube
duodenostomy. Idea is to divert gastric contents and biliary contents away from the
duodenal repair, provide nutrition thru the gastrojejeunostomy and provide a
controlled lateral fistula.
B.) PYLORIC EXCLUSION- easier procedure with less resection and no biliary
drainage. Make a gastrotomy inferiorly on antrum along greater curve. Grasp
pylorus with babcock and close pylorus with 0 suture and construct loop
gastrojejunostomy. Diverts gastric flow away from duodenal injuries until they heal.
The pylorus will open in 2 weeks to 2 months and the gastrojejunostomy will close.

113
PANCREATICODUODENECTOMY - only for massive injuries of the duodenum and
head of pancreas with destruction of ampulla with distal CBD or proximal pancreatic
duct making reconstruction impossible. When indicated this procedure is really just
debridement of devitalized tissue. Can be performed in situation where patient is
stabilized from associated vascular injury.

COMPLICATIONS:
FISTULAS - more frequent with pancreaticoduodenal injuries. Most of these are minor
(<200cc/day) and will spontaneously resolve in 2 weeks. High output failures (>700
cc/day) will take longer to close. I f the volume doesn’t decrease in 10 days, get
ERCP to evaluate pancreatic duct. Treat patient via preplaced J-tube or TPN.

ABSCESSES - should be drained percutaneously or operatively as soon as possible.

SECONDARY HEMORRHAGE - should be treated operatively.

PSEUDOCYSTS - major deeterminant is the integrity of the pancreatic duct.If the


duct is intact, percutaneous drainage will be successful. If the duct has been injured
and the injury was missed with initial evaluation, percutaneous drainage will convert
the pseudocyst into a chronic fistula. ERCP is done before percutaneous drainage. If
abnormal, surgery is necessary for partial gland resection.

DUODENAL INJURIES -
DIAGNOSIS - should obtain a serum amylase when injury is suspected, though this is
not sepcific for duodenal injury. A persistently elevated amylase or rising amylase
should rasise suspicion of pancreatic duodenal injury. If retroperitoneal duodenal
rupture is suspected, it can be confirmed by CT scan with IV and PO contrast or an
UGI with Gastrograffin (or Barium if Gastrograffin is negative).

TREATMENT - 85% of injuries can be repaired with primary anastomosis. Overall


mrobidity and mortality is related to the severity of the injury.Mild injuries have 0%
mortality and 2% fistula rate. Major injuries have 6% mortality and 10% fistula rate.

DETERMINANTS OF SEVERITY MILD SEVERE


Agent stab Blunt or missile
Size <75% wall >75% wall
Duodenal site 3, 4 1, 2
Time from injury to surgery <24 hrs. > 24 hrs.
Adjacent Injury No CBD injury CBD injury

Mild injuries = primary repair


Severe injuries = more complex repair

EXAMPLUS GRATIAS:

114
DUODENAL LACERATION WITH NO PANCREATIC INJURY - primary repair with
no diversion.
DUODENAL TRANSECTION - mucosal debridement and primary repair can be used
for all transection s except those involving the ampulla.
- can lso bring a Roux en Y limb to the proximal duodenal defect and
closure of the distal duodenal defect. Of course this is more work.

PPROTECTING PRIMARY DUODENAL REPAIR IN SEVERE INJURY -


1.) Omental patch - data has not shown benefit.
2.) Serosal patch with loop of jejunum. - data has not shown benefit.
3.) Pyloric exclusion - see above
4.) Duodenal diverticulization - see above.
- complications of gastric diversion is marginal ulceration at the site of the
gastrojejunostomy.
5.) 3 Tubes - gastrostomy for decompression, jejunostomy proximally for decompression
and distal feeding jejunostomy.
** regardless of type of diversion (3-5.) , data shows that some type of diversion is
benefical and lowers mortality and fistula rate.

DUODENAL HEMATOMA - many will obstruct within 48 hrs. from fluid shifting into
the hematoma. Diagnosis made by Gastrograffin swallow (followed by Barium for
delineation of detail). Coiled spring sign or stacked coin signs.
- MUST BE ABLE TO RULE OUT ASSOCIATED INJURIES, ESPECIALLY TO
PANCREAS WHICH OCCURS 20%.
- start NGT suction and TPN. OK if looks obstructed. If signs of obstruction persist,
repeat upper GI at 5-7 days.
-- explore at 2 weeks if conservative therapy has failed to yield results to look for
contained perforation or injury to the head of the pancreas which would lead to
obstruction.

- INTRAOP - if duodenal hematoma is seen introp, it should be explored via


complete Kocher maneuver to look for perforation. This will often release a
subserosal hematoma.

# 49 - Shotgun Injury to Flank


22 y.o. male is brought to E.R. after suswtaining a shotgun blast at close range to his right
flank. It is immediately appearent that the peritoneal cavity has been entered. His BP
is 70/palp., heart rate = 150, respirations = 42, anxious but alert and oriented.

Issues:
a.) Preoperative resuscitation, ABC’s.
b.) Preoperative workup, if any>

115
c.) Intraoperatively the patient is found to have a complete transection of the ascending
colon, multiple small bowel injuries from the shotgun pellets, a 6 cm segment of R
mid ureter is missing , and there is a retroperitoneal hematoma just to the right of
the midline which proves to be a caval injury. How would you go about dealing with
these injuries? What are the options for repairing the damaged ureter in this setting?
If the ureter is repaired primarily over a stent, and there is a leak postoperatively,
what caqn be done nonoperatively to assist healing.

# 50 - Gunshot Wound to Abdomen


22 y.o. male with gunshot wound to abdomen presnts to E.R. hypotnesive and
unresponsive.

Issues:
a.) ABC’s.
b.) Evaluation of possible ureter injury.
c.) Intraoperatively, you find a hole in the right colon with gross spillage and a large
central hematoma. how would you get vascular control for exploration of this central
hematoma? how does one control the vena cava proximally and distally?

ANSWER:

URETERAL INJURY

KEY - PRIMARILY AN INJURY OF PENETRATING TRAUMA.


DIAGNOSIS - via IVP showing cutoff or extravasation. However, IVP is negative 15%
of time and diagnosis should be made via retrograde ureterogram. However, many
instances present with unstable trauma and do not have the time for a preoperative
IVP.

- with penetrating trauma, often a ureteral injury can be ruled out based on the path of the
penetrating object. However, this may be difficult in the case of a massive
retroperitoneal hematoma. Get an intraop IVP. Dissecting out entire ureter to look
for injury is not a goood idea.
- use IV methylene blue to look for extravasation from the ureter.

TREATMENT -
SHORT OR NO SEGMENT LOSS - primary ureteroureterostomy with stenting or
reimplantation into the bladder.
LONG SEGMENT LOSS -
Lower Injury - reimplantation into the bladder via a “psoas hitch” or bladder pedicle
flap.
- may also try a Transureteroureterotomy
Middle - may try a Transureteroureterostomy.
Upper - either an autotransplantation of the kidney or bowel interposition.

116
Missed Injuries - diagnose with IVP or retrograde urethrogram.

COLON AND RECTAL TRAUMA

DIAGNOSING COLON AND RECTAL TRAUMA

PENETRATING TRAUMA -
Gunshot Injury - 85% organ injury rate. Mandatory laparatomy.
Stab Injury - 33% organ injury rate . Selective laparotomy
- Lateral flank injuries should be highly suspicious for colon injuries.
These patients do not need to be mandatorily explored. May be evaluated by serial
physical exams or with contrast enhanced CT scans if physical examination can not
be performed reliably.
- Anterior abdo wall stab wound. Do not mandate exploration . Should be
followed with physical examination if possible. Those showing signs of persistent
bleeding or those with peritoneal signs should be operated on. Those who cannot be
followed with physical examination should have diagnosis aggressively pursued.
Problem is that RBC limit for positive DPL is not agreed upon for stab wounds. CT
scan is not reliable. Favor laparoscopy or formal laparotomy if colonic injury cannot
be ruled out.

RECTAL INJURY - look for gross blood (100%reliable). Otherwise proctoscopy in


ER.

BLUNT TRAUMA - difficult because associated head injuries make reliable physical
exam difficult . CT scan has several diagnostic and suggestive CT criteria for
colon perforation. Takes small amount of blood (20 cc) to make a DPL positive -
DPL’s can lead to high rate of negative laparotomies.

PREOPERATIVE PREPARATION - MUST HAVE PROPHYLACTIC IV


ANTIBIOTICS. In several studies, blah, blah, blah, blah. Use single coverage with
cefotetan or cefoxitin, or triple coverage with ampicillin, aminoglycoside /third
generation cephalosporin and metronidazole.

TREATMENT OF COLON INJURIES

RESECTION -
REASONS TO RESECT:
1.) Preop shock (BP< 60 mmHg)
2.) Severe hemmorhage ( > 1L)
3.) Injury to more than 2 organ systems
4.) Signficant fecal contamination.
5.) Operation starting 8 hrs after injury.

117
6.) Severely destructive colon wounds.
7.) Loss of abdominal wall requirin g mesh.
Technique - resect to normal bowel, remove bowel close to mesentery to avoid vascular
injury . No need for wide resection. Suture ligatures in vascular structures. Use
stapler to divide bowel. Proceed to anastomosis or stoma.

PRIMARY ANASTOMOSIS -
Technique - use either a stapled side to side or 2 layer hand sewn anastomosis
(though this takes more time.)

COLOSTOMY - should be used whenever there is concern about an anastomosis


failing. Many of the reasons for failure of anastomosis are the same as reasons to
resect rather than close (preop shock, severe hemmorhage, fecal contamination,
multiple organ system injuries, operation long after injury). Can be end or loop
colostomy.
- decision of end colostomy as Hartmann’s procedure or mucus fistula depends on
situation. In situations in which there is hemodynamic compromise, it may make
more sense to do a Hartmann’s procedure and delayed maturation of the colostomy.
In a hemodynamically stable patient , a mucus fistula will make the takedown much
easier and should be peformed.

LOOP COLOSTOMY - for similar indications as end colostomy (protect distal injury
site) but can be performed more quickly. Areas where the loop colostomy can be
created are areas where the colon can be brought out to the abdominal wall with the
mesentery sufficiently loose such that there is no tension ( most frequently the
transverse and sigmoid colon.) to ensure a complete diversion the distal loop may be
stapled. Ostomy can be matured later if the patient is unstable.

PRIMARY REPAIR - in general, the situations in which resection should be performed


are also reasons why a primary repair should not be performed (preop shock, severe
hemmorhage, fecal contamination, multiple organ injuries, operation long after
injury).
- in blunt trauma, the amount of energy needed to create a colon injury will usually
create a colon submucosal hematoma as well as a mesenteric hematoma. These
severe injuries would obviously need resection and favor ostomy.

RECTAL INJURIES - PROXIMAL DIVERSION WITH COLOSTOMY. If rectal


injury occurs above tghe peritoneal reflection should be repaired primarily with
proximal diversion. Those below the peritoneal reflection are difficult to repaair and
often associated with pelvic hematoma which should not be released. Presacral
drainage with closed suction drains exiting the perineum are key to treating injury.
- important to lower the infection rate of related hematoma and pelvic fracture
by irrigating out the rectum

118
-> leave skin open.

Complications = fecal fistula thru wound or entrance site.

# 51 - Penetrating Trauma to Chest


20 y.o. male is stabbed in the left anterior chest and is brought to the ER with a BP of 40
systolic.

Issues:
a.) ABC’s.
b) Patient goes into V-Tach and V.Fib. What do you do?
c.) Is an ER thoracotomy warranted in this situation?
d.) Is an ER thoracotomy indicated in the presence of intractable hypotension in a patinet
with blunt trauma?

45 y.o. male is stabbed in the left anterior chest and is brought to the ER hypotensive and
tachycardic.

Issues:
a.) ABC’s
b.) Pt. remains hypotensive and tachycardic despite fluid boluses. What intervention
would you perform?
c.) Chest tube output is minimal but patient remains hypotensive and tachycardic. What
intervention would you perform?
d.) Pericardiocentesis shows nonclotting blood, what would you do?
e.) Intraoperatively, pt has transection of LAD. What would you do?

Airway = for those comatose or obtunded.


Apneic = intubate orally.
Apneic with facial trauma = emergency cricothyroidotomy.
Breathing but need airway control = nasotracheal intubation.
Breathing but combative = rapid sequence induction with cri coid pressure and in-line
traction.
Breathing = pulmonary ventilation is necessary. Confirm by physical exam - no
ventilation can occur because of poor EET placement, hemothroax, pneumothorax,
aspiration of teeth and shit, pulmonary contusion.
Circulation= a.k.a. shock - 14 gauge IV X 2
- draw blood for HCT and T&C ( and other labs)
- Adults = 2L of LR.
- Peds = 20cc/kg
- check CVP.
- EKG.
Response - those that improve and stabilize are unlikely to have ongoing bleeding
and can be approached systematically. Those that do not improve or improve

119
transiently and then deteriorate arelikely to have ongoing hemorrhage and should be
taken to OR.

Secondary Survey - detailed physical exam . Rectal exam , Foley, NGT. CXR, pelvic
X-ray.

PRESENTATION - may present stable or in complete cardiac arrest.

ER THORACOTOMY -

Prehospital factors predicting poor outcome in patients with trauma to the thoracic
vasculature/heart - absence of vital signs (before hospital), fixed and dilated pupils,
absence of cardiac rhythm, absence of motion in extremities, absence of palpable
pulse, cardiopulmonary arrest.

INDICATIONS FOR ER THORACOTOMY -


1.) CARDIOPULMONARY ARREST SECONDARY TO PENETRATING
THORACIC TRAUMA.
2.) SBP < 60 SECONDARY TO EXANGUINATING HEMORRHAGE OR
PERICARDIAL TAMPNADE.

CONTRAINDICATION TO ER THORACOTOMY -
1.) CARDIOPULMONARY ARREST SECONDARY TO BLUNT TRAUMA

OBJECTIVES OF ER THORACOTOMY - resuscitate agonal patients with


penetrating thoracic trauma, evacuation of pericardial tamponade, control
hemmorhage from massive cardiovascular injury, clamping of descending aorta to
redistribute blood to the carotids and coronaries, control of exsanguinating thoracic
vascular injuries, intracardiac massage.

TECHNIQUE - left arm elevated, thorax prepped widely. Incision made on left from the
sternocostal border beneath the nipple to the latissimus dorsi. Incision carried thru
the skin, subcutaneous tissue and serratus anterior until the intercostal muscles have
been reached. Divide the intercostal muscles with scissors. Finochietto retractor is
put in place and ribs separated. The left lung is elevated medially and the aorta is
localized, it can be compressed against the thoracic vertebral bodies. Aorta can be
cross clamped by dissecting the aorta circumferentially with index finger and thumb
away from esophagus.
- next observe the pericardium which is likely to be bluish and tense. Locate the
phrenic nerve. The pericardial sac is opened anterior to the phrenic nerve and
extended inferiorly and superiorly. ( Make a small incision in the pericardium with
knife, grasp the edges with Allis clamps and elevate, divide with scissors.)

120
- evacuate clot in the pericadial sac. Examine heart to determine the rhythn, the
site of injury, and the blood volume remaining in the cardiac chambers.
A flaccid, empty heart is a bad prognostic sign, as is empty corronary arteries and air
inthe coronary veins.

TREATMENT OF INJURIES IN ER:

VENTRICULAR INJURIES - place finger over hole and oversew with 2.0 prolene.

ATRIAL INJURIES - control with partially occluding Satinsky clamp

LARGE INJURIES - occluse with the balloon of a Foley catheter until definitive
cardiorrhaphy can be performed in OR. Do not attempt to use Tefon or pledgets as
it takes too much time.

- may need to shock heart after repair to establish sinus rhythm with 20-50 joules directly
to the heart.

TECHNIQUES OF CARDIAC INJURY REPAIR


INCISIONS:

ATRIAL WOUNDS - control with Satinsky clamp. Close wound with running 2.0
prolene suture. Running vertical mattress with an over and over baseball stitch.

VENTRICULAR WOUNDS - place finger over wound and close with simple
interrupted or horizontal mattress sutures of 2.0 prolene. Often as the compromised
myocardium becomes more friable, the sutures will tear thru creating a larger
bleeding defect. This is when biosynthetic materials such as Tefon are needed to
buttress the suture line.

CORONARY ARTERY INJURIES - proximal injuries will require cardiopulmonary


bypass to repair. Distal third injuries should be ligated with a “U” stitch. In
desparation, proximal injuries may be ligated, though this will cause infarction and
need for aortocoronary bypass and/or IABP.

ACUTE INDICATIONS FOR THORACOTOMY -


1.) Cardiac tamponade.
2.) Acute deterioration - cardiac arrest (ER thoracotomy).
3.) Vascular injury at the thoracic outlet.
4.) Loss of chest wall substance .
5.) Massive air leak in chest tube.
6.) endoscopic or tracheal injury by radiography or endoscopy.
7.) Radiographic evidence of great vessel injury.
8.) Penetrating injury through the mediastinum

121
9.) bullet emboli to the heart or pulmonary artery.
10.) Massive (>1500 cc) or continuing (>250 cc/hr).

CHOICE OF INCISIONS:
1.) LEFT ANTEROLATERAL THORACOTOMY - should be used for resucitation
under circumstances of acute deterioration or cardiac arrest. Right anterolateral
thoracotomy has little use in trauma.
2.) TRANSSTERNAL ANTERIOR THORACOTOMY - an extension of the
anterolateral thoracotomy across the sternum (sort of a clam shell incision) - this
allows better exposure to the heart . Will need to ligate the internal mammaries.
3.) POSTERLATERAL THORACOTOMY - better access to the lower lung, esophagus
and descending aorta. Also allows exposure to the proximal left subclavian artery on
left. Right posterolateral thoracotomy - good exposure to the trachea, proximal
esophagus or pulmonary injury. Good exposure to SVC and IVC.
4.) “Book” or “Trapdoor” incision - for exposure to left sided thoracic outlet
injuries.Allows exposure to the long segment of the left subclavian artery and left
common carotid. Composed of a left anterolateral thoracotomy + a median
sternotomy with saw + incision above clavicle (with SCM if necessary). This
incision stretches brachial plexus and is related tocomplications. Use only if
absolutely necessary.
Third interspace left anterolateral thoracotomy with a clavicular incision is a better
choice for proximal control of left subclavian artery injury.
5.) MEDIAN STERNOTOMY - limited use in trauma - only for injuries to the anterior
hear or proximal great vessels. Must be sure that these are the only injuries that need
to be fixed. Incision of choice for ascending aorta and innominate artery injuries.

# 52 - Penetrating Trauma to Base of Neck/Chest


25 y.o. male who is stabbed in the left neck in Zone I just above the clavicle. Pt brought
in without external bleeding to the ER.

20 y.o. male involved in MVA with isolated injury to right chest. Arteriogram shows a
pseudoaneurysm of the proximal right subclavian artery.

Issues:
a.) ABC’s
b.) What tests would you want for evaluation?
c.) Angiogram shows a tapering of the distal L subclavian artery. What would you do?
How would you expose the subclavian artery? how would you obtain proximal and
distal control? how would you repair a laceration? What would you do if a primary
repair gives you a 50% sten osis?
d.) What would you do with a subclavian vein injury?
e.) What are the physical findings of an inferior cord brachial plexus injury? How should
this be treated?
f.) Anatomically, what are the branches of the subclavian artery?

122
Airway = for those comatose or obtunded.
Apneic = intubate orally.
Apneic with facial trauma = emergency cricothyroidotomy.
Breathing but need airway control = nasotracheal intubation.
Breathing but combative = rapid sequence induction with cri coid pressure and in-line
traction.
Breathing = pulmonary ventilation is necessary. Confirm by physical exam - no
ventilation can occur because of poor EET placement, hemothroax, pneumothorax,
aspiration of teeth and shit, pulmonary contusion.
Circulation= a.k.a. shock - 14 gauge IV X 2 - avoid arm IV’s especially on side of
injury. If suspected innominate , subclavian injury put the IV in the leg.
- draw blood for HCT and T&C ( and other labs)
- ABG.
- UA
- Adults = 2L of LR.
- Peds = 20cc/kg

- check CVP.
- EKG.
Response - those that improve and stabilize are unlikely to have ongoing bleeding
and can be approached systematically. Those that do not improve or improve
transiently and then deteriorate arelikely to have ongoing hemorrhage and should be
taken to OR.

Secondary Survey - detailed physical exam . Rectal exam , Foley, NGT. CXR, pelvic
X-ray, UA

MECHANISM :
PENETRATING - GSW, stab traversing the upper chest or the base of the neck. May
also have penetrating injury beneath the clavicle.
BLUNT - severe deceleration injuries.

PREOPERATIVE EVALUATION:
- HCT, UA, ABG, CXR, EKG.

- IV ABX.

ARTERIOGRAM - BLUNT TRAUMA - = strong history of mechanism and widening of


the superior mediastinum more than 8 cm, loss of aortic knob,multiple left rib
fractures, pleural cap or left pleural effusion indicates retrograde femoral
arteriography to detect site of aortic tear. Site of injury at the ligamentum
arteriosum distal to the origin of the left subclavian artery.

123
- PENETRATING = proximity of projectile or penetration to vascular
structures should prompt arteriography in cases of suspected innominate, carotid
and subclavian artery only. All other great vessel injuries will usually present with
hemodynamic instability and severe hemorrhage.
- for subclavian, innominate or carotid injury , arteriogram is critical prior to OR for
operative planning.

OPERATIVE MANAGEMENT -

PREP - chin to knees, supine.

IN EXTREMIS WITH THORACIC OUTLET INJURY - LEFT ANTERIOR


THORACOTOMY WITH EXTENSION TO THE LEFT IF NECESSARY.
If the incision is extended, it should be extended to an upper interspace to enhance
exposurefor proximal control. Obtain proximal control.
- after proximal control is obtained, if the incision has not been extended across the
sternum, it can be extended via median sternotomy to the neck.

NON EXTREMIS THORACIC ARTERY INJURY - HIGH LEFT ANTERIOR


THORACOTOMY IN THIRD INTERSPACE. The proximal subclavian can be
controlled here. The distal control is obtained via a supraclavicular incisionway
fromthe hematoma. This avoids using a supraclavicular incision for exposure and
getting into hematoma without proximal control and killing the patient.
?remove clavicle - only in life saving situation, morbid procedure.
Distal injuries may be approached from a supraclavicular incision away fromthe hema
toma for proximal control.
- those that cannot be repaired should have interposition Dacron graft placed.

RIGHT SUBCLAVIAN ARTERY INJURY - use median sternotomy with right cervical
extension for proximal control

INNOMINATE ARTERY INJURY - should be approached via a median sternotomy. An


extension up the right neck may be necessary.
BLUNT TRAUMA - usually involves the proximal innominate artery.
PENETRATING TRAUMA - usually involves the distal innominate artery.
- repair using lateral arteriorrhaphy with Satinsky and running 4-0 prolene if
possible for a partial circumferential tear.
- if not possible, need to bypass from the ascending aorta to the distal innominate
using a 10 mm Dacron bypass graft. The ascending aorta is clamped with a Satinsky
clamp and the proximal anastomosis is performed. The distal innominate is dissected
out and divided. The distal anastomosis from the graft to the divided distal
innominate is performed end to end.before the hematoma is entered and the injury is
addressed. The injury is repaired. No anticoagulation or bypass needed.
INNOMINATE VEIN INJURY - can be ligated with impunity.

124
PEDIATRIC TRAUMA

# 53 - Pediatric Duodenal Hematoma


10 y.o. boy fell over the handle bars of a bicycle 12 hrs ago. Initially, he had minimal
pain and was brought in for persistent nausea and vomiting and midepigastric pain of
modest proportion. Physical examination shows only a moderate midpigastric
tenderness. Vitals and labs are normal with exception of mildly elevated amylase.

Issues:
a.)Initial care and differential diagnosis.
b.) Diagnostic tests of choice.
c.) Assuming the UGI shows near complete obstruction of the duodenum and CT scan
shows a intramural duodenal hematoma near the head of the pancreas , what is the
diagnosis and treatment.
d.) Is TPN indicated, if so when?
e.) What is the likelihood that this will resolve without surgery? How long would you
wait before intervening surgically? If you did intervene surgically, what would you
do? If you created a defect in the duodenum, how would you repair it?
f.) Supposing that the initial diagnosis was duodenal hematoma by UGI (Note - CT scan
important for ruling out a central hematoma) and conservative treatment is chosen -
what would you do if n the third postop day the patient develops fever, worsening
abdominal pain and CT scan shows a central fluid collection consistent with a
pancreatic injury - what would you do? How would deal with the pancreatic duct
after performing a distal pancreatectomy and splenectomy? (Duodenal hematoma
may cause pancreatic duct obstruction and cause a pancreatic duct leak)

Potential Pitfall - duodenal defect and open pancreatic duct - both requiring Roux en Y
limbs for drainage.

ANSWER:

DUODENAL INJURIES -
DIAGNOSIS - should obtain a serum amylase when injury is suspected, though this is
not sepcific for duodenal injury. A persistently elevated amylase or rising amylase
should rasise suspicion of pancreatic duodenal injury. If retroperitoneal duodenal
rupture is suspected, it can be confirmed by CT scan with IV and PO contrast or an
UGI with Gastrograffin (or Barium if Gastrograffin is negative).

INTRAOP DIAGNOSIS: bille staining of retroperitoneum, periduodenal or


peripnacreatic hematoma, crepitance or “gas bubbles” in retroperitoneum,
saponification of retroperitoneal tissues.

125
TREATMENT - 85% of injuries can be repaired with primary anastomosis. Overall
mrobidity and mortality is related to the severity of the injury.Mild injuries have 0%
mortality and 2% fistula rate. Major injuries have 6% mortality and 10% fistula rate.

DETERMINANTS OF SEVERITY MILD SEVERE


Agent stab Blunt or missile
Size <75% wall >75% wall
Duodenal site 3, 4 1, 2
Time from injury to surgery <24 hrs. > 24 hrs.
Adjacent Injury No CBD injury CBD injury

Mild injuries = primary repair


Severe injuries = more complex repair

EXAMPLUS GRATIAS:
DUODENAL LACERATION WITH NO PANCREATIC INJURY - primary repair with
no diversion. (CLASS I)
DUODENAL TRANSECTION - mucosal debridement and primary repair can be used
for all transection s except those involving the ampulla.
- can lso bring a Roux en Y limb to the proximal duodenal defect and
closure of the distal duodenal defect. Of course this is more work. (CLASS II).
DUODENAL INJURY WITH MINOR PANCREATIC INJURY (no injury to the main
pancreatic duct) - close the duodenal defect and utilize either triple tube, duodenal
diverticulization or pyloric exclusion.(CLASS IV).
DUODENAL INJURY WITH MAJOR PANCREATIC INJURY - use triple tube
,duodenal diverticulization or pyloric exclusion if possible. If not possible, perform
Whipple.

PPROTECTING PRIMARY DUODENAL REPAIR IN SEVERE INJURY -


1.) Omental patch - data has not shown benefit.
2.) Serosal patch with loop of jejunum. - data has not shown benefit.
3.) Pyloric exclusion - see above
4.) Duodenal diverticulization - see above.
- complications of gastric diversion is marginal ulceration at the site of the
gastrojejunostomy.
5.) 3 Tubes - gastrostomy for decompression, jejunostomy proximally for decompression
and distal feeding jejunostomy.
** regardless of type of diversion (3-5.) , data shows that some type of diversion is
benefical and lowers mortality and fistula rate.

DUODENAL HEMATOMA -
- INTRAMURAL DUODENAL HEMATOMA - collection of blood in the
subserosal layer of the duodenal wall . Severity of the hematoma may make the
viability of the duodenal wall questionable. Often affects the second and third

126
portion of the duodenum and may extend proximally to the first or distally to the
fourth.
- because these often present after the trauma 24 hrs - 2 weeks, it is likely related to
the shift of fluid or osmotic forces of the blood in the wall which can cause complete
obstruction.
- because those that present with isolated duodenal injury usually present with
painless obstruction, those that present immediately after trauma (as opposed to
delayed presentation) with abdominal pain and tenderness are more likely to ahave
associated injuries as opposed to isolated duodenal hematoma.

-many will obstruct within 48 hrs. from fluid shifting into the hematoma. Diagnosis
made by Gastrograffin swallow (followed by Barium for delineation of detail).
Coiled spring sign or stacked coin signs.
- MUST BE ABLE TO RULE OUT FULL THICKNESS PERFORATION AND
ASSOCIATED INJURIES, ESPECIALLY TO PANCREAS WHICH OCCURS
20%.
- start NGT suction and TPN. OK if looks obstructed. If signs of obstruction persist,
repeat upper GI at 5-7 days.
ADULTS-- explore at 2 weeks if conservative therapy has failed to yield results to
look for contained perforation or injury to the head of the pancreas which
would lead to obstruction.
PEDIATRICS - trial nonoperative period of 5-10 days. If the patient has not shown
at least partial resolution at 5 days or complete resolution with resumption of
oral intake at 10 days.

- INTRAOP - if duodenal hematoma is seen introp, it should be explored via


complete Kocher maneuver to look for perforation. This will often release a
subserosal hematoma. It should be gently unroofed or evacuated to ensure that
the duodenal wall is intact.

DRAINAGE OF HEMATOMA
- TECHNIQUE - short longitudinal incision is made over the lateral aspect of the
duodenum over the hematoma. Hematoma is evacuated with irrigation and sponges.
Careful not to injure the mucosa (if question of a defect, use methylene blue via
NGT)
- if the hemotoma extends to the fourth portion of the duodenum, the incision should be
extended to evacuate the hematoma. Specific bleeding points should be ligated.
Close the proximal portion of the incision with interrupted sutures, but leave the
midportion open to promote drainage and prevent reformation of the hematoma.
Leave drain.

# 54 - Pediatric Splenic Trauma/ Head Injury

127
8 y.o. male with left chest and abdomenal trauma. Broken ribs 10,11,12 on left.
BP=80/60 with a pulse of 130.

Issues:
a.) ABC’s and initial fluid resuscitation (how much for a bolus)
b.) Pt. stabilizes after fluid giving enough time for CT scan which shows a splenic
fracture and subcapsular hematoma. Pt. then becomesunstable, what do you do?
Would you transfuse? If so, how much?
c.) If you are forced to remove the spleen, how would you treat this patient long term
(oral antibiotics) and with vaccine?

d.) During the procedure the patient blows a pupil, how would you perform a Burr Hole?

CRANIOCEREBRAL INJURIES

A= Airway - unconscious children should be intubated.Prevents hypoxic injury on top


of traumatic injury. Intubate orally if possible, otherwise cricothyroidotomy followed
by tracheostomy.
Chossing ETT - can be approximated by choosing a tube that approximates the size of
the nares or the fifth digit. Use uncuffed tube. Don’t nasotracheally intubate.
Cricothyroidotomy - use a 12 or 16 gauge angiocath and institute jet ventilation.

B = Breathing - check ventilation.


C= Circulation
- estimated blood volume in child is 80mL/kg. (10 kg kid = 800cc total blood
volume.) Hypotension is a late maifestation of shock and need to recognize other
signs such as tachycardia, disorientation, tachypnea, decreased capillary filling.
IV’s = need 2 peripheral IV’s, should be 20 gauge or larger . Preferred sites are the
cephalic (antecubital fossa), greater saphenous, or external jugular.
Interosseus line - can be used to children up to 6 years. Place 2-3 cm below the tibial
tuberosity. Can be used as an emergency line until peripheral line can be established.

FIRST BOLUS = 20 ML/KG LR if hypotensive


SECOND BOLUS = 20 mL/KG LR if no response to first bolus.
THIRD BOLUS = 10 mL/KG of type O/Rh (-) or type specific blood. If third bolus is
not successful, take patient to OR.

- CHILD IN COMA FROM TRAUMA SHOULD BE TREATED AS AN


EMERGENCY AND INVESTIGATED AND EVALUATED AS IF A SPACE
OCCUPYING LESION WERE PRESENT.

ASSESSING NEUROLOGIC FUNCTION -


LEVEL OF CONSCIOUSNESS - GCS
MOTOR COMMANDS = (obeys = 5, extensor response = 1)

128
EYE OPENING = (spontaneous =5, nil = 0)
VERBAL RESPONSE = (norm al = 5, nil = 0)

Normal = 15
Severe Brain Injury = 8
Brain Dead = 3.
- need to add pupil exam to GCS to get sufficient neuro exam.

INITIAL ASSESSMENT AND TREATMENT


- if abnormality on exam is noted, measure to control ICP should be instituted.
1.) HYPERVENTILATION - for a PaCO2 of 25-30 mmHg.
2.) ELEVATE THE HEAD
3.) OSMOTICS - 0.25-2.0 gm /kg (1.5 gm/kg good estimate)

RADIOGRAPHIC EVALUATION - CT scan of the head is the prefrred test - any child
who is not normal after trauma gets a head CT. Any child with suspicion of neuro
injury should have CT. If the CT is normal, a repeat is indicated for any neuro
deterioration because of delayed hemtoma, swelling.
- if no time for CT because of hemodynamic instability, take to OR for air
ventriculogram to look for mass lesion.
- if patient is deteriorating rapidly and there is no time for CT, take to OR for
burr hole or craniotomy
- mass lesion diagnosed, take measures to control ICP and then take patient
toOR to control mass lesion.

Who gets ICP monitor - those who are unconscious befor e surgery and those that are in
coma but do not have mass lesion.

INTRACRANIAL PRESSURE
NORMAL = 10 mmHg or less.
ELEVATED, BUT NOT DETRIMENTAL = 10-20 mmHg.
ELEVATED, NEED TREATMENT = greater than 20 mmHg.
NEED ICP = lower than 25 mmHg.

GUIDELINES FOR TREATMENT - if neurologic changes/deterioration occur at any


ICP , it should be treated. If no changes occur at any ICP, keep ICP lower than 25
mmHg.

SPECIFIC INDICATIONS -
EPIDURAL HEMATOMA - occur because of a tear in a branch f the middle meningeal
artery in dura, appear as lens shaped mass on CT scan. Can present with “lucid
interval” and then deteriorate or present with coma from head trauma with certain
neuorlogic status and then deteriorates. Can also have small epidural which do not

129
show mass effect or shift and these don’t require surgical treatment.(This is sthe only
situation in which an epidural does not require immediate surgery.)

SUBDURAL HEMATOMA - need to separate the acute from the chronic subdural
hematoma. These occur from tearing of bridging vein of sagittal sinus , direct
trauma with tearing of cortical vein or tearing of sinus.
- symptoms of an acute subdural are related to the parenchymal injury and the mass
effect of the clot. Treatment depends on the clinical situation and the size of the clot.
If the patient is thought to have severe neuro dysfunction from a clot, should have
craniotomy to remove clot. However, if there is only a small rim of clot and patients
neuro status is stable, it is likely that dysfunction is related to parenchymal injury and
unlikely that removal of a small amount of clot will have much effect.

SURGICAL DECOMPRESSION
- treatmant for epidurals and acute subdurals is surgical decompression as rapidly as
possible. Timing is critical, later than 4 hrs. leads to decrease in outcome.
Should be performed immediately in those who develop symptoms of herniation
in face of maximal medical therapy and clinical evidence of a mass lesion
(dilated pupil on one side)
- usually controlling ICP with hyperventilation and osmotics can temporize until
craniotomy can be performed. If not a temporoparietal “burr hole” needs to be
performed.
LOCATION - 1.5 - 2 fingerbreadths anterior and superior to the ear on the side of the
mass lesion and sthe blown pupil.

SPLENIC TRAUMA IN PEDIATRIC AGE GROUP


- GOAL OF TREATMENT IN CHILDREN IS TO SALVAGE SUFFICIENT SPLENIC
TISSUE TO PRESERVE IMMUNE COMPETENCE.

CLASSIFICATION OF SPLENIC INJURIES-

GRADE DESCRIPTION
I Localized capsular disruption or subcapsular hematoma,
no significant parenchymal injury. (HEMATOMA)

II Single or multiple capsular or parnechymal disruption, transverse


or longitudinal that do not extend to the hilar vessel. May or may
not be intraparenchymal hematoma ( MINOR -
PARENCHYMAL INJURY)

III Deep fractures, single or multiple, transverse or longitudinal,


extending into hilar area and involving major segmental
blood supply. ( MAJOR - FRACTURE THRU
PARNECHYMA TO HILUM)

130
IV Shattered or fragmented spleen or spleen separated from normal
vascular pedicle (SHATTERED SPLEEN)

INITIAL EVALUATION AND RESUSCITIATION:


A= Airway - unconscious children should be intubated.Prevents hypoxic injury on top
of traumatic injury. Intubate orally if possible, otherwise cricothyroidotomy followed
by tracheostomy.
Chossing ETT - can be approximated by choosing a tube that approximates the size of
the nares or the fifth digit. Use uncuffed tube. Don’t nasotracheally intubate.
Cricothyroidotomy - use a 12 or 16 gauge angiocath and institute jet ventilation.

B = Breathing - check ventilation.


C= Circulation
- estimated blood volume in child is 80mL/kg. (10 kg kid = 800cc total blood
volume.) Hypotension is a late maifestation of shock and need to recognize other
signs such as tachycardia, disorientation, tachypnea, decreased capillary filling.
IV’s = need 2 peripheral IV’s, should be 20 gauge or larger . Preferred sites are the
cephalic (antecubital fossa), greater saphenous, or external jugular.
Interosseus line - can be used to children up to 6 years. Place 2-3 cm below the tibial
tuberosity. Can be used as an emergency line until peripheral line can be established.

FIRST BOLUS = 20 ML/KG LR if hypotensive


SECOND BOLUS = 20 mL/KG LR if no response to first bolus.
THIRD BOLUS = 10 mL/KG of type O/Rh (-) or type specific blood. If third bolus is
not successful, take patient to OR.

HEMODYNAMICALLY UNSTABLE WITH NO OTHER INJURY ----------> OR.


HEMODYNAMICALLY UNSTABLE WITH OTHER INURIES (PELVIS, LONG
BONE FRACUTRES -------------------------------------------------------> DPL
HEMODYNAMICALLY STABLE -------------------------------------------------> CT

DIAGNOSIS
LABS - Hct/Hgb are often normal even in the face of arterial hypotension, not always
useful.
- base deficit of - 3 on ABG is indicator of ongoing hemorrhage.

DPL - still useful for quickevaluation of abdominal visceral injury inthe


hemodynamically unstable patient. However, particularly in the pediatric population
when nonoperative mangement of splenic injuries plays such an important role, blood
in the abdomen is not necessarily an indication for laparotomy.

CT Scan - offers qualitative (which organs ) and quantitative (how bad) assessement
of injury.

131
TREATMENT

NONOPERTIVE TREATMENT -
CRITERIA FOR CANDIDATE FOR NONPERATIVE TREATMENT
1.) Isolated splenic injury.
2.) Hemodynamically stable.
3.) Alert enough to allow repeated , serial physical examinations.

IF A CANDIDATE - place in ICU for monitoring and obtain CT scans at 3 and 7 days.
Perform serial abdominal exams. NPO and ready for OR. Monitor HCT q6hrs and
vital signs. Child will likely need transfusions to maintain hemodynaic stability.
Should trasfuse to a Hct of 20-25%. Child remains in ICU until need for transfusion
is abated.

HOW MUCH BLOOD - abort for need to transfuse 50% of blood volume in first 24 hrs.
Must set limit for transfusion at outset.

FAILURE OF NON0PERATIVE TREATMENT - hemodynam ic instability or


trasfusions exceeding preset limit.

OPERATIVE THERAPY -
Position - supine
Incision - upper midline.
- on entering, feel the suprarenal aorta to assess arterial perfusion. If bleeding from liver
and spleen, pack these. If circulatory embarassment persists, take the spleen out.
- if patient stabilizes and splenic salvage can be performed, mobilize and examine the
spleen. if the spleen is shattered and must be removed, the splenic artery can be
temporarily compressed on the vertebral column to prevent ongoing blood loss while
spleen is removed.
Mobilization - lift with right hand, place left over right with sponge and elevated with left
hand to expose the lienorenal and phrenicolienal ligaments . Incise ligaments from
caudad to cephalad. Right hand is then placed over left and fingers of right hand used
to develop plain between the left kidney anterior surface and the posterior surface of
the tail of the pancreas. Place 2 laps in LUQ.

Examination - remove clots from the surface, but do not remove clots from within the
edges of the fracture (this will start bleeding again.) If bleeding does recur, compress
the hilar vessels with fingers or clamp. Unless the spleen is shattered or removed
from its blood supply, splenic salvage via splenorrhaphy or parital splenectomy can
be performed.

Options for Splenic salvage -

132
1.) Application of omentum or a toical hemostatic agent, with or without simple capsular
sutures.
2.) Direct reapir of splenic parenchyma and capsule with simple or mattress sutures, with
orwithout pledgets of omentum.
3.) Suture ligation of individual splenic vessels.
4.) Partial splenctomy
5.) Capsular , parenchymal and vessel compression with application of mesh (vicryl or
Marlex)
6.) Large thru and thru mattress sutures perpendicular to the plain of injury.

POSTOPERATIVE MANAGEMENT:
THROMBOCYTOSIS - reserve prophylactic anticoagulation for platelet counts in excess
of 1 million.
VACCINATION - Pneumovax should be given, the timing is controversial and probably
unimportant.
ANTIBIOTICS - should get oral PCN, especially those under the age f 2 yrs. This should
continue until t hey are old enough to understand the significance of the problem and
have the ability to seek medical attention for the signs and symptoms of impending
infection. Antibiotics are used after this therapeutically if there are signs of
impending infection, even if trivial.

# 55 - Airway Management in Head and Neck


Trauma
15 y.o. male presents with massive head and neck injuries after a MVA.

Issues:
a.) How would you establish the airway?
b.) What would you do if an oral airway was not possible? What size tube would you
use?
How would you estimate the appropriate size tube.
c.) Assuming the patient had a larynx fracture and had a trach placed in ER, which no
longer functions, what would you do?
d.) How do you deal with a laryngeal fracture?
e.) Shortly after the placment of the trach, the patient has severe bleeding from the trach
site. What would you do?
f.) how would you repair a tracheoinnominate fistula?

ANSWER:

A= Airway - unconscious children should be intubated.Prevents hypoxic injury on top


of traumatic injury. Intubate orally if possible, otherwise cricothyroidotomy followed
by tracheostomy.

133
Choosing ETT - can be approximated by choosing a tube that approximates the size of
the nares or the fifth digit. Use uncuffed tube. Don’t nasotracheally intubate.
Cricothyroidotomy - use a 12 or 16 gauge angiocath and institute jet ventilation.

ABSOLUTE INDICATIONS FOR INVASIVE AIRWAY MANAGEMENT :

ACUTE AIRWAY OBSTRUCTION - direct laryngeal trauma, especially when


associated with fractures of the thyroid cartialge, produce submucosal hemorrhage
and rapidly progressive edema. Can present with life threatening compromise of an
airway when little evidence of external trauma exists.
- expanding neck hematomas from arterial lacerations can cause obstruction from
deviation of the larynx and from mechanical compression of the larynx.
- transected airway occurs at the cricotracheal junction. The trachea retracts into the
lower neck and the surrounding tissue provide a “pseudo - airway”. External
pressure on the neck can cause this to collapse.

APNEA - usually related to cerebral or spinal cord injury (above C4).

HYPOXIA -

EXPANDING HEMATOMA OF THE NECK -

CRICOTHYROIDOTOMY - indicated for any patient who has the indication for
intubation but the trachea cannot be intubated for some reason. Relatively
contraindicated in pediatric age group (those under 12 years of age) because of the
risk of subglottic stenosis.
TECHNIQUE - palpate the cricothyroid membrane, stabilize larynx. Make a transverse
incision over the membrane . Locate the membrane with finger and make a small
incision.
bluntly open with hemostat and insert a standard 8 mm outside diameter tube.

TRACHEOSTOMY - indicated for patient withacute laryngeal trauma in whom


placement of a tube thru the cricothyroid space can contribute to the patients
exisiting injury and in those patients under 12 yrs. in whom a cricothyroidotomy is
contraindicated (relatively) . would like to have the patient intubated orally first, but
this is undesirable in patients who have severe laryngeal fracture. May use a
percutaneous kit if available.

LARYNGEAL FRACTURE - symptoms include respiratory distress, hoarseness,


hemoptysis and subQemphysema. Should have antomy restored to normal to prevent
airway and speaking difficulty. Leave tracheostomy in for 4-8 days to allow
swelling to diminish.

134
TRACHEOINNOMINATE FISTULA - may arise from erosion of cuff or tracheosotmy
tube in to the underlying innominate artery. May see “sentinel bleed” into the
trachea.
Confirm diagnosis - deflating cuff will result in major hemorrhage.
Temporary control - overinflate trach. cuff or digitally compress the innominate artery
against the sternum thru the tracheal stoma while the patient is transferred to the OR.
Becuase of contamination, a graft should not be placed. Perform a median sternotomy
with right neck extension to expose the innominate artery. Resect the involved artery
and oversew the ends using nonabsorbable suture and cover suture line with thymic
or mediastinal fat. Do not resect trachea. Incidience of neuro compromise with
acute ligation of the in nominate artery is low.

# 56 - ACLS/Basic Protocols
As preparing to do a RIH repair on 73 y.o. male with no previous medical history.
Procedure has begun under local anesthesia with 1% lidocaine. As procedure begins,
notified the patient has no pulse.

Issues:
a.) Differential diagnosis, what do you do? Explain CPR?
b.) Assuming he is resuscitated promptly, do you proceed with hernia repair?
c.) What kind of hernia repair would you perform and why?
d.) What would be the postoperative management of this patient?

# 57 - Management of ARDS
Pt. in SICU one week S/P resuscitation for hemorrhagic pancreatitis. Ventilator settings
are FiO2 of 40%, TV=800, rate = 12, PEEP=5. Nurse calls you with pCO2 = 32,
pH=7.32, pO2=52.

Issues:
a.) What is your interpretation of these blood gases? Differential diagnosis for
hospitalized patients with this ABG? How would you rule out P.E.?
b.) What changes would you make in the ventilator settings? How would you adjust
these as the patients status changes?
c.) What ventilator settings would you use if the respiratory pressure was 44? what can
you do to reduce barotrauma? What is the problem with elevated inspiratory
pressures?
d.) When would you place a Swan Ganz catheter and why?

ANSWER:

DIFF. DIAGNOSIS:
1.) ARDS
2.) Pulmonary Emboli
3.) Pneumonia

135
4.) MI with cardiac failure.
5.) Pneumothorax
6.) Sepsis
7.) Atelectasis
8.) Pleual Effusion

Diagnosis:
PaO2 - need to have Pa02/Fio2 ratio of less than 150 (130 in this case).
pH - slightly elevated
PaCO2 - mid to low 30’s.
CXR - though picture of ARDS will often not manifest itself for 24 hrs or more, it is
useful to rule out other conditions such as pneumonia , evidence of pulmonary edema
from cardiac failure, or pneumothorax.
Increase Fi02 - if the Pa02 rises over 100 with increase in Fi02, this is likely due to
increased dead space ventilation and will respond to only an increase in Fi02. If
increased Fi02 brings only a moderate increase in Pa02, there is likely an
intrapulmonary shunt (like ARDS) and more than increasing Fi02 will be needed for
adequate therapy.

Pulmonary artery catheter - in ARDS should show normal PCWP and normal to slightly
elevated cardiac output. Those in congestive heart failure will show elevated PCWP
and low cardiac output.

V/Q Scan - use to rule out pulmonary emboli.

CHOICE OF VENTILATION:
VOLUME CONTROL - pump a preset volume of air.
PRESSURE CONTROL - pump air until a preset airway pressure is reached
Problem with pressure ventilation - as the chest wallcompliance gets stiffer, the less
volume will be delivered.

IMV- ventilator provides breath at set rate to supplement the patients own spontaneous
breathing (spontaneious breathing plus extra breaths)
AC - whenever patient makes inspiratory effort, the ventilator provides a given volume .

TREATMENT OF RESPIRATORY INSUFFICIENCY

HYPOXEMIA -
Increase FiO2 - can help by increasing the alveolar p02, and increasing the amount of
02 that diffuses into the blood stream. However, increased 02 can be toxic.
Increased FiO2 does not help in areas of the lung where there is little gas exchange
because the alveoli are collapsed.

136
-> can elevate Fi02 to .50, but only for a couple of hours. Once the Pa02 reaches 85-
95, the Fi02 can be lowered. Use the lowest Fi02 possible to keep the Pa02 greater
than 60.

PEEP/CPAP - improves the oxygenation by decreasing the amount of intrapulmonary


shunt and decreasing the ventilation perfusion mismatching. Acts to open the
partially or completely collapsed alveoli (alveolar recruitment) and improve
compliance to the lung.

Problems with PEEP -


1.) Decreased Cardiac Output - negates the normal enhanced cardiac return with a
negative intrathoracic pressure . This decrease in cardiac output can be overcome
by fluid and increasing the intravascular volume, however, judging how much fluid to
give requires of Swan Ganz catheter. If PEEP > 10cm H20 needed, or a fall in
cardiac output/hypotnesion with anyPEEP, place a Swan Ganz catheter.

2.) Direct Pulmonary Barotrauma - rarely seen with peak inflation pressures less
than 60mmHg.

VENTILATION
- initially , the patient can be hypocapnic (as in this case), but eventually the patient will
be come hypercapnic as ventilation fails. Can start ou t ventilation with high tidal
volumes if desired (12-15cc/kg or twice normal), but as the disease progresses and the
chest wall compliance decreases and lungs become stiffer the high tidal volumes will
lead to increased airway pressures and more barotrauma. AS ARDS PROGRESSES
AND BAROTRAUMA IS CAUSED BY ELEVATED AIRWAY PRESSURES,
SMALLER TIDAL VOLUMES ARRE RECOMMENDED.
-useful to generate a tidal volume- respiratory pressure curve to find the optimal tidal
volume. Generally, a tidal volume is selectedproducing an airway pressure of less
than 60cmH20.

ALTERNATIVE VENTILATION - progressive decreases in lung compliance and


increases in minute ventilation. May opt for “pressure control” ventilation which
will reduce mean airway pressures. In addition, inverse ratio ventilation (IRV) can
be used along with pressure control ventilation to keep alveoli open longer while
decreasing airway pressures.

# 58 - Management of Pt. in Need of Surgery after Recent MI


Cholecystectomy - 76 y.o. female 2 months S/P MI presents with cholecystitis.
Rectal CA - 70 y.o. female 2 monthe S/P MI with near obstructing rectal CA.

Issues:
a.) How do you want to evaluate the patient’s cardiac status ?

137
b.) Assuming the cardiologist places the patient at a moderate to high risk of surgery,
what will you decide to do?
c.) What surgical options will allow little or no anesthesia/surgical intervention?

ANSWER:

ASSESSMENT OF THE CARDIAC RISK

GOLDMAN CLASSIFICATION:

CRITERIA POINTS
HISTORY
Age >70 5
MI in last 6 months 10

PHYSICAL EXAM
S3 or JVD 11
aortic Stenosis 3

EKG
Rhythm other than sinus 7
>5 PVC’s per minute 7

GENERAL STATUS
p02< 60 , pCO2 >50, K<3.0, HC03<20, 3
BUN >50 or Cr >3 or bedridden from non-
cardiac disease.

OPERATION
Intraperitoneal, intrathoracic or aortic 3
Emergency 4
____________________________________________
53

NYHA CLASSIFICATION
CLASS POINTS RISK

CLASS I 5 OR LESS 1%
CLASS II 6-12
CLASS III 13 - 25
CLASS IV > 26 ONLY LIFE SAVING OPERATIONS

PATIENTS WITH MI IN LAST 6 MONTHS = 30-40% CHANCE OF


REINFARCTION

138
REINFARCTION = 70% MORTALITY
25% OF THOSE UNDERGOING SURGERY AFTER HAVING HAD MI IN
LAST 6 MONTHS HAVE CARDIAC DEATH.

NOTE - each of these patients, age 70 or greater, MI in last 6 months, intraabdominal


procedures, +/- emergency without additional assessment for labs, EKG or physical
exam have scores which place them in class III or class IV - these patients should
have lifesaving operations only.

DIPYRDAMOLE THALLIUM - usually predict severe CAD and safer than cardiac
cath.Dipyrdamole induces maximal coronary dilation, thallium redistribution
folowing dipyrdamole indicates hypoperfusion that can be corrected by surgery.

PERIOPERATIVE MANAGEMENT -
- support of the myocardium can lower the risk of infarction and is directed at lowering
the myocardial oxygen demand (determinants are heart rate, contractility and wall
tension).
- supportive therapies of beta blockers and Ca channel blockers aim at slowing heart rate
while nitrates cause venodilation which lowers the ventricular end diastolic volume.
- Pulmonary artery catheter + continuous arterial monitoring = assist in avoiding
episodes of hypotension and maintaining adequate 02 supply while minimizing
mycardial. 02 demand.

ANSWER - WHAT CAN BE DONE TO GET THE PATIENT OUT PAST 6 MONTHS
FROM MI, WHEN THE RISK OF REINFARCTION GOES FROM 30-40% TO 6%
AND STABILIZES. ONLY OPERATE FOR LIFE SAVING SITUATIONS WHEN
NO CONSERVATIVE OR TEMPORIZING MEASURES EXIST.

# 59 - Management of Septic Patient/Pt. in Shock


72 y.o. male with CAD, history of urinary obstruction who now appears septic?

Issues:
a.) Pt. on flor, ill, in respiratory distress, what do you do?
b.) Intubated in ICU, what do you do? Explain how a Swan Ganz catheter is placed.
Antibiotics? How would you manage fluids
c.) Pt. initially had high C.O. and low SVR (sepsis) but now has low C.O. - what could
have happened?
d.) How do you managment this patient who is septic with evolving MI? Ventilator?
Drips? Other parameters?
e.) How would you manage decreasing urine output?

ANSWER:

PART #1 - SEPTIC SHOCK

139
SEPSIS - manifested by fever, tachypnea, tachycardia, isolated end-organ dysfunction.
SEPTIC SHOCK - hemodynamic instability in addition to above .

EARLY SHOCK =“WARM” SHOCK = HYPERDYNAMIC SHOCK


C.O. HIGH + SVR LOW.
SVR is low even if no systemic hypotension present. Venous capacitance decreased
reducing the circulating volume. Because of this decreased filling pressure, the
cardiac output (which is normal or increased ) is not enough for demands resulting in
underperfusion. Fluid resuscititation can restore ventricular filling.
- fluid resuscitiation may make the cardiac output very elevated but this is apporpriate for
the state. No change in cardiac output with fluid resuscitation may reflect
underlying cardiac disease or the hypodynamic phase .

LATE SHOCK = “COLD” SHOCK = HYPODYNAMIC SHOCK


C.O LOW+ HYPOTENSION.
Oliguria and renal failure can arise. Looks like hemorrhagic shock, but with much less
volume depletion. Likely to have decrease cardiac output even with normal filling
pressures. May involve other factors such as underlying heart disease or inadequate
resuscitation.

TREATMENT OF SEPTIC SHOCK


1.) INVASIVE MONITORING - Arterial lines, Swan Ganz catheter, Foley.
2.) RESPIRATORY SUPPORT - 02, intubate if in respiratory distress.
3.) RESTORE EFFECTIVE CIRCULATING VOLUME - Isotonic crytalloid, colloid or
blood products to get the PCWP to 15mmHg.
- to optimize 02 carrying capacity, blood transfusion to get the Hgb above 12 or 13
may help.
4.) ANTIBIOTICS - appropriate for the type of presumed infection, otherwise broad
spectrum.
5.) SURGICAL INTERVENTION - should be withheld until the patient is resuscitated.
6.) INOTRPIC SUPPORT:
HYPERDYNAMIC = HIGH CO+LOW SVR - Dopamine @ 4-10 mcg/kg/min.
HYPODYNAMIC = LOW CO + LOW SVR - Dobutamine @ 2-10 mcg/kg/min+/-
Dopamine.
- Norepi @ 2-4 mcg/min. if needed.

PART # 2 - CARDIOGENIC SHOCK


PUMP FAILURE - with left sided heart failure, fluid which cannot be handled coming
from the right side of the heart will back up into the pulmonary system.
- As the limits of the capacitance of the pulmonary veins are exceeded, pressures within
the pulmonary vasculature rise. At 20 mmHg , pulmonary interstitial edema beings
to form, at 22 - 24, alveolar pulmonary edema forms.

DIAGNOSIS - EKG

140
- CXR - look for signs of heart failure (cardiomegaly, effusions, edema)
- ABG - look for acidosis and adequacy of oxygenation
- Cardiac enzymes - R/O MI.

Swan Ganz catheter = indices consistent with low cardiac output (index less than 2.2),
elevated SVR and elevated filling pressures (usually 18 or greater).

MANAGEMENT OF CARDIOGENIC SHOCK.


OXYGENATION - supplemental 02 and mechanical ventilation . Mechanical
ventilation - beneficial for providing 02 but also for decreasing 02 consumption and
workload on heart by decreasing the work of breathing.

OPTIMIZING CARDIAC SUPPORT - Preload, afterload and contractility using a Swan


Ganz catheter.
Preload - aim for a PCWP of 15-18 mmHg. If it is lower than this , try infusing prbc’s
for a Hgb>12. PCWP above 18 can be manipulated via imporving contractility or
intravascular volume.
Contractility - Dobutamine @ 2- 10 mcg/kg/min. imporves contractility without
increasing rate.
- Dopamine @ 4-10 mcg/kg/min. may be necessary in presence of
hypotension.
- venodilators (nitrates and MS04) and diuretics may be used to lower filling pressures
when pressors are instituted and hypotension is no longer a concern.
Afterload - decrease in resistance to flow may augment cardiac output.
- Nitroprusside @ 0.25-0.50 mcg/kg/min. may be used in epidsodes of
hypertension or in the presence of pressors which will provide hemodynamic
stability.
-> find the optimal place on the Starling curve.

Refractory Hypotension - IABP .


- in cases where shock is determined to be cardiogenic and due to Acute
Myocardial Infarction, consideration should be given to invasive angiography,
angioplasty and possibly surgery if indicated.

# 60 - Oliguric Renal Failure Postop.


70 y.o. male in PACU S/P APR with low urine output, period of hypotension intraop.
H/O MI 5 yrs. ago.
Issues:
a.)What is the differential for low urine output in this patient with low urine output?
(bleeding, severe hypovolemia, transfusion reaction, ATN, MI, ureter injury, sepsis)
b.) What initial interventions should you perform?
c.) Initial fluid boluses has minimal effect, what interventions might you perform now?

141
d.) What lab tests would you order? (Urine lytes, fractional excretion of sodium, renal
scan)
e.) Pathophysiology and treatment of renal failure.

ANSWER:

PATHOPHYSIOLOGY OF RENAL FAILURE = ISCHEMIC AND TOXIC


INJURY

ISCHEMIC INJURY - 02 deprivation leads to injury and reperfusion leads to


reperfusion injury which the kidney is sensitive to. Prolonged ischemia results in
swelling and disintegration of proximal tubule cells, debris collects in the tubule
lumen
TOXIC INJURY - include products of tissue breakdown from ishemic injury or sepsis,
bacterial products , antibiotics and contrast material.

CLINICAL SIGNS AND SYMPTOMS


PATIENTS AT RISK FOR RENAL FAILURE = preexisting renal disease, decreased
renal function because of advanced age or generalized vascular disease, those
experiencing prolonged hypotension or decreased cardiac output, multisystem organ
failure.
HYPOTENSION/DECREASED CARDIAC OUTPUT - ischemic injury alone.
BURN/CRUSH/LIMB ISCHEMIA - toxic injury alone.

PERIOPERATIVE OPTIMIZATION OF RENAL FUNCTION

PREOP ASSESSMENT - RENAL FUNCTION - calculate creatinine clearance in all


patients over 50-60 years.
- CARDIAC FUNCTION - should be carefully evaluated in any
patient over 65 yrs of age.

INTRAOP - titrate fluids to keep urine output at least 0.5 ml/kg/hr. (approximately 200-
250cc/hr while keeping up with blood loss cc for cc). Can judge fluid status with PA
catheter if available. Check periodic ABG’s to assess oxygenation and ventilation.
Note - restoration of blood pressure after hemorrhage intraop or from trauma does not
assure that adequate blood volume has been restored.

POSTOP - usual shit.

DIAGNOSIS OF ACUTE RENAL FAILURE -


1.) Low Urine Output
2.) Rising BUN/Creatinine

142
Potential Problems - Nonliguric renal failure may present with only rising BUN and
Creatinine which may not present for several days (preventing early diagnosis and
treatment). Low urine sodium is present in the patient with low renal flow from
hypotension as well as in the patient who is going into renal failure.

STEP #1 - LOW URINE OUTPUT = less than 0.5ml/kg/hr. Place a PA catheter to


assess volume status and cardiac output. If cardiac output is adequate and patient has
adequate volume status, may start Dopamine at 3 microgrmas per kg per min.
- US - to rule outobstruction.
- Hct/Hgb - rule out bleeding
- UA - look for urine casts or cells.

STEP #2 - URINARY CREATININE, UREA AND OSMOLALITY


Normal Values:
Urine Creatinine/Plasma Creatinine > 2.5/1.0
Urine Urea/ Plasma Urea > 10/1
Urine Osmolality/ Plama Osmolality > 1.1 / 1.0
- when any of these are below critical levels and there is low urine output despite
adequate medical therapy, the patient has impending renal failure and should
be treated.
- diagnosis may be more difficult in patient with nonoliguric renal failure.

TREATMENT OF ACUTE RENAL FAILURE

STEP #1 - using data from PA catheter , continuous arterial monitoring with ABG’s,
Hct/Hgb - need to optimize oxygen carrying capacity. Transfuse to Hct of 30,
supplement 02, pressors in addition to dopamine as needed to augment cardiac
output, NTG as needed for MI, antibiotics as needed for sepsis.

STEP #2 - DIURESIS - Mannitol = 6-12 grams IV


- Furosemide = 20mg, 40 mg if no response in 1-2 hrs., 80 mg if
no response in 1-2 hrs.
- should assist in increasing renal blood flow and decreasing swelling , maintaining
tubular blood flow.

STEP #3 - SUPPORTIVE CARE


- Hct/Hgb = maintain at around 30.
- Control serum electrolytes using fluid s administering 0.5meq/kg/day of sodium.
- total daily fluid intake at 800 - 1000cc/day in addition to urine and other fluid
losses.
- monitoring of serum K+ - use Kayaxelate 25% in 50cc orally or enema as needed
for elevated K+ ( can’t do either one in this case).
- start TPN - renal failure is associated with increased catabolism and increased
protein breakdown.

143
- adjust medications accordingly and change nephrotoxic drugs.

STEP #4 - DIALYSIS
Indications - 1.) Acidosis
2.) Hyperkalemia
3.) Fluid Overload
4.) Symptomatic Uremia - bleeding, nausea

# 61 - Pulmonary Emboli in Postop Patient.


25 y.o. male, 5 days S/P MVA in whci he sufferedbilateral femur fractures, pelvic
fractures, left hemopneumothorax and pulmonary contusion develops shortness of
breath and tachypnea with respiratory rate of 42 with P=120, BP=90/60.

Issues:
a.) Initial resuscitation and evaluation. What is differential diagnosis (pneumonia,
PE,MI,
fat emboli)
b.) Is a lung scan indicated when CXR shows bilateral pulmonary infiltrates?
c.) When lung scan shows intermediate probability, what is the next step?
d.) What do you do when the pulmonary angiogram shows an embolus to the right lower
lobe? (filter)
e.) What kind of filter would you use, how would you place it?
f.) What would you do if prior to placement of a filter the patient develops recurrent
emboli with worsening hypotension - what would you do? What are the indications
and contraindications for lytic therapy?

ANSWER:

Differential Diagnosis: pneumonia, pulmonary contusion, fat emboli, PE, ARDS,


pneumothorax, sepsis, myocardial contusion.

DIAGNOSIS:

LABS: WBC - less than 15,000 will rule out pneumonia effectively.
ABG - will show hypoxemia generally (pO2 < 60 mmHg) but this will not
differentiate PE from other diagnosis. Reduction in PaC02 is the most
discriminating finding for pulmonary emboli. If hypoxemia and jypocarbia not
present is not P.E.
EKG - primary reason is not to get an EKG diagnostic for P.E.
but to rule out M.I.
CXR - also useful to rule out other diagnosis such as pneumonia, pneumothorax,
esophageal perforation or CHF. Is necessary for ventilation scan .

LUNG SCAN -

144
Normal - excludes P.E.

LOW PROBABILITY- LOW SUSPICION - check venous duplex


- HIGH SUSPICION - treat as Intermediate Prob.

INTERMEDIATE PROBABILITY - check for previous cardiopulm dis.


(-) cardiopulm. disease (-) duplex = follow.
(-) cardiopulm disease (+) duplex = treat with anticoag. if poss.
Can’t anticoagulate ( - ) cardiopulm disease = pulmonary angio

(+) cardiopulm disease (-) duplex = pulm. angio


(+) cardiopulm. disease (+) duplex = treat with antioag. if poss.

HIGH PROBABILITY - treat .

PULMONARY ANGIOGRAM - normal test excludes pulmonary emboli.

TREATMENT :

ANTICOAGULATION/HEPARIN - MINOR P.E. (Pa02 = 65-80, PaC02 = 30-35,


tachycardic, anxious+hyperventilating, 20-30% PA occlusion) . Heparin should be
used at 150U/kg to achieve a PTT twice normal, unless contraindicated. Heparin
should not increase bleeding risk , but with this patient ‘s degree of injury, ti would
be best avoided. Contraindicated for this patient because this is not a minor PE
and recent trauma.

INFERIOR VENA CAVA INTERRUPTION/FILTER - MINOR P.E. Should be


used in situations in which anticoagulation can not be used or in situations in which
anticoagulation has failed.

TECHNIQUE - either via femoral or jugular vein, under local anesthesia the jugular or
femoral vein is exposed and controlled with umbilical tape and clamp. Needle is
placed in vein and and guidewire is passed. A venocavogram is performed (should
not be performed if there is clot in the system above the insertion site - should use
opposite side or the jugular vein).
- sheath is passed over wire into the IVC under flouro. Carrier is passed over the
sheath to the desired level . Filter deployed.

THROMBOLYTIC THERAPY - MAJOR P.E. (Pa02 < 65, PaCo2 < 30, elevated
CVP, hypotensive but responds to fluids) MASSIVE P.E. ( PA02 < 50, PaCO2< 30,
greatly elevated CVP, hypotensive that is refractory to fluids and requires pressors) or
MINOR PE ON EXISTING SEVERE CARDIOPULMONARY DISEASE. USE
IN THE ABSENCE OF CONTRAINDICATIONS.

145
ABSOLUTE CONTRAINDICATIONS:
Active Bleeding
Intracranial Disease
Recent Eye Operation

RELATIVE CONTRAINDICATIONS:
Recent Operation ( 10 days)
Recent serious trauma
Postpartum (7 days)
Recent external cardiac massage
Biopsy at site which is inaccessible to compression.
History of GI Bleed / Gastric or Duodenal Ulcer.
Pregnancy
Uncontrolled Hypertension.
Arterial Aneurysm al Disease.
Knitted or Dacron Graft.

DOSE: STREPTOKINASE BOLUS = 250,000 U over 20 minutes.


INFUSION = 100,00 U per hour
T- PA BOLUS = 10 mg
INFUSION = 100 mg over 2 hrs.

HEPARIN CONTINUED AFTER LYTIC THERAPY STOPPED.

PULMONARY EMBOLECTOMY - MASSIVE EMBOLI (Pa02< 50 , PaC02<30,


elevated CVP and hypotension refractory to fluid requiring pressors) . In patients
who have collapse and suspicion of PE, should be anticoagulated with heparin and
started on presssors to support blood pressure while the diagnosis is being
confirmed by angio. Can use Dopamine or Isoproteronol. If the patient responds to
heparin and does not need pressors, place a filter and get out.
- under local anesthesia, exposed the femoral artery and vein, insert cannulas and
place the patient on partial bypass, this allows for hemodynamic support while
inducing general anesthesia. Median sternotomy performed and partial bypass
converted to complete bypass by placing a cannula in the SVC. Incision made in the
main PA and clot removed. Lungs are squeezed to mobilize distal clots proximally to
main PA.

Mortality - 40%. Should really only perform in those who need closed cardiac
compression to maintain BP or in those who catheter embolectomy is unsuccessful.

PEDIATRICS

146
# 62 - Intusseception
10 month old presents to ER with a history of severe crampy abdo pain mainifested by
crying loudly, pulling legs up and writhing. Passes currant jelly stool.

Issues:
a.) What would you do to evaluate the patient?
b.) What is the height of the column of Ba?
c.) BaEdoes not work. What would you do now?
d.) Intraop the patient has an ileocolic intusseception, hwo would you handle this?
e.)There is a mass at the leading edge, how would you manage this? What are the
margins?

ANSWER:
ABDOMEN - soft and nontender initially, tenderness and painb in dicate necrosis.
MASS - palpable on exam 85% of time.
RADIOGRAPHIC APPEARANCE:
EARLY - nonspecific , perhaps empty RLQ and mass at hepatic flexure.
LATE - appearance of mechanical obstruction.

BaE - diagnostic with almost 100% accuracy. Also allows the possibility of
therapeutic reduction for ileocolic intusseception.
All children without peritonitis should have BaE for diagnosis and possible
treatment.
- small bowel to small bowel intusseception is harder to diagnose early because of
the inability to visualize via contrast enema, usually diagnosed later when complete
obstruction presents.

TREATMENT:

BaE Guidelines:
1.) Column of Ba should not exceed 1 meter/ 3 feet 6 inches.
2.) Do not manipulate the abdomen.
3.) Stop the enema if the column has not advanced in 10 minutes.
4.) Do not stop the enema until the Ba refluxes into the ileum, indicating that the
reduction is complete.

- Failure or uncertainty of hydrostatic reduction mandates surgery. If there is


spontaneous reduction druing preparation for surgery/induction of anesthesia,
exploration to confirm this is all that is necessary.
- Viable bowel - manual reduction by milking /compressing the bowel distal to the
intusseception forcing the intussecepted bowel proximally.
- Inability to reduce indicates necrosis at the lead point and should not be pursued.
Resect with anastomosis.

147
- perform appendectomy.

Recurrence = 5% of time after hydrostatic or surgical reduction. Should be treated with


repeat hydrostatic decompression.

OLDER CHILDREN - the older the child, the more likely there is an anatomic
leadpoint - Meckel’s, intestinal lymphoma, cystic fibrosis, polyps, intramural bleed
from henoch-Schonlein purpura. Children older than 4 should have this owrked up
even if the hydrostatic reduction is successful.

ADULTS - associated with malignancy in 42% of cases, associated anatomic leadpoint


present in 85% of cases. Spontaneous (non-postoperative intusseception)
Intusseception is an indication for surgery in adults for evaluation of possibel
malignancy. Colocolonic intusseception should be resected without attempt as
reduction. Entero-enteric intusseception in adults should be reduced andevaluated for
the possiblity of malignancy.

# 63 - TE Fistula
Newborn becomes cyanotic every time it feeds,s alivates excessively and has
pneumonitis

Issues:
a.) What is the differential diagnosis?
b.) How would you confirm the diagnosis?
c.) What would contraindicate an immediate surgical correction?
d.) How would you repair this TE fistula?

ANSWER:

ESOPHAGEAL ATRESIA WITH DISTAL TRACHEOESOPHAGEAL FISTULA -


present in the vast majority of cases (85-90%).

Differential Diagnosis:
1.) Esophageal atresia with distal TEF.
2.) Esophageal atresia without TEF - recurrent aspiration with feeding.
3.) “H- Type” Fistula without esophageal atresia.

DIAGNOSIS: passage of nasogatric tube into stomach with CXR. Coiled NGT in
proximal pouch and air in stomach gives diagnosis. Can be confirmed with small
amount of Ba but this will increase the risk of aspiration of Ba.

148
=> important to find out preop the side of the aortic arch, which occurs on the right
in 5%. Usually this can be easily seen on CXR, if not then echocardiogram or CT
scan should be undertaken.

IMMEDIATE SURGERY - WATERSTON A CLASSIFICATION


- BIRTHWEIGHT OVER 5.5 LB. AND OTHERWISE WELL

DELAYED SURGERY - WATERSTON B+C CLASSIFICATION


-BIRTHWEIGHT 4-5.5 LB. AND WELL
OR
- BIRTHWEIGHT OVER 5.5 LB. BUT MODERATE PNEUMONITIS OR
SEVERE CONGENITAL ANOMALY

CONTRAINDICATION TO IMMEDIATE SURGERY = LOW BIRTH WEIGHT


(< 5 LB.), SEVERE PNEUMONIA, SEVERE ASSOCIATED CONGENITAL
ANOMALY (THINK VATER)

NO IMMEDIATE SURGERY -> place gastrostomy, establish proximal esophageal


pouch suction, start TPN. Allow pneumonia to resolve, gain weight.

SURGICAL APPROACH - posterolateral thoracotomy on the side opposite the aortic


arch. 4th interspace, retropleural apporach. Ligate the azygous vein ( if on the
right ) and the TEF will lie just beneath this. Have anesthesiologist place large tube
in esophagus to facilitate dissection of the proximal pouch. The distal TEF will
insert intothe trachea at the carina. Ligate the TEF with interrupted 5.0 silk leaving a
3 mm cuff of fistula to avoid narrowing the carina. The proximal pouch is
anastomosed to the distal esophagus in 1 or 2 layers using interrupted 5.0 silk.
- can extensively mobilize the upper pouch, but cannot mobilize the lower pouch
because of the segmental blood supply.
- 1 or 2 circular myotomies of the upper pouch can be performed for optimal length

LONG GAP ATRESIA - usually occur in patients with pure atresia and no TEF. If the
atresia is less than 2-3 vertebral bodies, it will be possible to perform anastomosis. if
not, place gastrostomy and stretch proximal pouch for 3-6 weeks. Can confirm the
shortening of the gap with radiographic studies.

#64 - Hirschprung’s Disease


4 day old male with abdominal distention , no vomiting, normal rectal exam.

Issues:
a.) How would you make the diagnosis?

149
b.) How would you intervene and confirm the diagnosis?
c.) How would you perform the rectal biopsy?

ANSWER:

PRESENTATION: generally these neonates will not pass meconium in first 24-48 hrs.,
though this will usually only be noticed in retrospect.
- will present with nonspecific signs of abdominal obstruction - bilious vomiting,
abdominal distention. May present early with symptoms of abdominal obstruction
(as in this case ) or later in childhood into early adulthood as chronic constipation.
- may present with symptoms of enterocolitis (fever, distention, bloody diarrhea) and
should be treated with IV fluids, antibiotics and decompression via colostomy
proximal to the transition zone. Though this presentation is unusual, the
possibility of this makes the need to pursue the diagnosis aggressively important.

PHYSICAL EXAM - usually normal. Occasionally, rectal exam will cause passsage of
stool.

DIAGN0SIS:

PLAIN RADIOGRAPHS - air-filled abdomen, cannot differentiate colon and small


bowel gas in a neonate. Will only indicate obstruction. Pneumatosis can be seen
with entercolitis or NEC.

BARIUM ENEMA - should follow plain radiographs when there is evidence distal bowel
obstruction. Do not perform any enemas prior to the BaE.
- insert plain catheter without balloon into rectum proximal to the internal sphincter and
the barium is slowly injected with syringe.

Diagnostic BaE - 1.) “Transition zone” - may not be present in early neonates because
the colon has not had time to dilate.
2.) Retained Barium after 24 hrs.
- must have histologic diagnosis prior to surgery.

BIOPSY - suction biopsy safe and can be performed at bedside. Suction biopsy should
be taken at least 1.5 - 3 cm proximal to the dentate line. Biopsy of mucosa and
submucosa is taken - risk of perforation is very small. This specimen should have
portion kept fresh for cholinesterase activity and portion taken for examination for
ganglion cell s in the intramural plexus.
- if diagnosis is not made on suction biopsy, then a full thickness biopsy should be taken
undeer general anesthesia. May need to do this in older children.

TREATMENT :

150
1.) COLOSTOMY - the colostomy should be done using a loop of normally
ganglionated colon. In order to properly locate the transition zone, mapping should
be done using serial frozen sections of seromuscular biopsies. May place colostomy
as either a sigmoid (above transition zone) or transverse colostomy. If a transverse
coloswtomy is placed, it is likely that a third procedure will be necessary to colose
this colostomy. A sigmoid colostomy caqn be taken down as part of a pull through
procedure as a definitive two-stage procedure.

2.) PULL THROUGH PROCDEDURE - at 9 - 12 months of age.


DUMAMEL - blunt retrorectal dissection is performed. Incision in the posterior wall of
the aganglionic rectum. Retrorectal pullthrough performed after resection of
aganglionic segment. The end of the ganglionated colon is anastomosed to the side of
the aganglionic rectum.
SOAVE - Resection of aganglionic segment and endorectal pull through with colorectal
anastomosis above the sphincter mechanism. This leaves a long rectal sleeve that the
ganglionated colon is pulled through.
SWENSON - the rectum is divided at the transition zone with stapler. Extramural
dissection is performed to free up the rectum. Rectum is pulled through the anus.
The rectum (outside the anus) is divided just above the sphincter and anastomosis is
performed. Colorectal anastomosis is performed.

#65 - Incarcerated Inguinal Hernia


8 yo male with incarcerated inguinal hernia.

Issues:
a.) How will you reduce this hernia?
b.) If the hernia is reduced, when would you operate?
c.) If Strangulated, what would you do?
d.) The bulge is red-hot, the patient has fever, and the intern reduces the hernia on the
way to the OR. What do you do?

ANSWER:
- must be distinguished from acute hydrocoele or inguinal lymphadenitis. If the
bulge has been present for more than 24 hrs without signs or radiographic
evidence of intestinal obstruction, it is unlikely an incarcerated hernia. Palpable
normal cord above the mass excludes an incarcerated hernia.

COMPLICATIONS - incarceration leading to strangulation and necrosis. Also ischemic


necrosis of the testicle from inpingement of hernia onto spermatic vessels. Because
of these complications, inguinal hernias should be repaired when diagnosed.
Period. No minimum weight or age is needed. Operate on during
hospitalization.

151
AGE < 50 WEEKS - hospitalize over night .
AGE > 50 WEEKS - same day surgery.

INCARCERATION: if there is no peritonitis, attempt may be made at reduction . The


patient is placed in bed with legs elevated and sedated. Gravity and sedation are
often enough to reduce. If not, pressure can be placed on the testicle. If reduced,
allow edema to resolve for 12 -24 hrs and then operate to repair.
DIFFICULT REDUCTION - ADMIT AND OBSERVE.
FAILED REDUCTION - OPERATE.

CONTRALATERAL SIDE - EXPLORE OPPOSITE SIDE IN PATIENT UNDER 2,


BECAUSE OF 60% CONTRALATERAL HERNIA RATE.

# 66 - Congenital Diaphragmatic Hernia


Know the different types and how to reduce. Associated with cardiac anomalies?

TYPES OF CONGENITAL DIAPHRAGMATIC HERNIAS:


1.) FORAMEN OF BOCHDALEK
2.) FORAMEN OF MORGAGNI.
3.) EVENTRATION OF THE DIAPHRAGM
4.) TRAUMATIC HERNIATION OF DIAPHRAGM - ACUTE OR CHRONIC.

FORAMEN OF BOCHDALEK HERNIA - posterolateral defect, more often on the


left side than the right. Herniation of abdominal contents into chest causes collapse
of ipsilateral lung and shift of the mediastinum to the contralateral side.
- can be diagnosed via chest X-ray = showing left hemithorax filled with air/fluid levels
of bowel with shift of mediastinum. If this is not diagnostic, upper GI series with
Barium will show small bowel in the chest. Bowel should be decompressed prior to
surgery with NGT.

ASSOCIATED ANOMALIES - VSD and Coarctation of the aorta are the most common,
but all have been reported - preop echocardiogram are routine. Almost all will have
PDA from the pulmonary hypertension and this should be alllowed to close on its
own after surgery.
- other associated anomalies are pulmonary sequestration , GERD, CNS
malformation.

SURGICAL TREATMENT - transverse abdominal incision, reduce with gentle traction.


Close with suture or prosthetic material for large defects. Closely examine the
reduced abdominal viscera for atresias. If the abdominal contents do not fit , a silo
can be created.

152
FORAMEN OF MORGAGNI HERNIA - defect anterior retrosternum ( paraster nal as
opposed to midline) where the superior epigastric artery transverses the diaphragm.
Plain films generally show air filled sac in the mediastinum. Reduction via a
transabdominal approach. These tend to present at later age.

ONCOLOGY

# 67 - Facial Melanoma
Elderly female has a 1 cm black lesion on her right cheek, and clearly palpable ipsilateral
neck node. Other physical examination and medical history are notpertinent.

Issues:
a.) What is the technique of the original biopsy?
b.) Pathology report comes back malignant melanoma = 1mm depth
= 1.3 mm depth
= 3.0 mm depth
= 3.4 mm depth
= 5 mm depth what is the
operative procedure of choice?
c.) Describe the operative procedure in detail including what would be removed and what
would be left. How large a margin would you take for the lesion?
d.) What do you do as far as adjuvant therapy when a single node is found to be positive?
e.) The patient is found months later to have mental status changes with obtundation. CT
scan shows a single met. What do you do? What if the patient comes back with a
single lung met. What do you do?
d.) Explain in detail Clark’s and Breslow’s level of melanoma. Which one is better?

ANSWER:

ORIGINAL BIOPSY -

EXCISIONAL BIOPSY - indicated for suspicious lesions that are not large (<1.5 cm),
located on areas where amount of skin does not matter (trunk). The orientation of
the biopsy long axis should be oriented toward the regional lymph node basin for re-
excision purpose (extremity melanoma should be oriented on the long axis).
- biopsy should have 2 mm margin of normal tissue and should include subcutaneous
tissues.

INCISIONAL BIOPSY - USE HERE - performed when the amount of the skin
removed is critical, such as the hands, feet or face. Can also be done when and
excisional biopsy will be a large undertaking. Can use scalpel or 6 mm punch biopsy
to include subcutaneous tissue. Can be done in office. No disadvantages (survival
or local recurrence) with this approach.

153
DEFINITIVE EXCISION OF MELANOMA PRIMARY - must widely excise the
biopsy site or tumor with margin of normal appearing skin. Since local recurrence
depends more on tumor depth than it does on amount of normal skin margin, the
amoun t of normal skin margin should be dictated by the melanoma depth.

IN SITU MELANOMA - it cannot metastasize, but it can locally recur. Excise with
0.5 - 1.0cm margin.

THIN MELANOMA - (<1 mm in depth) - no clear studies to indicate the minimally


effective margin, so no clear answer. 1.0 cm margin.

INTERMEDIATE THICKNESS AND THICK MELANOMA ( any equal to or greater


than 1 mm in depth) - studies show that smaller margins for thin melanomas are
alright but no clear minimal guidleline for melanoma greater than 1 mm in depth or
for any subgroup from 1- 4 mm. 2.0-3.0 cm margin.
- with excision or re-excision , orient the long axis of the incision toward the regional
lymph node basin (this allows melanoma cells in transit in lymphatics to be taken
with the primary specimen.) Margins should be the radius of the short axis of the
ellipse. The long axis should be at least twice the short axis , but more importantly it
should be long enough that when the ellipse is primarily closed there are no dog ears.

SPECIAL SITUATIONS:
FACE - CAN USUALLY ONLY GET 1 CM MARGIN ON THE FACE BECAUSE OF
VITAL CONTIGUOUS STRUCTURES. Must use judgement based on depth.
Use margin of twice the largest diameter of the lesion.

FINGERS AND TOES - Fingers - take at the middle interphalangeal joint.


- Thumb - take proximal to distal joint of thumb.
- Toe - amputate entire toe.

SOLE OF FOOT - avoid excision of weight bearing surfaces of t he plantar surfaces of


the foot.

EAR - initial wedge excsion of the helix with reexcision if + for malignancy.

THERAPEUTIC LYMPH NODE DISSECTION


- removal of positive lymph nodes is the only chance for control of local disease or cure.

HEAD AND NECK MELANOMA


- melanomas anterior to the pinna of the ear metastasize to the parotid, submandibular,
submental , upper jugular and posterior triangle lymph nodes.
- for head and neck melanoma with positive node clinically , a MRND should be
performed.

154
- melanoma arising from the scalp or face, anterior to the pinna of the ear, above the
commissure of the lip are at risk for mets to the parotid gland. Parotid nodes are
contiguous with cevical lymph nodes. Melanoma can metastasize to the superficial
parotid or to the nodes in the tail of the parotid gland, therefore, superficial
parotidectomy should be performed.
- for this melanoma of the cheek with a positive clinical node, perform a MRND and
a superficial parotidectomy.
Complication - pain, seroma or skin slough. Shoulder drop if you fuck up.

GROIN DISSECTION FOR LOWER EXTREMITY MELANOMA


- 2 lymph node basins, the superficial nodes located in the femoral triangle and the deep
inguinal, obturator and iliac nodes.
Palpable /suspicious groi n nodes - should have radical groin dissection obtaining
femoral and ilioinguinal nodes.
TECHNIQUE - curving incision from 4-5 cm anteromedial to the anterior superior iliac
spine over the middle of the inguinal ligmant to the apex of the femoral triangle
( extended trasnplant incision). Skin flaps are raised medially to the pubic tubercle
and the adductor longus and laterally to the sartorius
- dissection of deep n odes is performed by dividing the inguinal ligment to provide
better exposure and to allow removal of superficial and deep nodes in continuity. I f
these nodes appear normal, stop at the common iliac bifurcation. If these nodes are
abnormal continue to the common iliac nodes.
Non-palpable /no susupicious groin nodes - for what would be considered an ELND,
proceed with superficial node dissection of the femoral triangle nodes, if one of these
is found positive on frozen section, proceed with deep node dissection.
Complication - lymphedema - use early elevation and compression stocking.

AXILLARY LYMPH NODE DISSECTION:


- should perform a complete axillary lymph node dissection of all levels (I-III) ,
continuing dissection to include nodes beneath the pectoralis monor ( the pectoralis
minor may be taken with this dissection)
Complication - lymphedema.

POPLITEAL NODE DISSECTION:


- this area is usually not involved with tumor cells, so dissection of the popliteal space is
not indicated unless the lesion lies directly over this area.

DEMONSTRATED IN TRANSIT METS IN EXTREMITY - isolated limb perfusion


should be tried as it can offer dramatic results. This obviously is not offered for
melanoma of the trunk.

ADJUVANT THERAPY - no good demonstrated chemotherapy for melanoma. Some


evidence of benfit using alpha interferon or IL-2 stimulated LAK cells. Recommend
a clinical trial.

155
RADIATION THERAPY - use for bone and brain mets.

TREATMENT OF DISTANT METASTATIC MELANOMA


- tendency to metastasize to distant subcutaneous sites, GI tract (usually small bowel).
- resection is indicated for palliation of symptoms . Isolated brain mets should be
treated with a combination of surgery and radiation. GI tract melanoma should
be resected fro palliation of obstruction orbleeding.
- isolated lung met should be treated with resection. Nultiple lesions are not treated
with surgery.
- liver metastasis should NOT be treated with surgery because of short life
expectancy.
- bone = radiation.

CLARKS LEVEL - determines the level of invasion by the depth of invasion into the
layers of the skin.Tends to be subjective , requires multiple sections and a
committed pathologist.

BRESLOW’S LEVEL - PRESENT STANDARD. Thickness of tumor is determined


by the depth in millimeters. Less subjective . Better.

# 68 - Trunk Melanoma
45 y.o. male presents with pigmented lesion that was 4 cm superior and lateral to the
umbilivus that was about 1.5 cm in diameter. There is no axillary adenopathy.

Issues:
a.) How would you do the original biopsy?
b.) The biopsy comes back as a 1.5 mm in depth in melanoma. What would you do?
How can you tell which lymphatics drain this melanoma?
c.) At what levels would you do a lymph node dissection?

SENTINEL NODE BIOPSY


- preop lymphoscintigraphy is needed to determine if more than one regional lymph
node basins are draining a particular area.
- Isosulfan blue dye is injected intradermally (0.5-1.0 cc) on either side of the resection
scar. Wait 5-10 minutes. Incision is made over the regional lymph node basin (about
1/2 that used for complete lymph node dissection) and skin flaps are raised. The
lymphatic channel staining blue is found and traced toward the lymph node. The
lymph node staining blue is removed and sent for frozen section.
(+ )sentinel node = 37% chance that other nodes are positive, complete node dissection is
performed.

156
(-) sentinel node = 99% assurance that remaining nodes are negative. Procedure is
stopped at this time.
**importance of this technique is to get around the decision for ELND , the
indications for which are still controversial because subgroups of intermediate
thickness melanomas whcih would benefit from ELND have yet to be identified.

ELECTIVE LYMPH NODE DISSECTION: ONLY FOR INTERMEDIATE


THICKNESS (1-4.0 MM) MELANOMA.
- THIN MELANOMA (< 1.0 MM) = 95% cure rate and disease is localized. No
therapeutic benefit for ELND in these patients.
- INTERMEDIATE THICKNESS MELANOMA ( 1.0-4.0 mm) = 60% chance of
harboring occult lymph node metastasis but only a 20% chance of distant mets.
Potential benefit of ELND.
- THICK MELANOMA (>4.0 MM) = > 60% chance of occult lymph node metastasis
but > 70% chance of occult distant mets. These patients do poorly because of distant
mets negating the benefit of removing positive regional nodes.

PREOPERATIVE LYMPHOSCINTIGRAPHY - needed to define the regional


lymph node basin for head and neck and trunk.

# 69 - Seminoma/Testicular Mass
18 y.o. male presents with a painless mass that he notices in his left testicle.

Issues:
a.) Differential diagosis and preoperative work-up (U/S, AFP, CEA, beta-HCG, Ct scan)
b.) Operative approach to this lesion - should it be biopsied or excised, should it be
approached through the groin or the scro-bag?
c.) Approach if several 2-3 cm nodes are seen along para-aortic chain on CT scan and
pathology found to be seminoma? Similar scenario but path comes back as non-
seminomatous?
d.) Subsequent treatment of seminoma - adjuvant therapy?
e.) What if later a single lung lesion is found , bx (+) for seminoma? Bilateral lung
lesions?
What if later a single lung lesion is found, bx (+) for non-seminoma? Bilateral lung
lesions?

ANSWER:

Differential Diagnosis:
1.) Testicular torsion
2.) Epiddymitis
3.) Epididymal orchitis.

157
4.) Hydrocoele
5.) Hematoma
6.) Hernia
7.) Spermatocele

PREOP WORKUP
ULTRASOUND - used to tell if the mass is intratesticular or extratesticular. If the mass
is intratesticular , it is testiclular cancer until proven otherwise. Quick ,easy way to
narrow the differential.
LABS - AFP, beta - HCG, LDH
Seminoma - 10% will have elevated HCG.
Non-seminoma - 90% will have elevations of either or both of AFP or beta- HCG.
CXR
CT scans - chest, abdomen and pelvis.

DIAGNOSIS - RADICAL ORCHIECTOMY. ONCE TISSUE IS OBTAINED AT


ORCHIECTOMY, METASTATIC WORKUP IS COMPLETED AND FURTHER
MANAGMENT IS DECIDED.

RADICAL ORCHIECTOMY - approach thru and inguinal incision and the cord is
ligated high at the inguinal ring to eliminate the risk of scrotal or lymphatic seeding.

STAGING = SPECIMEN + CT SCAN + TUMOR MARKERS.

SPECIMEN = SEMINOMA OR NON- SEMINOMATOUS


T1 = localized to testicle.
T2 = extends beyond the tunica albuginea.
T3 = involving the epididymis
T4 = invading spermatic cord or scrotal wall.
- also need to know whether there is lymphatic or vascular invasion.

CT SCAN - MOST EFFECTIVE WAY OF DETERMINING RETROPERITONEAL


LYMPH NODE INVOLVEMENT. Can identify para-aortic LN of less than 2 cm in
size.
(+) CT Scan = likely to identify metastatic disease accurately.
(-) CT Scan = 25% chance of mets even with negative CT scan.

TUMOR MARKERS :
AFP - seen in a number of malignancies (liver, pancreas, lung, stomach) . Can be
produced by pure germ cell tumors (embryonal, teratoCA, yolk sac) or
combined tumors, but not seen in pure seminoma. If it looks like seminoma but
the AFP is elevated, it is considered nonseminomatous and should be treated as
such.

158
beta - HCG - can be produced by both seminoma and nonseminoma.

POST-OP TUMOR MARKERS - because marker half life is 5-7 days for AFP and 2-3
days for HCG, persistent elevations post-op indicate residual tumor. However,
normalization of tumor markers may mask the presence of residual tumor in the
retroperitoneum which is present 40% of the time.

STAGING FOR SEMINOMA AND NON-SEMINOMA


A CONFINED TO THE TESTICLE
B1-B3 SPREAD TO THE RETROPERITONEAL NODES
C METASTATIC DISEASE

TREATMENT

SEMINOMA-
LOCALIZED - STAGE I or A or Min. STAGE II or B - ORCHIECTOMY +
RADIATION - total of 2500-3000 Gy radiation should eradicate any tumor cells in
retroperitoneal nodes and give 5 yrs survival in 90-95% of cases. Radiation is
directed ipsilateral iliac, para-aortic and paracaval. Do not radiate the mediastinum.

ADVANCED - Advanced or Bulky STAGE II or B - STAGE III OR C -


ORCHIECTOMY + CHEMOTHERAPY - Cisplatin based combination
chemotherapy is best. This is defined as disease above the diaphragm, distant mets or
bulky abdominal disease. If there is response to this ( 60-100%), there is no need for
further therapy.

NONSEMINOMATOUS -
LOCALIZED - STAGE I OR MINIMAL STAGE II - ORCHIECTOMY +
RETROPERITONEAL LYMPH NODE DISSECTION - GOLD STANDARD FOR
TREATMENT AS WELL AS COMPLETE STAGING - Conventional methods will
understage Stage I disease 20-25% of the time. Alternatively, the patient may be
followed closely with serial tumor markers, anticipating a recurrence rate of 20% -
this is only for the very compliant patient. This treatment is for patients with disease
localized to the testicle/cord or those with retroperitoneal lymph nodes < 5 cm.

ADVANCED - Advanced or Bulky STAGE II or STAGE III - ORCHIECTOMY +


INDUCTION CHEMOTHERAPY - bleomycin, etoposide and cisplatin are used as
induction therapy for distant disease or bulky retropertioneal disease. Those showing
a complete response to chemotherapy may be observed. Those showing a partial
response may have the residual retroperitoneal disease resected via RPLND.

RPLND TECHNIQUE - can be done via a midline approach for transabdominal or via
a thoracoabdominal approach. The vena cava and aorta should be exposed and lymph

159
nodes removed from the ipsilateral iliac bifurcation to the SMA along the aorta and
from the caval bifurcation to the renal veins.

PULMONARY METS -
Criteria For Resection:
1.) Presence of nodules consistent with mets.
2.) Control of primary tumor.
3.) Potential for complete resection.
4.) Sufficient pulmonary reserve for resection
5.) Absence of uncontrolled pulmonary mets.

PULMONARY METS AND GERM CELL TUMOR - chemotherapy is the primary form
of treatment for both seminoma and non-seminomatous testicular CA mets. Surgical
resection is reserved for those with residual disease after initial therapy when tumor
markers have fallen to normal level
- seminoma is seen as increased mediastinal adenopathy , nonseminoma is seen as new
pulmonary nodules.
BILATERAL DISEASE - not a contraindication to resection if all of the above criteria
met.
Can be resected via staged bilateral thoracotomies or via a median sternotomy resecting
both sides at once.

# 70 - Hodgkin’s Disease
38 y.o. woman presents with an anterior cervical triangle neck mass - otherwise
asymptomatic.

Issues:
a.) Initial evaluation and workup of this lesion.
b.) How would you biopsy this lesion? What if FNA is not diagnostic?
c.) What are the characteristic cells of Hodgkin’s Disease? What do they look like?
d.) What are the 4 types of Hodgkins? Which is the most common?
e.) What is the workup of Hodgkin’s Disease? When is a staging lap indicated?
What tissues are sampled during a staging lap? What are indications for chemo/XRT?

Potential Trap - general anesthesia for biopsy of cervical node with possibility of
Hodgkin’s disease and mediastinal compression on the trachea.

HODGKIN’S DISEASE
- characterized by single or multiple painless lymph node enlargements. May be
associated with fever, night sweats, weight loss.

DIAGNOSIS:

160
-excluding benign primary thyroid tumors, 80% of neck masses in adults are
maignant, of which 80 % will be metastatic. 80% of metastatic neck masses arise
from above the clavicle.

INITIAL EVALUATION AND EXAMINATION:

EXAM - if the mass moves on swallowing, it is likely unde the strap muscles and is
likely a thyroid primary.
- use otoscope, tongue depressor, nasal speculum, glove and mirrors to examine the
oral cavity (glove and tongue depressor and otoscope), oropharynx , hypopharynx ,
larynx and nasopharynx (nasal spectulum and mirrors for indirect laryngoscopy).
- also examine groin and axilla (adenopathy associated with Hodgkin’s disease),
breasts (breast CA) , abdomen and rectal exam (GI tract CA), respiratory tract (lung
CA).

BIOPSY - FNA
Positive Results:
Metastatic thyroid - definitive surgery.
Lymphoma - remove entire node to confirm and establish cell type.
Branchial Cleft cyst - definitive surgery.

INCONCLUSIVE FNA ----> OPEN BIOPSY


- important to perform a complete excisional biopsy as the stromal architecture is
important in diagnosis. Present for formalin fixation.

Reed - Sternberg Cells - large, multinucleated cells. Diagnostic of Hodgkin’s Disease.

TYPES OF HODGKIN’S DISEASE:


1.) Nodular Sclerosis
2.) Lymphocyte Predominance
3.) Mixed Cellularity
4.) Lymphocyte Depletion

DIAGNOSTIC WORKUP OF HODKIN’S DISEASE:


1.) Histologic Diagnosis
2.) CT Scan of Chest and Abdomen
3.) Lymphangiogram - only if initial CT Scan of abdomen is negative.
4.) CBC with Differential and Platelet Count.

STAGING OF HODGKIN’S DISEASE:


I - involvement of single lymph node area.
II - 2 or more lymph node regions on the same side of the diaphragm or involvement of
single extralymphatic organ with lymph nodes on same side of diaphragm.

161
III - involvement of lymph node basins on both sides of the diaphragm or involement of
extralymphatic organ. (lymph nodes or spleen)
IV - disseminated or multifocal involvement of one or more extralymphatic organs (bone
marrow, lung, liver , skin, GI tract).

A - no constitutional symptoms.
B - wieght loss of 10% of body weight in 6 months, unexplained fever of 38 degrees,
night sweats.

ROLE OF STAGING LAPAROTOMY - should be used for all IA an IIA stages in


which RT will be used as exclusive form of therapy. If RT is considered (no
contraindications ) , then the extent of disease must be determined by staging lap.
25% - downstage (worse disease than anticipated)
15% - upstage (better than anticipated.)

STAGING LAPAROTOMY - splenic hilar, celiac, porta hepatis, mesenteric ,


peripancreatic, para-aortic, paracaval, iliac nodes. Spleen. Core and wedge biopsies
of both lobes of the liver. Iliac crest bone marrow biopsy. +/- oophoropexy.

CONTRAINDICATIONS TO STAGING LAPAROTOMY - if radiation is not a


treatment option, patients with B symptoms, bone marrow involvement, extranodal
involvement, mediastinal involvement, and factors making general anesthesia a risk.

Large Mediastinal Mass - should be treated with chemo and radiation, so


treatment will not change based on staging lap. So don’t do it. Beware of
general anesthesia in patient with large mediastinal mass.
Do biopsy under local.

NONHODGKIN’S LYMPHOMA - most common type is diffuse histocytic lymphoma


(DHL), and is treated with chemotherapy. Do not do staging lap.

# 71 - Sarcoma
55 y.o. male presents witha mass on his anterior thigh. On physical examination it is
poorly defined but approximately 5 cm in diameter.

Issues:
a.) What further evaluation is required? (Radiologic evaluation)
b.) What type of biopsy is required and how is it performed? Would you get frozen
section? What is the grade?
c.) How does one perform definitive dissection assuming that this proves to be a
sarcoma?
d.) What type of margins, etc?

162
e.) Role of postoperative radiation therapy? Role of adjuvant chemotherapy? What is the
recurrence rate if clear margins are obtained and postop XRT used?

ANSWER:

STAGING = BASED ON HISTOLOGY AND CLINICAL (SIZE, NODAL STATUS


AND METS) INFO.

G1 = WELL DIFFERENTIATED
G2 = MODERATELY DIFFERENTIATED.
G3 = POORLY DIFFERENTIATED.

STAGE I = any well differentiated tumor, no nodes, no mets.


(A= < 5cm, B= > 5 cm)
STAGE II = any moderately diffrentiated tumor, no nodes, no mets
( A=< 5cm, B = > 5 cm)
STAGE III = any poorly differentiated tumor, no nodes, no mets.
( A= > 5cm, B = > 5 cm)
STAGE IV = any tumor with either nodes and/or mets.
(A= nodes without mets, B= mets with or without nodes)

RADIOGRAPHIC EVALUATION:

CT SCAN - OF MASS AND CHEST - for low grade lesions, the chance of chest mets
is only 15%, so chest X-ray is all that is necessary. For high grade lesions, chest CT
is necessary.
- for retroperitoneal sarcomas, CT scan of the abdomen should be obtained to rule
out metastatic disease of the liver , the most lilely site of mets in this case.

MRI - can be used in place of CT scan to evaluate the primary lesion to visualize the
relationship of the lesion to bone, muscle and vascular structures . May also use both
as the studies complement eahc other.

Nodal Mets - poorly assessed by CT or MRI, but not important because the chance f
nodal mets in sarcoma is low(4%). The prognostic significance of nodal met is the
same as a met in the case of sarcoma (both Stage IV).

BIOPSY - incisional biopsy is performed in almost all cases- the exception is small
tumors (<3-5 cm.) which have low risk of malignancy and in which excisional
biopsy can be perofrmed without burning bridges for subsequent treatment in case
sarcoma is diagnosed.

163
TECHNICAL POINTS - for extremity sarcoma, the incision should be placed along the
long axis of the extremity. For retroperitoneal sarcoma, the incision should be
placed in such a way that it can be easily excised with resection if sarcoma is
diagnosed. The biopsy of the mass should be placed over the portion of the mass that
is closest to the surface. Do not disturb tumor planes around tumor, minimize
dissection.
- sarcoma are often surrounded by a pseudocapsule which is composed of
compressed tissue and tumor. Do not enucleate from pseudocapsule as you will
leave tumor cells behind.
- send frozen section, not for definitive diagnosis, but to get confirmation of
adequate tissue and malignancy and to make certain that you have not biopsied
the pseudocapsule.
- careful hemostasis to prevent hematoma, as tumor cells will disseminate in
hematoma. Do not use drains.
- FNA will not yield appropriate tissue for tissue type and histologic grade and should
not be used.
- core needle samples are acceptable if several are provided.
- send tissue fresh, get a frozen.

SURGERY - KEY FOR LOWER EXTREMITY SARCOMA IS WIDE LOCAL


RESECTION FOR LIMB SPARING PROCEDURE.
- tumor should not be entered during dissection. If the tumor was small and removed by
excisional biopsy (<3-5 cm) and was found to be a malignant sarcoma, they should
be operated on for a wide local resection.
- ideal margin is 3-5 cm in all directions. When this is not possible, at least one
uninvolved fascial plane should be taken.
? Nervous/Vascular Involvement - should be sacrificed for high grade/poorly
differentiated sarcoma, but dissected free of tumor for low grade lesions. I f
these structures are taken , the vascular structures can be reconstructed. Resection of
nerves can leave a compromised but functional extremity.

TUMOR < 10 CM, RESECTABLE ON PREOP STUDIES --> RESECT + RADIATION


TUMOR < 10 CM, FIXED SARCOMA, NONRESECTABLE --> RADIATION +
CHEMOTHERAPY PREOP FOR LIMB SPARING TREATMENT.
TUMOR INVOLVING BONE OR RECURRING AFTER LIMB SPARING
->AMPUTATE

Negative margins are necessary.

POSTOPERATIVE RADIATION AFTER WIDE LOCAL RESECTION FOR LOWER


EXTREMITY SARCOMA.
1.) Low grade sarcoma , small with desirable clean, 3-5 cm margin -> can observe
without radiation.

164
2.) Low grade sarcoma with either clean but close or questionable magins because of
proximity to bone, vascular structure or nervous tissue --> RADIATION.
3.) Any intermediate or high grade lesion, regardless of margin ---> RADIATION.
Nota Bene - radiation lowers local recurrence but does not imporve survival.

POSTOPERATIVE CHEMOTHERAPY AFTER WIDE LOCAL RESECTION FOR


LOWER EXTREMITY SARCOMA. - since 40% of patients who die of systemic ,
hematogenous spread, adjuvant chemotherapy makes sense. However, current
combinations only slightly improve disease free interval but do not improve survival.
Routine adjuvant chemo is not recommended, but should enroll in clinical trial.

FOLLOW UP - annual CT scan of primary site and chest for first 5 year for high grade
lesions. Low grade lesions should have CT scans of local site.

LOCAL RECURRENCE - RECURRENCE RATE FOR EXTREMITY SARCOMA


AFTER RESECTION = 15% - for small tumors that were treated with local
resection and no radiation, resection and multimodality therapy should be offered.
For those previously treated with resection and radiation, radical amputation should
be performed at one joint level above lesion.

METASTATIC DISEASE - patients with multiple lesions are still candiate for resection,
though those with 3 or less do best. If these lesions are seen before resection of
primary , obtain tissue of lung lesion for diagnosis, then plan a staged procedure for
resection of primary followed by lung lesions.
- if lesions are found on follow-up, resect if possible. Bilateral lesions should be
treated with staged bilateral thoracotomy or median sternotomy.

SPECIAL SITUATIONS:
Distal Extremity, involving neurovascular structure, bone, joint - likely amputate.
Proximity to Pelvis - consider hemipelvectomy .
Popliteal or antecubital fossa tumor - amputate.
Hand - some for of local amputation is recommended.
Retroperitoneal Sarcoma - most are very large at presentation (10-20 cm in diameter)
and intermediate to poorly differentiated. 79% involve other organs and will need to
be resected with specimen. Postop radiotherapy is recommended for these lesions,
but has not shown the success of radiation in extremity sarcoma because of difficulty
in dirrecting high doses to the right area along with poorer tissue tolerances. Radiate
anyway.

# 72 - Incidental Ovarian Mass during Upper Abdo Surgery


While doing a cholecystectmy you palpate an ovarian mass.

Issues:
a.) How would you evaluate the mass.

165
b.) At what size would you remove themass?

ANSWER:

CYSTIC MASSES:
- may be benign (functional cysts) or malignant (mixed cyst)
- in general, when an ovarian mass is present, whether cystic or solid, it is removed
regardless of size because the only way to exclude cancer is to patholoically
examine the entire mass. The exceptions to this rule are (1) the functional
ovarian cyst ( which presents in women of reproductive age.) and small (<
5cm) asymptomatic , unilateral cysts
in postmenopausal women.

Functional Cysts - most are corpus luteal cysts. These cysts are unilateral, unilocular
and mobile. These cysts will usually resolve over 3-4 menstrual cycles, and can be
induced to resolve by treating with oral birth control pills if desired. Cysts larger than
10 cm are less likely to resolve spontaneously.
- other functional ovarian masses are theca lutein cysts, pregnancy luteoma, polycystic
ovaries.
-see Greenfield, p. 2014, Fig. 116-6.

NON-CYSTIC OVARIAN TUMORS:


- may be benign (serous cystadenoma, mucinous cystadenoma, cystadenofibroma,
Brenner tumor) , malignant serous cystadenocarcinoma, mucinous
cystadenocarcinoma, clear cell CA, endometroid CA, undifferentiated tumor.

Ovarian Malignancy - 80-90% of epithelial origin. Usually present between 50 and 60


yrs of age. 5 yr.survival ranges from 85% for disease localized to the ovary to 15%
for advanced disease. Most present with disease outside of the ovary , so early
detection and treatment could have impact on survival.
- tendency for malignant cells to exfoliate, and are transported via peritoneal fluid flow in
clockwise fashion , affecting the right side more often than left.

SURGICAL PRINCIPLES: REMOVAL OF MASS (FOR COMPLETE


RESECTION OR DEBULKING) AND COMPLETE STAGING.
1.) Removal of Tumor - complete removal is essential , even if only for debulking.
Resect any colon or small bowel associated with tumor. Classical debulking
resection for biopsy proven cancer includes removal of the uterus, cervix, ovaries,
fallopian tubes.
Problem - Woman of Child Bearing Age -
i.) is the tumor malignant or benign? - do not perform a complete debulking
operation on a woman of child bearing age based on frozen section. Remove tube
and ovary , biopsy contralateral ovary and complete staging. If benign, done. If

166
malignant, all staging done and decision can be made whether pt. is candidate for
conservative therapy.
ii.) can conservative resection be performed? - if tumor is unilateral, confined to
ovary, fully encapsulated , favorable histology, then a unilateral
salpingooophorectomy, contralateral ovary wedge biopsy, omental biopsy, liver
capsule and right hemidiaphragm. If contralateral ovary biopsy is positive, then a
second operation to remove other ovary and uterus must be done.
2.) Staging - biopsy right paracolic gutter, liver capsule, right hemidiaphragm (tendency
for peritoneal spread on right because of clockwise rotation of peritoneal fluid), the
omentum, paraaortic nodes and any other suspicious areas. Ascites should be
collected and sent for cytology. If no ascites is present, irrigate peritoneum with 50
cc of saline along peritoneum, liver and right diaphragm and send for cytology.

- tumor must be aseessed , resected and staged. Close this incision and make a
midline. If the patient is being explored for appendicitis, extend this for exploration.

Other situations:
1.) Explore and find abdomen caked with tumor - patient will need debulking operation
which may include resection of small bowel and colon. Make sure the patient has had
a bowel prep.

# 73 - Evaluation of Mass/Cold Nodule


40y.o. female with cold nodule in the right superior thyroid. PMHx and family history
are all negative. Physical exam is negative except for the lesion.

Issues:
a.) Preop labs, imaging and possible FNA of this lesion.
b.) FNA comes back as follicular cells with suspicion for malignancy. What is you
operative procedure? Frozen section shows follicular CA.
c.) Definitive procedure for follicular CA? For papillary CA?
d.) Describe in detail the surgical technique of thyroidectomy.
e.) What do you do if there are also a few palpable nodes?
f.) You are called to the bedside and the patient is obtunded (Ca is 6). The patient is
cyanotic with a massive hematoma. What do you do?
g.) What are the possible complications of total thyroidectomy?
h.) How would you evaluate for mets postop?

ANSWER:

History - although negative - shoulde focus primarily on history of radiation exposure


and family history of thyroid cancer or MEN syndromes (II).
Exam - negative - but are concerned primarily with ythyroid nodule and lymph nodes.
Labs: TFT’s (most euthyroid), calcitonin level.

167
FNA - TECHNIQUE - patient in the supine position. Nodule site is prepped with
alcohol. 27 gauge needle is placed in the lesion and plunger pulled back. With
plunger pulled back. Place on slide and fix. Cystic lesions should be aspirated.
Large lesions suspicious for anaplastic CA or lymphoma should have larger
specimens taken via a TruCut biopsy.
Results - suggestive of malignancy, suspicious for malignancy, benign or inadequate
specimen. FNA is extremely accurate for diagnosing medullary or papillary CA, but
is difficult to diagnose follicular CA because of criteria for angioinvasion which
cannot be diagnosed .

TREATMENT -
1.) Neck exploration.
2.) Exploration and evaluation of both thyroid lobes.
3.) Lobectomy on affected side and isthmusectomy.
4.) Frozen section on lesion.
5.) Benign ----> close.
6.) Malignant ...

TREATMENT OF FOLLICULAR CA OF THYROID - the pathologic criteria for


distinguishing malignant from benign is the presence of capsular or angioinvasion of
the follicular cells (these features cannot be seen on FNA, only the follicular cells can
be seen.)

Hurthle Cell CA - Hurthle cells are “oncocytes” , cells that are derived from follicular
cells, that appear as large bizarre, polygonal shaped cells with hyperchromatic nuclei.
Hurthle cells may be present in Hurthle cell adenomas or in Hurthle cell carcinoma.
Hurthel cell Carcinoma is differentiated from a benign adenoma by capsular or
angioinvasion of Hurthle cells (as differentiating benign from malignant follicular cell
tumor.)

- these tumors(both Hurthle cell and follicular CA) tend to have distant mets via
hematogenous spread to lung , bones and brain rather than to the lymph
nodesvia lymphatic spread.

OPTIMAL SURGICAL TREATMENT - TOTAL THYROIDECTOMY.


- at minmum, a “near total thyroidectomy” should be performed.

POST OPERATIVE CARE - total or “near total thyroidectomy” decreases the “sink”
effect of normal thyroid uptake of I-131. Postoperative I-131 scanning should be
performed for detection of metastasis. If mets are detected, therapeutic I-131 can be
used. I-131 treatment does not work as well for Hurthle cells because of their poor
ability to take up iodine, but should be used anyway.

168
TREATMENT OF PAPILLARY CA OF THE THYROID.
- RECURRENCES FOR PAPILLARY CANCER TEND TO OCCUR AT THE
EXTREMES OF AGE, 40% under 20 yrs. and 30% over 59 yrs.

MIXED PAPILLARY - FOLLICULAR CARCINOMA = should be treated as a pure


papillary carcinoma.

PAPILLARY CA - CLASSIFICATION
1.) MINIMAL OR “OCCULT” - measure less than 1.5 cm, are usually discovered either
incidentally during thyroid resection for other benign condition or after diagnosis of
lymph node met as a “lateral aberrant thyroid rest”. Extremely rare to die of a
minimal papillary thyroid cancer.
2.) INTRATHYROID - measure greater than 1.5 cm but are still isolated to the thyroid
galnd and show no extension outside of the thyroid.
3.) EXTRATHYROIDAL - invade the thyroid capsule and grow into contiguous
structures.
** multicentricity is common in papillary cancer. Cervical mets are also common in
papillary cancer, seen in up to 40%. Extranodal mets are seen in 10%.

LOBECTOMY + ISTHMUS - proponents state that the survival results are the same as
a total lobectomy while the complication rates are lower in the case of minimal
thyroid cancer ( small and without extension outside of the capsule) . A papillary
cancer in the contraleateral lobe diagnosed later can easily be removed.
- the need to remove the remaining thyroid for the purposes of using diagnostic or
therapeutic I-131 is unnecessary since the course of this cancer is so benign that I-
131 is really not needed.

TOTAL LOBECTOMY - effective at eliminating multicentric bilateral tumor foci and


preventing contralateral recurrence. Allow I-131 to be used for both diagnostic and
therapeutic reasons. Also allows the use of thyroglobulin as a marker for recurrence.
Should be done by experienced thyroid surgeon to decrease incidence of
complications.

RECOMMENDATIONS FOR TREATING PAPILLARY THYROID CA:

1.) INCIDENTAL MINIMAL THYROID CA - subtotal thyroidectomy . Should be


placed on TSH thyroid hormone suppression and followed closely.
2.) OCCULT MINIMAL PAPILLARY CA WITH LATERAL ABERRANT
THYROID REST - should undergo total or near total thyroidectomy with lymph
node dissection so that the patient can be treated with radioactive I-131 therapy.
3.) INTRATHYROID (> 1.5 CM) OR EXTRATHYROID PAPILLARY CA -
should undergo total or subtotal throidectomy.

169
LYMPH NODE METASTASIS - first mets go to peritracheal nodes and enbloc
dissection of these nodes should be taken with the initial operation. Modified
radical necdk dissection should be performed.

POST OPERATIVE CARE - serum calcium and phosphorus drawn at 5 hrs and the
following morning. If papillary Ca is confirmed by path, do not give thyroxine
postop, hold until the patient is slightly hypothyroid and the TSH rises to 35. Perform
I-131 scan at about 2 weeks. If mets are seen or there is residual thyroid tissue left
over, proceed with I-131 ablation. The patient is then treated with TSH supressive
doses of thyroxine.
- follow every 6 months for first 2 years.
- if the thyroglobulin level rises, thyroxine is held to allow TSH to rise so that I-131
can be used for ablation.

LOBECTOMY AND SUBTOTAL THYROIDECTOMY - TECHNIQUE - supine


position, roll between shoulders, occiput padded. Tranverse 1.5-2.0 cm incision 2
finger breadths abov e the sternal notch. Divide skin, subQ fat and platysma.
Superficial veins are ligated. Superior and inferior skin flaps are raised superficial to
the level of the strap muscles. Linea alba divided between the strap muscles . The
anteriro/superficial surface of the thyroid is dissected off of the posterior strap
muscles using blunt dissection. This dissection is carried laterally to separate the
lateral thyroid from the carotid sheath. Middle thyroid vein is found and ligated. The
thyroid lobe is rotated medially exposing the superior and inferior pole vessels.
- grasp the thyroid just below the vessels with a Kocher clamp and retract inferiorly.
Identify the superior laryngeal nerve if possible and ligate the superior pole vessels
flush with the thyroid gland using 4-0 silk. (This technique will lower the risk of
injury to the superior laryngeal nerve. The galnd can now be retracted even further
medially and and the recurrent laryngeal nerve can be found in the tracheoesophageal
grove beneaththe inferior thyroid artery . Inferior thyroide vein branches are taken.
The nerve is identified and separated from the thyroid gland. Once the nerve is freed,
the small vessels can be taken. Bleeding at this point is controlled with thrombin and
gelfoam and pressure. Caution is taken to avoid damaging blood supply to the
parathyroids and minimizing over dissection of the parathyroids. If the parathyroid is
identified and not part of ythe diseased thyroid, it should be left as is. If the
parthyroid is devascularized, it may be diced into 2-3 mm pieces and placed in the
forearm or pocket of sternocleidomastoid. Mark with vascular clip. The thyroid
attached to the pretracheal fascia is divided with cautery. Send the lobe for frozen.

SUBTOTAL THYROIDECTOMY - lobectomy on one side and mobilization of the


thyroid from the trachea on the opposite side. The course of the recurrent lartyngeal
nerve is identified alon g with the parathyroid glands. Place hemostats on mobilized
thyroid to leave 1 cm rim of tissue . Avoid parathyroids and recurrent laryngeal
nerve. leave the inferior thyroid artery to lower the risk to the recurrent laryngeal n.

170
TOTAL THYROIDECTOMY - repeat steps for thyroid lobectomy on oppositie side.

COMPLICATIONS OF TOTAL THYROIDECTOMY -


1.) DAMAGE TO RECURRENT LARYNGEAL N. - it is possible to damage a non-
recurrent laryngeal nerve, which can only occur on right.
2.) DAMAGE TO PARATHYROIDS - should check phosphorus level 5-8 hrs. postop, if
low there is little concern. If the level is rising (> 4.5) , there is concern about
permanant hypocalcemia. Have Ca gluconate at bedside.
3.) DAMAGE TO SUPERIOR LARYNGEAL NERVE.
4.) HEMORRHAGE - with loss of airway. Evacuate at bedside if necessary.

# 74 - MCT
Issues:

a.) How would you work up a patientt with a suspected MCT prior to resection?What is
your surgical resection of choice? How would you evaluate the family members?

ANSWER:

MCT - arise g=from C cells or parafollicular cells which produce calcitonin. In their
more aggressive form, they show evidence of lymphatic or vascular invasion early ,
metastasizing to perithyroid and paratracheal nodes to nodes of the jugular chain and
upper mediastinal nodes.

Familial Syndromes - 75% of cases of MCT are sporadic and 25% are associated with
either MEN Iia, MEM Iib or falilial non-MEN MCT.

- those with MEN- 2A can also develop parathyroid hyperplasia and pheochromocytoma.
While MEN 2B can often be diagnosed by multiple mucosal neuromas, the outward
phenotype of MEN -2A cannot be detected. The only way MEN-2A can be detected
is by the presence of endocrine neoplasia.

- diagnosis of MEN-2A and MCT can be made by screening for calcitonin levels.
Pheochromocytomasmay be detected by measurement of urinary catecholamines
and metanephrines . Hyper parathyroidism is detected by measuring serum
calcium levels

- typically those with palpable disease will typically present with nodal mets.

- nonfamilial cases are usually unifocal and unilateral, MCT associated with
familial syndromes are usually multicentric and bilateral.

INITIAL DIAGNOSIS - palpable mass - FNA cytology can be done but thyroid
lobectomy with histologic diagnosis. All patients suspected of MCT should have

171
CEA levels and stimulated calcitonin levels for screening. Serum calcium
should be checked to rule out parathyroid hyperplasia. 24 hr. urine collection
should be done for VMA, catecholamines and metanephrines. Even those with
“sporadic MCT “ should have screening.

STIMULATED CALCITONIN - infuse Calcium= 2mg/kg/min. followed immediately


by Pentagastrin 0.5 microgram /kg/5 sec. with blood draws at 1,2,3, and 5 minutes.

SURGICAL TREATMENT - TOTAL THYROIDECTOMY AND CENTRAL NODE


DISSECTION
1.) TOTAL THYROIDECTOMY

2.) CENTAL NODE DISSECTION - from the hyoid bone superiorly to the
innominate vessels inferiorly. Laterally, the nodes and thyroid tissues from the
carotid sheath to the carotid sheath. Those with large masses in the thyroid (> 2cm )
or palpable paratracheal or jugular nodes should have a modified radical neck
d9issection on that side. Generally when MCT is resected alsong with central nodes,
the parathyroids must be taken to clear the specimen. these parathyroids should be
minced and autotransplanted into the forearm.

RECURRENCE/PERSISTENT ELEVATIONS OF CALCITONIN - many have


persistent elevations of calcitonin after adequate resection with presence of nodal
mets. Recommend observation in absence of clinical disease.

?RADIATION - I-131 is not effective because C cells are not derived from follicular
cells and will not concentrate iodine. External beam radiation is not beneficial.

?CHEMOTHERAPY - mixed results but not shown to be effective.

SCREENING FOR MCT - CALCITONIN LEVELS

# 75 - Insulinoma
14 y.o. female with behavioral changes over the past several months, manifested by
psychotic episode s and blackouts.

Issues:

a.) What is the differential diagnosis?

b.) How would you work up this lesion to confirm the diagnosis?

172
c.) Assuming this is an insulinoma, what is the chance that this is chance that this is
malignant? an adenoma? diffuse benign islet cell hyperplasia/nesidioblastosis?

d.) What are the localization modalities and when are they indicated? What do you do if
you are not able to localize the lesion preoperatively? What are intraoperative
methods of tumor localization?What if no tumor can be localized? How can you
confirm diagnosis of hyperplasia? What do you do if the patient has hyperplasia and
has responded to diazoxide? What do you jdo if the patient has not responded to
diazoxide and has hyperplasia?

ANSWER:

DIFFERENTIAL DIAGNOSIS:

Factitious Insulin Administration - usually occurs in typically whacked out chick.


Hepatic Insufficiency
Mesothelioma
Functional Hypoglycemia
Insulinoma
Nesidioblastosis - hyperplasia of the Islet cells in pancreas, typically in
infants.

DIAGNOSIS: based on symptomatic hypoglycemia and hyperinsulinism.

SUPERVISED FAST AND SERIAL SERUM GLUCOSE AND INSULIN LEVELS -


patient placed in hopital with heplock in place. Fast for up to 72 hrs. Draw blood for
insulin and glucose measurements before fast starts and evrey 6 hrs. When
neuroglycopenia (symptoms suggesting hypoglycemia) develops, glucose, insulin, C-
peptide and proinsulin levels are drawn. All patients will develop symptoms of
hypoglycemia in 72 hrs., those without insulinoma will not.

RESULTS:
GLUCOSE - all will have glucose levels 45-50.

INSULIN - all will have elevated levels for degree of hypoglycemia (>5)

C-Peptide - elevated level indicates that the tumor is producing insulin and it is
not being given from surreptitious source.

Proinsulin - insulin precursor - produced in excess by insulinoma.

?Borderline levels = can use provocative agens such as Calcium, tolbutamide and
glucagon to stimulate insulin leves, but theset tests are not used routinely for
diagnosis.

173
CONTROLLING HYPOGLYCEMIA - ability to control the hypoglycemia dictates the
importance of removing the tumor with the first opertation. I f the tumor has not
been localized well prior to surgery,but the symptoms are well controlled with diet
and medications, a less aggressive operation may be performed. however, if the
tumor has not been localized well and the symptoms are not under control, the need to
remove the tumor with the first operation is critical and a more aggressive approach
must be taken with the first operation.

DIAZOXIDE - can be used to control the insulinoma syndrome, by dilating resistance


vessels of the islets and decreasing insulin secretion. Must be stopped 1 week
before surgery because of the risk of severe hypotension intraop

DIET - should be used as primary form of treatment for symptoms. larger and more
frequent meals.

PATHOLOGY:

MALIGNANT - 10%

BENIGN - 90% (10% associated with MEN I and will have multiple tumors and
hyperplasia, 80 (+) % associated with adenomas)

PREOPERATIVE RADIOGRAPHIC LOCALIZATION:

PROBLEM - tumors are small,like gastriinomas , and difficult to localize. Even with
invasive imaging techniques, localization of tumors is only about 50%. However,
unlike gastrinomas ( in which most of the tumors can be localized within the
gastrinoma triangle) insulinomas are uniformly distributed throughout the head,
body and tail.

-> even if tumor cannot be localized, it is important to try to isolate the lesion to one
portion of the gland so that surgical excision can be performed to remove the
tumor without have to do a “blind” total or subtotal pancreatectomy.

IMAGING MODALITIES

1.) CT, US, MRI - because tumors are so small (< 2cm), it is unlikely that any of these
modalities will be able to localize the tumor. However, one of these should be doneto
make sure there is not a large tumor or numerous mets.

2.) SELECTIVE CELIAC ANGIO - sensitivity was thought to be high, but studies
show it is only about 50%.

174
3.) PORTAL VENOUS SAMPLING - for insulin - it does not localize the tumor , it
does suggest which part of the pancreas contains the tumor. Most sensitive modality
(82%).

4.) CALCIUM ANGIOGRAPHY - variation of secretin angio used for gastrinomas


but uses Ca as the provocative agent. In this modality the calcium is injected into
each selectively catheterized artery and plasma samples are collected from the hepatic
veins for determination of insulin concentration. LIMITED DATA, BUT BEST
METHOD FOR DIAGNOSING INSULINOMA AND LOCALIZATION OF
PORTION OF PANCREAS.

5.) ENDOSCOPIC ULTRASOUND - using a high frequncy transducer, tumors as small


as one cm can be localized.

SEQUENCE OF TESTS:

CT SCAN WITH IV CONTRAST - INVASIVE


LOCALIZATION STUDIES - USE EITHER PROTAL VENOUS SAMPLING OR
CALCIUM ARTERIOGRAPHY.(provide correct localization in 80-100% of
patients.)

PREOP PREPATION - give pneumovax, bowel prep and periop antibiotics

EXPLORATION FOR INSULINOMA - enter abdomen thru a bilateral subcostal


incision and place self retaining retractors. Don’t expect to find tumor outside the
pancreas be cause of the low risk of malignancy. Mobilize the splenic and hepatic
flexures of the colon. Divide the gastrocolic ligament to expose the body and tail of
pancreas. Wide , complete Kocher maneuver to examine the posterior head of the
pancreas. Mobilize the entire inferior border of the pancreas to be able to palpate the
body and tail of the gland between the thumb and forefinger. May need to take short
gastric vessels and mobilize the spleen to adequately assess the tail of the pancreas.

POTENTIAL PROBLEM - if the tumor is not localized at this point, excessive biopsy of
the pancreas is likely to lead to bleeding, fistulas and abscesses.

VISUALIZATION - tumors are reddish brown.

PALPATION - generally sensitive method, but can easily miss tumors in the head
because of the thickness of the gland here.

INTRAOP ULTRASOUND - can not only localize tumor but also give anatomic detail
of the relationship of thetumor to the SMV, portal vein, etc.

RESECTION/REMOVAL - if possible (as in the likely case of the benign tumor) the
mass is enucleated and sent to path for frozen identification of an islet cell tumor.

175
LOCALIZED ADENOMA - ENUCLEATE AND CLOSE
LOCALIZED ADENOMA WITH MEN - SUBTOTAL PANCREATECTOMY -
because of the high rate of multiple tumors.

LOCALIZED CARCINOMA - SUBTOTAL PANCREATECTOMY (FOR TUMORS


IN THE BODY OR TAIL) OR WHIPPLE (FOR TUMORS OF THE HEAD)

LOCALIZED CARCINOMA WITH METASTASIS - RESECT FOR PALLIATION.


UTILIZE MEDICAL THERAPY (DIAZOXIDE)

NONLOCALIZED WITH GOOD CONTROL OF HYPOGLYEMIA - may opt for


following and allowing the tumor to grow and plan for resection at second operation.
May biopsy the pancreas to rule out the rare conditions of beta cell hyperplasia or
adult nesidioblastosis. Blind subtotal pancreatectomy has only 50% chance of
removing the tumor.

NONLOCALIZED WITH POOR CONTROL OF HYPOGLYCEMIA -


SUBTOTAL PANCREATECTOMY.

BETA CELL HYPERPLASIA/NESIDIOBLASTOSIS - SUBTOTAL


PANCREATECTOMY.

# 76 - Aldosteronoma
38 y.o. female with h/o SVT and negative PMHx. BP = 200/110 and found on routine
blood work to have a potassium of 2.0.

Issues:

a.) Assuming that the patient has hyperaldosteronism, how can you rule out secondary
causes?

b.) What tests will confirm the diagnosis of aldosteronoma?

c.) Assuming the patient has an aldosteronoma, how will you differentiate hypeplasia
from an adenoma? How will you localize the tumor? What are treatment options?

ANSWER:

PRIMARY HYPERALDOSTERONISM - results from excess secretion of aldosterone


frm the zona glomerulosa of the adrenal cortex. Caused by an adrenocortical
adenoma (60%) or from adrenocortical hyperplasia (40%).

176
SECONDARY HYPERLDOSTERONISM - cirrhosis, pituitary adenoma, renal artery
stenosis, etc.

-important to differentiate primary from secondary causes of hyperaldosteronism


and adrenal cortical hyperplasia from adenoma because adenoma is amenable
to surgical resection and hyperplasia is not treated with surgery.

DIAGNOSIS OF HYPERALDOSTERONISM - Most effective test is to see


nonsuppressibility of aldosterone secretion during a period of salt loading.

TECHNIQUE - infuse 2000 cc of Normal Saline over 4 hrs. If the plasma aldosterone
level is not suppressed to less than 10ng%, it is likely primary hyperaldosteronism.
The presence of hypokalemia and suppressed renin activity add further support to
this diagnosis.

CAPTOPRIL CHALLENGE - captopril will replace NaCl infusion as a means of


lowering the renin level and ultimately the aldosterone level by inhibiting the
angiotensin converting enzyme.

DIFFERENTIATING ADENOMA FROM HYPERPLASIA - can differentiate


adenoma from hyperplasia by postural changes.

UPRIGHT POSTURE > 4HRS -> nochange , decreased aldosterone = ADENOMA


UPRIGHT POSTURE > 4 HRS-> increased aldosterone =HYPERPLASIA

CT SCAN - can reliably localize adenomas > 1.5 cm.

ADRENAL VEIN RENIN SAMPLING - 100% EFFECTIVE at differentiating


adenoma from hyperplasia and also localizes side. This method is invasive and
should be used only in the event that preop tests suggest an adenoma but the CT
scaan fails to localize adenoma to one side.

TREATMENT OF PRIMARY HYPERALDOSTERONISM - UNILATERAL


ADRENALECTOMY FOR ADENOMA AND SPIRONOLACTONE FOR
HYPERPLASIA.

SURGICAL APPROACH - FLANK INCISION

# 77 - Pheochromocytoma
28 y.o. female swimmer gets headache after workouts. BP = 220/120 after workout.
Normally the patient has a BP of 130/80. CT scan shows a right adrenal mass. Pt is
referred to you.

Issues:

177
a.) What is the differential diagnosis, i.e. surgically correctable causes of hypertension?

b.) How would you workup this patient? what are the localization modalities? When is
an MIBG scan indicated?

c.) How would you prepare this person for the operating room?

d.) Describe how you would approack this person surgically? What type of incision?
Unilateral or bilateral exploration? How would you approach the mass? How would
you improve the exposure on the right or left? how would you control blood pressure
intraoperatively? What if you remove the mass and the patient’s BP does not
improve?

ANSWER:

DIFFERENTIAL DIAGN0SIS:
1.) PHEOCHROMOCYTOMA
2.) COARCTATION OF THE AORTA
3.) RENAL ARTERY STENOSIS
4.) ALDOSTERONOMA

DIAGNOSIS - most effective means is by measuring the catecholamine levels and levels
of metabolic products in the urine.
24 hr. Urine Collection - for either catecholamines or metanephrines . Levels elevated
above normal limit for each test will yield a sensitivity of 95%. The two tests
combined will yield a sensitivity of 98%. Addition of urinary VMA can yield almost
100% sensitivity. Important for collection to be started close to an episode of
symptoms or documented hypertension as those who have pheochromocytoma and
are episodic secretors may not demonstrate elevated levels if the collection is not
started at the time of documented HTN/symptoms.

Urine vs. Plasma measurements - collection of urine over 24 hrs. smooths out all the
variations , peaks and troughs of catecholamine secretion which occur naturally.
Plasma measurements capture only one point in time and make it difficult to
differentiate a normal surge of catecholamine from a minimally secreting
pheochromocytoma.

LOCALIZATION STUDIES -
CT SCAN - can reliably detect tumors of 1 cm in size 80% of time and pick up tumors
3-4 cm 100% of time. Problem is that it cannot differentiate benign from malignant
tumors and functional from non-functional tumors. Overall, CT scan will localize
96% of pheochromocytomas.

178
MIBG SCAN - should be used if CT scan does not show adrenal mass or if there is
concern about metastasis. Useful for the diagnosis of mets or in localizing an
extraadrenal pheochromocytoma.
Nota Bene - use of IV contrast in CT scan or thru the use of arteriography can
precipitate a hypertensive /hypercatechol crisis in those with pheochromocytoma and
should be avoided. Contrast may be used if the unenhanced CT does not show tumor,
but only after the patient has been adequately alpha adrenergically blocked.

PREOPERATIVE PREPARATION:
1.) ALPHA BLOCKADE - PHENOXYBENZAMINE - MUST BE USED FIRST -
start with 20 mg per day and increase by 10 mg per day increments until the BP is
controlledwith patient in the erect and supine position. This should be done such that
the drug is on board 7-10 days before surgery. This drug is stopped at midnight
prior to surgery. Slow progressive alpha blockade will allow volume expansion over
the week preop and will obviate need for fluid and prevent hypotension intraop.
2.) BETA BLOCKADE - PROPRANOLOL - START 48 HRS. BEFORE SURGERY.
- start at 30-40 mg per day. Last dose is given 1 hr. preop with sip of water.

INTRAOP BP CONTROL -
MONITORING - at least central line and arterial line. PA catheter should be used
selectively in those with cardiac history and all over 60 yrs of age.
BP CONTROL - PHENTOLAMINE AND NITROPRUSSIDE - should be used intraop
for BP control. Should use lidocaine and propranolol for arrhythmias.
ANESTHESIA - use enflurane instead of halothane be cause of arrhythmogenic
properties.

SURGERY -
?POSTERIOR APPROACH - should be avoided because it necessitates too much
maniplation of the tumor and gives poor visualization of the adrenal vein, both things
needed in removing a pheochromocytoma.
INCISION - LONG MIDLINE OR BILATERAL SUBCOSTAL - place self retaining
retractor .
EXPOSURE - RIGHT ADRENAL - take down the hepatic flexure of the colon and
Kocherize the duodenum. Use left hand to retract the right kidney caudad. Divide
Gerota’s fascia without detaching adrenal gland. The liver can be mobilized by
dividing the right coronary ligament and reflecting the liver superiorly and to the
midline. The right adrenal vein is broad and short and often enters the IVC
posteriorly. Hemoclips may be used here to avoid avulsing the vein.
- LEFT ADRENAL - divide the gastrocolic omentum to enter the lesser
sac. Divide the peritoneal covering of the inferior border of tha pancreas and divide
the fatty tissue for superior rotation of the pancreas. The spleen can be mobilized
completely and developing plane between the posterior aspect of the pancreas and the
anterior surface of Gerota’s fascia. The left adrenal vein is longer and narrower as it
enters the left renal vein. This should be tied.

179
RESECTION - after ligation of the vascuar supply, the entire adrenal and margin of
adjacent fat should be removed. Avoid rupture of the capsule as this can predispose
to recurrence.

HYPOTENSION AFTER RESECTION - expected. If there is persistent hypertension


after the adrenal is removed, search should be made for second pheochromocytoma in
the abdomen ( contralateral adrenal gland or in the organ of Zuckerkandl.)
REMEMBER - 10% ARE BILATERAL
- 10% ARE EXTRA-ADRENAL
- 10 % ARE MULTICENTRIC

BILATERAL PHEOCHROMOCYTOMA - necessitates bilateral adrenalectomy

PALLIATION - since mortality with these tumors is related more to excess


catecholamine secretion and less to tumor spread, attempts should be made to resect
as much tumor as possible in the case of metastatic disease. This may make the
hypercatecholamine syndrome easier to control with phenoxybenzamine.

POSTOPERATIVE CARE - patient should remain in ICU for 24 hrs for BP monitoring.
Catecholamine levels should be checked before the patient leaves hospital. Most
patients will become normotensive, but 25-30% will remain hypertensive postop
because of development of essential hypertension or renovascular changes.

MEDICAL THERAPY - for those who cannot tolerate surgery or those who have disease
that cannot be resected, continuous alpha and beta blockade with proprnlol and
phenoxybenzamine.

# 78 - Incidentaloma
50 y.o. white male is evaluated for right hypochondriac pain with an ultrasound. Normal
gallbladder, but a 4 cm mass in the right adrenal gland. No symptoms.

Issues:
a.) What is the differential diagnosis of this mass.
b.) What tests would you want to order to narrow your differential diagnosis? Would you
biopsy the lesion if you were to decide to follow this lesion?
c.) Assuming all tests come back negative, at what size would you operate on an
incidentaloma?
d.) How would you approach this? (Posterior, flank, transabdominal).

ANSWER:

DIFFERENTIAL DIAGN0SIS:

180
1.)Adrenal Cortical Adenoma
2.)Adrenocortical Carcinoma
3.) Pheochomocytoma
4.)Ganglioneuroma
5.) Cyst
6.) Organized Hemorrhage or Fibrosis
7.) Myelolipoma
8.) Adenolipoma
9.) Metastasis from nonadrenal primary (e.g. lung)

LABORATORY TESTS -
METASTASIS FROM MALIGNANCY - stool for occult blood
- CXR
- Mammogram ( in woman)
HORMONAL TESTING (in aymptomatic patient)
- serum potassium (rule out aldosteronoma) - if this is decreased, then a 24 hr.
collection for aldosterone should be done. If this is found to be elevated, then a
serum renin should be performed. Hypokalemia, elevated aldosterone in 24 urine
collection and decreased serum renin confirms aldosteronoma.
- 24 hr. urine collection for catecholamines, VMA and metanephrines. (rule out
pheo)- this alone will generally diagnose pheo.
- 24 hr. urine collection for 17- hydroxysteroids and 17-ketosteroids (rule out
Cushings disease) - this can be followed by a Dexamethasone suppression test if
these levels are elevated in 24 hr. urine collection. High Dexamethose should act on
pituitary to decrease 17-hydroxycorticosteroid levels in urine after administration.
17-hydroxycorticosteroid levels will remain elevated in patient with autonomously
functioning adrenal tumor. This test should be confirmed with a decreased serum
ACTH level.
- elevated 17- ketosteroid levels will generally demonstrate some signs of
feminization or masculinization but should be confirmed with serum estrogen and
testosterone levels.

CYSTIC ADRENAL MASS - a cystic adrenal mass measuring 3-6 cm b y CT and


having a thick wall should be percutaneously biopsied. Fluid from this cystic
structure should be sent for cytology. Malignant cytology necessitates resection.

ADRENAL MASS IN PATIENT WITH HISTORY OF MALIGNANCY - tend to come


from melanoma, renal cell CA, breast, lung and gastric CA. In patients with history
of malignancy, the mass should be confirmed by percutaneous needle biopsy. Before
undergoing percutaneous biopsy, the patient should be screened with ruinary
catecholmines, VMA and metanephrines to avoid a hypertensive crisis if this turns
out to be a pheochromocytoma.

181
- while a percutaneous needle biopsy can distinguish a metastasis from a primary
adrenal tumor (malignant or benign) , it cannot differentiate a malignant from a
benign adrenal tumor.

MANAGEMENT OF INCIDENTAL ADRENAL MASS -


CRITERIA FOR RESECTION:
1.) AGE < 50 AND MASS 3 CM OR LARGER.
2.) MASS 3-6 CM WITH SUSPICIOUS CT CHARACTERISTICS
3.) RESECT ALL MASSES 5 CM OR GREATER - because of greater risk of
malignancy.
4.) AGGRESSIVE APPROACH - ANY GREATER THAN 3 CM.

CRITERIA FOR OBSERVATION:


1.) AGE OF 50 YRS. OR OLDER AND MASS 3-5 CM.
2.) ALL MASSES LESS THAN 3 CM AND NOT HORMONALLY ACTIVE.

OBSERVATION - CT Scan every 3 months for first year and annually thereafter. Along
with CT scan, patient should have serum potassium, 24 hr urine collection for VMA,
metanehrines and catecholamines along with 17- ketosteroid and17-
hydroxycorticosteroids.

SURGICAL APPROACH - transabdominal apporach is best to be able to examine the


abdomen for evidence of other primary tumor ( making the adrenal tumor a met) or
for staging the abdomen for mets if this adrenal mass turns out to be an adrenocortical
carcinoma.
- see algorithm in Surgical Clinics , June ‘95, p. 508.

# 79 - Severe Hypercalcemia
50 y.o. male presents with lethargy progressing to unresponsiveness.

Issues:
a.) What is your initial ER workup to determine the cause of his unresponsiveeness ?
(cbc, Ca= 16.5, sodium = 145, pH=7.2, glucose =110)
b.) What are the possible causes of hypercalcemia? What other lab tests would you want
to narrow the differential? (PTH level, Cl-/PO42- ratio) What radiographic tests
would help narrow the differential diagnosis? The patient has a right neck mass on
physical examination.
c.) What is the medical managment of this patient? What is the tentative diagnosis and
how would you plan to treat this? What are the percentages of this being an
adenoma? hyperplasia/ multiple adenomas? parathyroid CA?
d.) What is the surgical approach to this problem? Incision, site of exploration, chance of
more than one adenoma? Where would you look for ectopic parathyroid glands?
What would you do if three normal glands are located? Would you perform a median

182
sternotomy if the adenoma has not been found? How would you monitor the patient’s
Ca++ level postop?

ANSWER:
INITIAL WORKUP:
1.) CBC
2.) ELECTROYTES WITH GLUCOSE
3.) LFT’S WITH AMMONIA AND CALCIUM
4.) ABG
5.) HEAD CT

CAUSES OF HYPERCALCEMIA:
1.) HYPERPARATHYROIDISM - parathyroid adenoma, parathyroid hyperplasia,
parathyroid carcinoma.
2.) MALIGNANCY/DIFFUSE BONY METASTASIS
3.) MULTIPLE MYELOMA
4.) THIAZIDE DIURETICS
5.) HYPERTHYROIDISM
6.) MILK-ALKALI SYNDROME - those using milk and antacids for treatment of ulcer.
7.) SARCOIDOSIS
8.) PROLONGED IMMOBILIZATION

WORKUP OF HYPERCALCEMIA:
1.) PRIMARY HYPERPARATHYROIDISM - Calcium, albumin, phosphate, chloride,
Parathormone Level

Calcium, albumin - need albumin to calculate the actual Ca as patients with slightly
elevated calcium may have very low albumin. Alternatively, obtai n ionized
calcium.

Chloride/Phosphorus Ratio - greater than 33 is virtually diagnostic of primary


hyperparathyroidism.

Parathormone level - combination of elevated parathormone level with elevated Ca


is almost diagnostic of parimary hyperparathyroidism.

HAND X-RAYS - show the subperiosteal bone resorption in the middle and terminal
phalanges of digits.
BONE DENSITOMETRY

2.) MALIGNANCY/DIFFUSE BONE METS-


- in cases of diffuse metastatic malignancy causing hypercalcemia, the phophate level
will not be depressed as in primary hyperparathyroidism.

BONE SCAN

183
CXR

LOCALIZATION OF PARATHYROID TUMOR


- in the hands of an experienced endocrine surgeon, the success rate with initial neck
operation is 95%. However, it is sometimes difficult to differentiate single gland
from multiple gland disease and to distinguish a normal from an abnormal gland.
Preoperative localization should definitely be performed in cases of recurrent or
persistent hyperparathyroidism aftert an unsuccessful first operation.

MODALITIES:
US/CT SCAN - sensitivities of 50-60% because tumors are small .
THALLIUM/TECHNITIUM - Thallium taken up by thyroid and parathyroid, Tech
taken up by thyroid. Subtract the 2 images. Useful for inferior pole and mediastinal
tumors.

DSA - sensitivities of 50-60%.


Parathyroid Venous Sampling - may be difficult in case of previous surgery as the
veins may have been ligated. Has sensitivity of 80%. Should try SSA or DSA prior
to attempting parathyroid venous sampling.

TREATMENT OF SEVERE HYPERCALCEMIA -


- though surgery for severe hypercalcemia is the ultimate therapy, it is unwise to proceed
to neck exploration until the calcium level has decreased.
- Admit to ICU for monitoring. Place central line. Place Foley catheter
FLUID - Normal Saline - 1L every 3- 4 hrs. (250-300cc/hr.)
DIURETIC - Lasix - 100 mg/hr.
---> want to ensure a urine output of at least 100 cc/hr. May need to fluid resuscitate
prior to administering Lasix. Careful of pulmonary edema in GOMERS.

CALCITONIN - 1-5 units/kg body wieght per day IV - weak agent but works
quickly.

MITHRAMYCIN - 25 micrograms /kg body weight - useful in malignancy but can


cause thrombotic disorders and liver damage/failure.

PROBLEM - elevated calcium levels from hyperplasia or benign adenoma will often
decrease with medical therapy but those with hypercalcemia from tumor (parathyroid
or other ectopic parathyroid secreting malignancy) often will not respond to medical
therapy.

CAUSES OF PRIMARY HYPERPARATHYROIDISM:


1.) ADENOMA - 85%
2.) HYPERPLASIA - 14%
3.) CARCINOMA - 1%

184
SURGICAL THERAPY FOR PRIMARY HYPERPARATHYROIDISM:

INITIAL OPERATION:
PREOP PREPARATION - assess vocal cord function preop.

Standar d collar incision is made. Subcutaneous fat and platysma are divided.
Superficial veins are ligated. Tissue plane superiorly and inferiorly created. Strap
muscles divided thru the linea alba. The posterior strap muscles dissected from the
anterior surface of the thyroid thru loose areolar tissue. Middle thyroid veins are
ligated. The thyroid lobes are mobilized and rotated medially. The recurrent
laryngeal nerve on each side is visualized.

ADENOMA - REMOVE, OTHER GLANDS NOT DISTURBED.


MICROADENOMA - REMOVE, BIOPSY GLAND ON SAME SIDE - to make sure
that this gland is not an adenoma or hyperplastic gland.
HYPERPLASIA - REMOVE ALL GLANDS AND AUTOTRANSPLANTATION.
2 ENLARGED GLANDS + 2 NORMAL GLANDS - remove the enlarged glands and
leave the normal glands.
3 ENLARGED GLANDS + 1 NORMAL GLAND - remove enlarged glands and leave
the normal gland.

ONLY 3 GLANDS IDENTIFIED:


SUPERIOR GLAND ECTOPIC SITES - within thyroid gland, retroesophageal
(posterior mediastinum), carotid sheath, superior thyroid pedicles.

INFERIOR GLAND ECTOPIC SITES - inferior pole of thyroid, within the thymus,
within the thyroid, superior/anterior /posterior mediastinum.

3 NORMAL GLANDS + NO GLAND IN ECTOPIC SITES - perform thyroid


lobectomy on side of missing gland.

GLAND IN MEDIASTINUM -
1.) Suspected Gland in the Mediastinum - can mobilize the thyrothymic ligament
thru the cervical incision to pull the thymic remnant into the incision for inspection.
2.) Median Sternotomy - because of high morbidity associated with median
sternotomy, the following criteria should be met:
I.) previous neck exploration by experienced surgeon.
II.) preop studies showing suspicious area in chest/mediastinum.

PARATHYROID CARCINOMA - rare. Usually have very high Calcium and


palpable neck mass. Perform en bloc resection of carcinoma and ipsilateral thyroid
lobectomy. perform radical neck dissection for any palpable nodes.

185
POSTOPERATIVE HYPOCALCEMIA -
- expect the Ca level to drop by 1-2 per day until it levels off.
- need to check for symptoms at least 2X per day
“Chvostek’s” - tap facial nerve anterior to the ear.
“Trousseaus’s” - carpal spasm after inflating BP cuff above systolic pressure for 2-3
minutes.
- check calcium twice per day for first three days.

Symptoms or Ca < 7.5 - give 2-3 amps 10% Ca Gluconate (Ca chloride will burn the
veins) no faster than 2ml per minute. This will likely be “transient hypocalcemia”
that will resolve when the other glands become reactivated - no further therapy is
needed.

Persistent Hypocalcemia - start oral calcium at 1 gram of calcium carbonate per day.
When the patient is asymptomatic and the Ca level has stabilized, the patient can be
discharged. The oral calcium can be stopped after several weeks. It is preferable not
to add vit.D therapy unless permanent hypocalcemia is suspected or if an
autotransplant into the forearm has been performed. If this doesn’t work, add vit D.

Permanent Hypocalcemia - vit. D 50,000 - 100,000 units per day along with 2 gr of Ca
Gluconate. Check Ca and phosphorus levels 2X/month.

VASCULAR

# 79 - Presentation of Ruptured AAA


Massively obese 55 yr. male presents to the ER with a protuberant abdomen, HR= 140
and BP=60/palp.

Issues:
a.) Initial therapy and diagnosis including how do you establish venous access in this
patient.
b.) Assuming he remains hypotensive after fluid administration and he is too large for
traditional imaging modalities, what do you do?
c.) Assuming a midline exploration shows a midline lower retroperitoneal hematoma,
what is the most likely diagnosis and how do you manage it?
d.) Diagnosis is a ruptured AAA, how do you operatively manage this problem
immediately and definitively?

ANSWER:

ABC’s
A= Airway - if obtunded and unresponsive - intubate. However, muscle relaxants,
narcotics or sedatives should not be used for induction until the patient is prepped
and draped and incision is about to be made.

186
B= 02 - 5-10L per face mask.
C= Circulation . Infuse 2L LR thru quickly placed Swan Ganz Introducer. Type and
cross for 10U prbc’s, 10U platelets and 10U FFP. Do not try to push fluids to achieve
normotension before the aortic cross clamp as this may overcome the tamponade
effect of the retroperitoneal blood.
- place Foley catheter.
- place art line in upper extremity if capabilities exist and this can be done quickly.
- give prophylactic antibiotics.(Vanco 1 gr. IV).

TRIAD: ABDOMINAL/BACK PAIN, HYPOTENSION AND PULSATILE


ABDOMINAL MASS - Ruptured AAA until proven otherwise.

Physical Findings - pulsatile abdominal mass may or may not be present. Flank
ecchymosis is a reliable finding of retroperitoneal bleeding but often a late finding.

OR - the place to diagnose a ruptured AAA, especially with severe hypotension.


CT SCAN - only if : hemodynamically stable and if doubt exists about the diagnosis.
Obviously not indicated in this case.

OR -
- Hold Induction of General Anesthesia until the patient is prepped and draped and
incision is ready to be made. Have anesthesiologist place Swan Ganz catheter /IJ
Swan Ganz Intro prior to induction of anesthesia.

Prep - Neck to Kness - make certain that the left chest is adequately prepped in case a
thoracotomy is necessary for cross clamping above the diaphragm because of
hypotension in an infrarenal aorta or upon finding a juxtarenal or suprarenal
aneurysm.

Incision - large midline

PROXIMAL CONTROL OF AORTA -

INTRAPERITONEAL RUPTURE OF AORTA - occlude with finger and place a Foley


catheter with 30 cc balloon in hole and inflate balloon. This can temporize until
proximal neck can be dissected out.

REASONABLY STABLE - gain initial control at the diaphragm. Divide the left
triangular ligament of the liver and rotate the left lateral segment medially. Palpate
the NGT and sweep the esophagus to the right. Palpate the pulsatile aorta. Divide the
peritoneal covering over the pulsatile aorta and identify the crural fibers. Dissect the
crural fibers away from the aorta and place clamp tips against vertebral bodies.

187
- deterioration to profound shock - alternatively can occlude the aorta here by
compressing the aorta against the vertebral bodies using a fist or metal retractor.
Problem is that this compression is not as secure or complete as the cross clamp.

PROFOUND SHOCK - Left Thoracotomy - assistsin quick cross clamp and control
while dissecting out aorta and preventing injury to renal arteries, IVC, pancreas
PREOPERATIVE CARDIAC ARREST - Left Thoracotomy - allows access for cardiac
massage.
PREVIOUS UPPER ABDOMINAL SURGERY - Left Thoracotomy - for obvious
reasons.

EXPOSE NECK AND PLACE INFRARENAL CROSS CLAMP - eviscerate the


transverse colon and small bowel and pull the duodenum to the right . Divide the
peritoneal covering along the Ligament of Treitz. The retroperitoneal hemorrhage
will usually dissect the IVC away from the neck, but care must be taken to avoid
injuring the IVC, lumbars or left renal vein when placing the cross clamp.
- if the left renal vein in jured during dissection or obscures the neck, it should be
ligated.
- once the infrarenal clamp is placed, let of the suprarenal clamp slowly and expect a
period of hypotension. Advise the anesthesiologist prior to removing clamp to allow
time to prepare the patient with fluid bolus.

DISTAL CONTROL - identify and clamp at the iliacs. If the area will have dissection
obscured by hematoma, pinch each iliac and gain control from within the aorta using
large Fogarty catheters or Foley catheters.

OPEN ANEURYSM - open and remove mural thrombus. Gain control of iliacs from
inside if this has not already been done. Oversew the IMA and any lumbars.

GRAFT - TUBE GRAFT IF POSSIBLE - small iliac aneuryms are ignored. Place a
bifurcated graft only for (1) large iliac aneuryms (2) severely calcified aortic
bifurcation (3) severe iliac occlusive disease.

BACKBLEED ILIACS - if backbleeding is poor, pass Fogarty catheters and flush with
dilate heparin.

CHECK DISTAL PULSES - check the femorals for palpable pulses and the pedal
arteries for dopplerable signals. If pedal pulses are absent, consider a distal
thromboembolectomy - base decision on the condition of the patient.
- close aneurysm sac and check the sigmoid colon for viability.

SPECIAL CONSIDERATIONS -

188
JUXTARENAL/SUPRARENAL AAA - most of these are juxtarenal rather than
suprarenal. In this case, the clamp should be placed above the renals and the
proximal anastomosis performed incorporating the lower margins of the renal artery
ostia into the anastomosis if necessary. Once the anastomosis is completed , it is
tested and the clamp is repalced on the graft below the anastomosis to allow perfusion
of the renal arteries.
- for a true suprarenal AAA , perform a medial rotation by mobilizing the spleen,
pancreas, Left colon and left kidney toward the midline. The celiac, SMA , right
renal artery are taken on a large patch and anatomosed to the graft. The left renal
artery orifice is anastomosed to the graft separately.

AORTOCAVAL FISTULA - rare as hell. Present with pain, mechanical abdominal


bruit, hematuria (from venous bladder congestion), and CHF. Will not present with
exanguinating hemorrhage. Once this is diagnosed, do not attempt to dissect out
fistula. Occlude the vena cava with sponge sticks. Place proximal and distal clamps.
Open the aneurym sac and close the defect in the cava from within the aortic sac
using large bites.

AORTODUODENAL FISTULA - also rare as hell. Present with upper GI bleed and
occasionally with pain. Unlike a secondary aortoduodenal fistula in which the
previously placed graft is exposed to the duodenum, the risk of infection is low.
- primarily close the duodenal defect and repair AAA with Dacron graft.

COMPLICATIONS OF RUPTURED AAA REPAIR -


COMPLICATIONS COMMON WITH EMERGENCY OPERATION OR AAA
REPAIR:
- M.I.
- Hemorrhage
- Renal Failure
- Respiratory Failure.

COMPLICATIONS SPECIFICALLY OF RUPTURED AAA:


- Ischemic Colitis - chances of this complication are lowered if the left colon is carefully
examined after AAA repair. If the colon is left at the end of the operation must be
vigilent and aggressive inthe evaluation of DIARRHEA, SEPSIS, ACIDOSIS.
-> FULL THICKNESS NECROSIS - resect and end colostomy.
-> DUSKY MUCOSA - antibiotics, gut rest, optimize cardiac output and observe.
- Spinal Cord Ischemia - paraplegia, fucked.

# 80 - Elective AAA with Hx of Colon CA.


65 y.o. male with history of transverse colectomy for colon cancer 5 yrs. ago. Ultrasound
shows 5 cm AAA.
Issues:

189
a.) How would you evaluate this patient preoperatively? (Cardiac workup and colon
workup). Radiographic evaluation? (make sure and have a CT scan to differentiate a
suprarenal aneurysm or a mycotic aneurysm).
b.) Anticipating the marginal artery has been disrupted with the previous colon resection,
would you reimplant the IMA?

# 82 - AAA/Reimplantation of IMA/Ischemic colitis


79 y.o. man is found on routine exam to have a 5 cm AAA. HTN controlled with meds,
very active, otherwise healthy.

Issues:
a.) How would you evaluate this AAA? At what size would you operate on his AAA?
b.) how would you evaluate his cardiac status? How would you explain to this patient the
risks of operative repair vs the risk of rupture?
c.) What are the indications for reimplanting the IMA? If the IMA is not reimplanted, or
is reimplanted and subsequently thromboses and the patient develops ischemic colitis
- how would you workup this patient’s ischemic colitis? How would you treat this?
If you explore the patient and find normal bowel ( sigmoidoscopy shows edematous,
boggy bowel with dusky gray mucosa) - what do you do? Disease that is not
transmural will likely heal and should not be resected.

ANSWER:

PREOPERATIVE EVALUATION:
PERTINENT HISTORY - chest pain, shortness of breath, exercise tolerance, symptoms
of claudication and ability to have erections.
EXAM - palpable pulses in lower extremities, esp. palpable femoral pulses, rectal.
LABS - cbc, lytes, BUN, Cr, PT/PTT, LFT’s(if indicated)
CXR
EKG
ABI’s (if indicated)
PFT’s - if clinical evidence of COPD.
Carotid Dopplers if evidence of cerebrovascular disease. Only symptomatic carotid
disease should be operated on prior to AAA.
CT Scan of Abdo/Pelvis - to identify an inflammatory aneurysm or the presence of a
horseshoe kidney.
Angiogram - if evidence of iliac disease/occlusion, suspicion of renovascular
hypertension or history of visceral ischemia symptoms.
Exercise Thallium - especially with history of chest pain/angina , previous MI, SOB, no
exercise (because of disability ,disabling claudication , severe SOB, etc) or evidence
of previous MI on EKG. Those with reperfusion abnormality should have coronary
angio . If coronary angio shows disease amenable to bypass, CABG should be
performed prior to attempted AAA repair. AAA repair should follow CABG by 6-8
weeks.

190
Severe Uncorrectable CAD - those with Ejection Fraction < 20%, and those with
unreconstructible CAD should have AAA repair only for large AAA.

RISK OF RUPTURE - in an asymptomatic aneurysm, main issue is whether the risk


of rupture exceeds the risk of elective operation. Factors to be considered include
(1) size, type and location of the aneurysm (2) comorbid diseases and risk factors.

SIZE AND TYPE OF ANEURYSM - 5 year risk of rupture exceeds the risk of operation
on ce the AAA reaches 6 cm. Because of the 20% risk rupture of small AAA, the
recommendation is to repair AAA once it reaches 5 cm.
- Aneurysms of 4 cm or less should be followed with CT scan every 6 months.
When progressive enlargemnt occurs or there is a complication (such as
embolization), operation is recommended.

ARTERIAL ANATOMY - the blood supply to the left colon is mainly thru branches of
the IMA. However, the IMA is frequently occluded or thrombosed in patients with
AAA. When IMA is occluded, collateral circulation is provided through collateral
vessels from the SMA (via the meandering mesenteric artery and the marginal artery
of Drummond, which has been interrupted in this case from the transverse
colectomy) and from the hypogastric (internal iliac) arteries.

COLLATERAL PATHWAY FROM SMA ->IMA -


1.) Marginal Artery of Drummond - LESS IMPORTANT- from SMA to the left branch
of the middle colic artery via the marginal artery of Drummond at the splenic flexure
of the colon to the left colic branch of the IMA.(peripherally located)

2.) Large Meandering Mesenteric Artery - MORE IMPORTANT - is located to the left
of an infrarenal aorta and originates near the SMA at the middle colic origin and
connecting the IMA circulation at the left colic artery origin. This anatomical variant
is found in 2/3 or normal people and can dilate up to 3 mm in a patient who develops
compromise of the IMA. Alternatively, if the SMA becomes stenotic, the IMA may
feed much of the SMA circulation (entire midgut) thru the meandering artery .
(centrally located) . It is unlikely that the Meandering Artery has been
compromised during a transverse colectomy.

HYPOGASTRIC ARTERIES - middle and inferior rectal branches of the hypogastric


may communicate with the superior rectal branch of the IMA.

Patients at Increased Risk for Postoperative Intestinal Ischemia - (1) History of Visceral
Ischemia Symptoms. (2) Previous Colon Resection (3) Emergent Aneurysm Repair.

Preoperative Arteriogram - if done, should focus on (1) patency of IMA (2) patency or
stenosis in SMA (3) Presence of Meandering Mesenteric Artery as well the direction

191
of flow. If there is flow from the SMA to the IMA , it is alright to ligate the IMA. If
there is flow from the IMA to the SMA, ligation if the IMA will result in infarction of
the small and large bowel. If there is flow thru the MMA from the IMA to the SMA,
will need to reimplant the IMA and revascularize the SMA. The mere presence of a
meandering artery does not provide enough information to justify the ligation of the
IMA (4) the presence of hypogastric flow. If both are occluded, the rectal blood flow
is coming from the IMA or the SMA thru the marginal or meandering artery. If this
is the case, will want to revascularize one hypogastric artery.

INTRAOPERATIVE EXAMINATION OF THE IMA - if there is no bleeding (occluded)


or brisk pulsatile backbleeding (adequate descending colon collateral flow), the IMA
can be ligated.
- if there is sluggish, nonpulsatile flow, the IMA should be surrounded with a Potts’ed
vascular tie for temporary occlusion which will not damage the artery. Need to assess
adequacy of collateral flow.

INTRAOPERATIVE ASSESSMENT OF COLLATERAL FLOW - examination of colon


serosa is not considered an adequate indicator of collateral blood flow.
2 METHODS:
1.) DOPPLER - doppler signal at the antimesenteric border during temporary IMA
occlusion indicates inadequate collateral flow.
2.) IMA stump Pressure - > 40 mmHg or ratio >0.4 can be used as indicators or
collateral flow and allow safe ligation of IMA.

ISCHEMIC COLITIS COMPLICATING AAA REPAIR -


PATHOPHYSIOLOGY - colonic hypoperfusion is usually the result of ligation of a
hemodynamically important IMA. May also be due to marginal mesenteric artery
with an interruption of hypogastric collateral flow during the aortic cross clamp.
Colonic ischemia is the most common ischemic complication though the small bowel
may be compromised if the IMA is ligated and there is a stenosis in the SMA (this is
rare).

3 CLASSES OF ISCHEMIC COLITIS BASED ON DEPTH OF PROCESS:


1.) MUCOSAL - 20% of time. Mild and resolves.
2.) MUSCULARIS - 20% of time. Does not cause perforation but causes fibrosis and
eventual stricture.
3.) TRANSMURAL - 60% of time. Gangrene and perforation result.

PROBLEM WITH ISCHEMIC COLITIS - difficult to diagnose postoperatively, so


evaluation should be aggressive, including mandatory colonoscopy in high risk
patients.
SYMPTOMS - bloody or watery diarrhea, unexplained fever, leukocytosis, abdominal
pain, abdominal tenderness, increased fluid requirements, occult sepsis and metabolic
acidosis. Presence of any of these mandates evaluation.

192
Findings - mucosal ulceration should be treated with antibiotics, bowel rest ,
hyperalimentation , optimization of cardiac output ( if this is a contributing factor)
and serial colonoscopy.

Exam - abdominal findings on physical exam imply a transmural process , making


exploration and treatment mandatory

SURGICAL TREATMENT OF ISCHEMIC COLITIS - immediate measures to


minimize contamination and possible infection of graft. Evaluate the colon visually
and using Doppler exam.
COLON ISCHEMIA - resect compromised colon, end colostomy , Hartmann’s or mucus
fistula.
RECTAL ISCEMIA - debride involved rectum and irrigate . Provide drainage.
SMALL BOWEL ISCHEMIA - debride frankly necrotic or perforated small bowel.
Revascularize the SMA , with resection of necrotic bowel. Plan second look
operation.

# 81 - Elective Large AAA with Colon CA


70 y.o. male with constricting (not obstructing) sigmoid cancer and asymptomatic 8 cm
AAA.

Issues:
a.) Which one do you repair first?
b.) Describe sigmoidectomy.
c.) Describe AAA repair.
Pitfall - if choosing a retroperitoneal repair of AAA, be prepared to describe everything
and defend control of the right iliac as well as treatment of iliac aneurym.
d.) If forced to operate on the AAA because of leakage, how will you deal with the colon
CA?
Pitfall - if dealing with both at once, be prepared to deal with a graft infection.

Alternatively - Pt. with an 8 cm AAA and splenic flexure tumor with heme positive
stools.
Issues:
a.) Which do you operate first?

ANSWER:

BOWEL TUMORS AND ANEURYMS - performing both procedures concommitantly is


much more likely to be accompanied with life threatening complications and should
not be performed.

193
TUMORS DIAGNOSED PREOP - THE SIZE OF THE ANEURYSM AND THE
DEGREE OF OBSTRUCTION SHOULD BE CONSIDERED WHEN DECIDING
ON WHICH SHOULD BE MANAGED FIRST.

RESECT BOWEL FIRST - AAA is less than 5 cm diameter.


REPAIR AAA FIRST - if AAA is > 5 cm diameter or symptomatic.

TUMORS DIAGNOSED INTRAOP - tumors should be left alone unless they are
obstructing or pre-obstructing.
Obstructing or Preobstructing - if bowel prep is done, conventional colon resection is
performed. If the bowel has only been prepped with enemas and there is there is an
obstructing tumor of the jejunum or right colon, can resect. If the lesion is anywhere
else and a complete bowel prep has not been done, close and prep the bowel.
Nonobstructing Tumor - leave alone, perform AAA and return for colon resection in
3weeks or later if postop course is slow.

Obstucting lesion and symptomatic AAA - repair AAA, close retroperitoneum , perform
proximal diverting colostomy to be opened later or exteriorize tumor and resect
later with double barrel colostomy.

# 83 - Chronic Mesenteric Ischemia


60 y.o. female w/o sig. PMHx presents with post-parandial pain, diarrhea, and 10 lb.
weight loss.

Issues:
a.) Differential diagnosis.
b.) How would you work this up?
c.) What would you expect to see on the visceral arteriogram?
d.) How would you approach this? What kind of conduit would you use?
Which vessel (s) would you revascularize and how would you place the graft?

ANSWER:

- Chronic Mesenteric Ischemia results from inadequate perfusion of the midgut during
periods of increased oxygen demand. 02 demands of bowel increase significantly in
the postparandial period because of rises in motility , secretion and absorption.
Chronic Mesenteric Ischemia is differentiated from Acute non occlusive mesenteric
Ischemia in which the mesenteric arteries undergo severe vasoconstriction from
vasoactive medications (norepi) or by decreased cardiac output from MI,
hypovolemia or arrhythmia.

- Hypoxic injury is manifested by ischemic visceral pain and abnormlitiesin absorption


and motility. Pain is similar to myocardial ischemia or angina.

194
- Atherosclerotic involvement of large arteries is almost always the cause, but it can be
related to vasculitis such as Buerger’s or polyarteritis.

Differential Diagnosis -
1.) Chronic cholecystitis
2.) Pancreatic CA
3.) Gatric CA
4.) Colon CA.
5.) PUD

WORK - UP:

DIAGNOSIS - No specific releiable test exists, diagnosis must be based on clinical


symptoms ( must have weight loss, should have pain) and angiographic evidence
showing occlusion of the splanchnic arteries and to a greater degree the exclusion of
other gastrointestinal causes.

ANGIO - stenosis or occlusion of one or more of the major visceral vessels on angio does
not itself establish the diagnosis of arterial insufficiency. Likely have stenosis
/occlusion of at least two of the three arteries. The presence of prominent
collaterals not only indicates major artery stenosis but also indicates a chronic
process.
Likely show a large meandering mesenteric artery.

INDICATIONS FOR SURGERY - (+) diagnosis = pain with (+) angio findings. Fear
of impending thrombosis is not an indication. The only indication for repair in the
absence of pain is patient undergoing artic reconstruction (AAA or occlusive
disease ) in which a preop angio shows a stenosis /occlusion of the SMA or celiac
with large meandering artery and patent IMA.

TREATMENT - limited success with transluminal angioplasty, but not much experience
or data - may be used in the case of sick patient who cannot tolerate an operation.

Preop Preparation - usually require a period of TPN becasue of the long period of
malnutrition. Albumin, PT/PTT should be checked preop.

SURGICAL TREATMENT OPTIONS:

REIMPLANTATION - transect the artery distal to the occlusion and reimplant it in the
aorta below the site of the artery takeoff. Technically difficult because of the severe
disease of the aorta and the long segment of an SMA/celiac stenosis. Should only be
done when aorta is being reconstructed and revascularization is being done
preventitively (see above).

195
ENDARTERECTOMY - can be done transarterially or transaortically. Problem with
transarterial approach is getting the portion of the disease at the most proximal aspect.
Problem with the transaortic approach is need to cross clamp in the supra celiac
position and the inherent risk of renal ischemia with cross clamp.

BYPASS - from the aorta or preferrably iliac to the SMA distal to the site of occlusion is
the procedure of choice. Conduit of choice - reversed autologous saphenous vein.
PTFE or knitted Dacron can also be used if the saphenous vein is unavailable or is
less than 5 mm diameter. There is controversy over the best site of origin of the graft
- debate is over the chance of kinking or twisting from the small bowel mesentery if
the suprarenal position is used vs. the enhanced rate of atherosclerosis in the
infrarenal aorta if this is used as the site of anastomosis.

TECHNIQUE - CELIAC - retract the stomach inferiorly and the liver superiorly to place
the lesser omentum on stretch. The gastrohepatic peritoneum is incised
(superiorly/cephalad) and the gastrocolic ligament is incised (inferiorly/caudad)
opening the lesser omentum. The celiac axis is exposed by sharply dissecting the
sympathetic fibers. Celiac axis is the confluence of the hepatic, splenic and left gastic
- these branches must be dissected out to select the most appropriate site for
anastomosis.

Tunnel - lift the pancreas forward and pass a finger thru the retropancreatic fascia anterior
to the left renal vein into the lesser sac. Once the tunnel is created, a clamp is passed
thru the tunnel and graft is passed.

- SMA - expose freeing the vessel from the root of the mesentery - primary branches
to the jejunum are isolated. Area of the SMA proximal to the major branching
points are selected as the site for the proximal anastomosis. Area proximal to this is
likely occluded.

# 84 - Carotid Disease
60 y.o. hypertensive, moderately obese attorney who smokes 1.5 packs of cigarettes per
day is referred with an assynptomatic right carotid bruit.

Issues:
a.) What do you do if the duplex shows a 40-50% smooth stenosis when the patient is
entirely assymptomatic? How often would you follow the patient?
b.) Assuming that a repeat duplex shows an 80% smoooth stenosis still without
symptoms, what do you do? The patient refuses surgery at this point. What if the
patient had 40% ulcerated plaque without symptoms? What about a 40% stenosis
with ulceration and symptoms?
c.) The patient returns in 1 month having had a bland stroke with complete resolution of
symptoms within one week. What would you advise him?

196
d.) On cardiac workup the patient is found to have a reversible defect on stress thallium.
What would you advise re: cardiac vs. carotid surgery? During wait for cardiac
surgery the patient has RIND. What would you recommend?
e.) How precisely would you perform a carotid endarterectomy? Type of anesthesia,
EEG monitoring, use of shunt, use of patch?
f.) Patient becomes hypotensive during procedure, what is the most likely cause and what
is your treatment?
g.) Postoperatively, the patient develops extreme hypertension in PACU. What is your
differential diagnosis. How would you rule new stroke? How would you treat this
patient? What doses would you use?

ANSWER:

INDICATIONS FOR CEA IN THE ASYMPTOMATIC PATIENT:


- Stenosis of ICA of 75-80%.
Asymptomatic Contralateral Carotid Stenosis Opposite Symptomatic Carotid -
contralateral lesions of less than 75% can be followed clinically. Lesions greater
than 75% should be considered for prophylactic endarterectomy.
Asymptomatic Carotid Stenosis is Patient Undergoing Surgical Procedure - risk of
stroke is 0.3% in patients about to undergo general surgical procedure, 1% in patients
about to undergo vascular procedures and 2-4% in patients about to undergo aortic
reconstruction or CABG/valve. In cases of peripheral or visceral vascular
procedures accompanied by significant carotid stenosis, the symptomatic lesion
should be repaired first. If both are asymptomatic, the carotid is done first (but it is
unusual for either a peripheral or visceral vascular procedure to considered if it is
asymptomatic)
Considerations for Carotid Stenosis and AAA Repair - if both lesions meet
indications for repair (AAA=5-6 cm and ICA=75% or greater), treat the symptomatic
lesion first. If the patient has bilateral asymptomatic critical lesions and an
asymptomatic AAA, fix the carotid first. If asymptomatic unilateral carotid stenosis
and asymptomatic AAA (both meeting criteria for repair), can treat either first. Only
if the AAA is symptomatic and carotid has cresecendo TIA’s should simultaneous
procedures be considered.

Consideration for Carotid Stenosis and Coronary Revascularization -


Risk of Stroke during CABG = 2-4%.
Risk of MI during CEA = 0.5- 2.5%, though it is acknowledged that MI rate is higher
among those with known cardiac history or symptomatic coronary disease. It is
unclear whether the risk of MI, stroke and death is greater for a staged procedure or a
combined procedure. Probably makes more sense to protect a patient with an
asymptomatic carotid stenosis from having a heart attack than a patient with heart
disease from having a stroke.

197
- several risk factors place a patient at risk for MI during or after a CABG - Q-waves,
exertional angina, recent MI, CHF, diabetes, age > 70. These patients should be
evaluated for cardiac disease the same way as the patient undergoing a AAA repair.
EVALUATION :
HISTORY AND PHYSICAL EXAM - for NYHA Classification
EKG
Exercise/ Dipyrdamole Thallium - if any of the above risk factors present .

(+) THALLIUM ---> CORONARY ANGIO.

(+) CORONARY ANGIOGRAM


(1) Lesion Amenable to PTA - should be performed and followed CEA with same or
later hospital admission.

(2) Lesion Not Amenable to PTA, Stable Angina + Bilateral Asymptomatic


/Unilateral Symptomatic Carotid Stenosis = perform CEA under local or under
general with intensive invasive intraop cardiac monitoring. Follow with staged
CABG in 4-6 weeks. Only situation in which carotid done first.

(3) Lesion Not Amenable to PTA, Stable Angina + Unilateral Asymptomatic Carotid
Stenosis = could perform CEA under local but makes more sense to perform CABG
first, followed by staged CEA.

(4) Lesion Not Amenable to PTA, Unstable Angina + Unilateral Asymptomatic -


perform CABG first followed by staged CEA in 4-6 weeks.

(5) Lesion Not Amenable to PTA , Unstable Angina + Unilateral Symptomatic or


Bilateral Asymptomatic = combined procedure .

(6) Lesion Not Amenable to PTA, Presence of Left Main Disease or Poor LV
function - combined procedure .

(7) Severe coronary disease with poor EF not amenable to CABG/PTA +


symptomatic carotid stenosis = CEA under local .

(8) Severe coronary disease with poor EF not amenable to CABG/PTA +


asymptomatic carotid stenosis = medical therapy.

TECHNIQUE = can perform median sternotomy at the same time as the neck dissection
or perform the CEA while harvesting the saphenous vein for the CABG. the cardiac
surgical team is ready, though this is never necessary.

INDICATIONS FOR CEA IN PATIENT WITH ULCERATED PLAQUE -


Classification of Ulcerated Carotid Plaque

198
“A” = small ulcer
“B” = large obvious excavation
“C” = multiple cavities or cavernous appearance on angio.

Ulcer - most common cause of symptoms is embolization from the caroid bifurcation,
which can occur from ulcers.

SYMPTOMATIC CAROTID PLAQUE --> CEA


ASYMPTOMATIC CAROTID PLAQUE - patient with asymptomatic carotid plaque is
initially evaluated by noninvasive Doppler. The Doppler will provide 2 pieces of
information - the degree of stenosis and the size of the ulcerated plaque. The degree
of stenosis should be dealt with the same way as a smooth carotid lesion. (80% or
greater stenosis --> CEA, less than 80% ---> medical therapy and serial duplex)

“A” Ulcer - small ulcer - treat the same way as a smooth stenosis. Treat with medical
(antiplatelet therapy). Follow with duplex Doppler every 6-12 mos.
“B” Ulcer or “C” Ulcer - the presence of a large ulcer should be confirmed and further
evaluated with angiography. This angio will give definitive definition of the
class of the ulcer and also a definitve degree of stenosis.
>80% stenosis ----> CEA
“C” Ulcer ---> CEA
“B” Ulcer - unclear , can be treated with CEA or antiplatelet therapy and serial Dopplers
every 6- 12 months.

MANAGEMENT OF EVOLVING STROKE WITH CAROTID STENOSIS:

Evolving Stroke - temporal progression of neurological signs which may wax and wane
but do not return to normal. Crescendo TIA’s return to normal in between.
Initial Evaluation of Patient with Evolving Stroke - BP control, 02, CT Scan - to rule
out hemorrhage (infarct may or may not show early), ?heparin, noninvasive studies (if
not already done).
- hemorrhage, hemmorrhagic infarct or AVM rule out surgical candidate.
- those showing no infarct on CT scan are the best candidates.
Neuro Deficit - those who are surgical candidates should have mild or moderate neuro
deficit (less than complete hemiparesis). Complete hemiparesis should not have
surgery.
Anatomic Evaluation - those who do not have infarcton CT and those with mild neuro
deficit and noninvasive studies showing critical stenosis should have angio (MRA
would be preferred as the risk of stroke related to procedure islower.)
- preop evaluation = cardiac evaluation is undertaken to dictate the type of anesthetic
management needed. There is no time for intensive investigation because
PTA/coroncary revasc. is not an option.

199
Operation - select group of patients who would have a critical stenosis with an acute
thrombosis or a patient with an ulcerated plaque and an acute thrombosis. The aim of
thge operation is to remove the site of thrombosis /embolization while improving
blood supply. If angio shows good flow round lesion or complete occlusion, do not
operate.

MANAGEMENT OF FIXED STROKE RELATED TO CAROTID STENOSIS

Stroke - fixed neuro deficit that has not cleared in 30 days.


- 40% of those having stroke will have new stroke in next 3-5 years when given
medical therapy . The risk of stroke if treated with surgical therapy is about 0-3% per
year (9-15% at 3-5 yrs.)

PATIENT SELECTION - should be a patient who has minimal neuro deficit or has
recovered cmpletely from stroke. Those with dense hemiplegia and those with
complete recovery from stroke are the best candidates for surgery. Those with dense
hemiplegia and decreased mentation have little to prevent by surgery.

EVALUATION - should have full cardiac evaluation (echocardiogram, Holter, EKG) to


rule out a stroke from a cardiac embolic source. Patient should also have
noninvasive studies (duplex Doppler) and CT Scan to document the extensiveness of
intracranial disease. Usual cardiac workup. Angio or MRA.

Minimal Plaque - if no other source can be found, CEA is justified only if the patient
develops new symptoms.
Moderate to Severe Stenosis - (>75-80%) - should have CEA after 4-6 week delay. Opt
for slightly earlier repair if the lesion is preocclusive.
Total occlusion - CEA if contralateral stenosis is greater than 50%.

RECOGNITION OF STROKE FOLLOWING CEA

Mechanisms of Stroke Following Carotid Endarterectomy


1.) Ischemia during clamping - can usually be circumvented by the use of shunting.
Those at highest risk for ischemic stroke related to clamping are those with
contralateral stenosis.
2.) Carotid Thrombosis at Operative Site - symptoms can vary from focal findings to
generalized global ischemia. Complete occlusion of the ICA postop is usually due to
thrombosis at the operative site and usually die to easily correctable causes (intimal
flaps, kinking of artery, buckling of posterior wall after too small a patch for
angioplasty.)
3.) Embolization - can occur either at the time of cross clamp, after clamp is released or
hours to days postop. Deficits are usually transient, and reexploration of the neck is
not indicated.

200
4.) Intracerebral hemorrhage - usually occurs in the first and tenth day. Course is
progressive and usually fatal. Craniotomy if indicated.

MANAGEMENT -
Neuro Deficit - thrombosis must be suspected.
Focal Defect , resolving - usually from emboli that occurred intraop.
Recurring Focal Deficit - usually from recurrent emboli, should be reexplored.

INDICATIONS FOR EMERGENT REEXPLORATION:


1.) Stroke or TIA and Duplex is positive.
2.) Severe stroke on awakening from anesthesia whether duplex is positive or negative.
- if no duplex Doppler is negative, return to OR.

STROKE AND DUPLEX DOPPLER NEGATIVE ---> ANGIO


STROKE AND DUPLEX DOPPLER, ANGIO OR NECK EXPLORATION
NEGATIVE ------> CT Scan to rule out hemorrhage.

INTRAOP REEXPLORATION - expose operative site and examine artery and closure.
If the artery is open and has normal pulsatile flow, perform and intraop angio with 20
gauge needle. If the angio is positive or the ICA/CCA is nonpulsatile, the patient
heparinized and vessels clamped and shunt placed. Arteriotomy opened and patient
reexplored. Close with patch angioplasty. Perform completion arteriogram.

BLOOD PRESSURE CONTROL AFTER CEA;


Cause - Unknown.
Risk Factors - preop hypertension, ICA stenosis over 75%.

Neuro Deficit - since it is impossible to differentiate whether hypertension causes a


neuro deficit or the neuro deficit causes the hypertension, one must assess the
latter. Subgroup whose first evidenceof neuro deficit was the development of
postop hypertension. Patients who develop postop hypertension should be evaluated
for thrombosis/emboli realted to technical factors as above.

Medical Treatment of Postop Hypertension - treat systolic greater than 200 mmHg with
Nitroprusside. Keep SBP between 160-200 mmHg.

CAROTID ENDARTERECTOMY TECHNIQUE -


EEG - leads placed before surgery, intraop EEG minitoring.
POSITION - shouler roll, head on cushioned donut, neck not hyperextended.
INCISION/DISSECTION - along anterior border of SCM. Divide the platysma down to
the SCM, divide the cervical fascia at the anterior border of the SCM and retract
laterally. Divide the tissue down to the IJ. Ligate the facial vein. Dissect along the
medial surface of the IJ. Retract the IJ laterally.

201
**vagus nerve lies beneath the CCA but then runs lateral to the ICA.
** hypoglossal nerve runs across the field, usually across the external carotid artery.
CAROTID DISSECTION - the carotid bulb is injected with 3-5 cc 1% lido without epi.
The common carotid is dissected circumferentially and encircled with an umbilical
tape. Likewise the ICA and ECA are dissected circumferentially and encircled with
vessel loops. Palpate the artery, feel for the plaque. Free the ICA up at least 1 cm
beyond the distal aspect of the plaque.
HEPARINIZE - 3500-5000 Units of heparin. After 2-3 minutes, the ECA is clamped
with a soft bulldog. The CCA is then clamped with an angled vascular clamp. The
ICA is clamped with a soft bulldog clamp and arteriotomy is made from the CCA to
the ICA above the plaque.

(+)EEG - loss of beta activity, loss of amplitude, emergence of slow wave activity .
(+) STUMP PRESSURE - less than 40-50 mmHg.

SHUNTING - if indicated - shunt is placed in the distal ICA above plaque, and allowed
to backflow from the ICA to fill the shunt. The shunt is then placed in the CCA and
the angled vascular clamp is removed. The shunt is put in place and umbilical tapes
snugged down. Use 10 Fr.

ENDARTERECTOMY - plane is between the diseased intima and media through


internal elastic lamina. Get nice tapered, feathered end. Tack down if necessary.
Irrigate
CLOSURE - close primarily unless the primary closure will narrow luminal diameter.
Close with 6.0 prolene.
IMAGING - should use intraop duplex or angio.

# 85 - Cold/Ischemic Lower Extremity S/P MI


70 y.o. female in the E.R. discharged 3 weeks after an uncomplicated hospitalization for
an acute MI presents with 6 hr. history of cold, numb left leg.

Issues:
a.) Initial evaluation , history and physical examination
b.) How do you decide if there is neurological compromise?
c.) How would you evaluate this patient prior to going to OR?
d.) Anesthetic ? Procedure? Fasciotomy? What size fogarty catheter would you use?

ANSWER:
- in 85% of cases, the clinical presentation and associated disease allow differentiation
between acute emboli (which do not require extensive arterial reconstruction) and
thrombosis of existing disease (which require extensive reconstruction).

202
HISTORY - key to differentiating embolus from thrombosis (and planning operative
intervention) is the presence of a history of symptoms such as claudication.
- the sudden onset and the history of recent MI, recent arrhythmia, valve dysfunction
or replacement are the keys to history of embolus.

EXAMINATION - pulselessness (absent Doppler signals), pallor (lack of capillary


refill), paresthesias and paralysis. Paresthesia - need to evaluate sensation and
proprioception. Motor - note and record the level of motor impairment.
Embolus - almost always found at the arterial bifurcation just above the site where the
Doppler signal disappears.

ANGIOGRAPHY - commonly done and can provide definitive diagnosis prior to


surgery. However, in deciding whether to proceed with angio, one must weigh the
certainly of the diagnosis vs. the potential for further neurologic compromise/negative
outcome if angio is done.

PREOPERATIVE MANAGEMENT - HEPARIN - done to prevent both the propogation


of the clot(treatment) and further embolization(prophylaxis). Start with bolus of 5000
units BOLUS and start drip at 500 U per hour (ain for PTT = 2.5 times normal).

ISCHEMIA TIME - no specific time interval is predictive of a functional outcome or


serves as a indication or contraindication for surgical therapy. More important is the
neuro status of the limb at the time of intervention and the ability to perform a
successful emblectomy.
PREOP NEURO STATUS - Motor - loss of dorsiflexion is the first sign. Extreme
dorsiflexion weakness and absence of dorsiflexion is a sign that ischemia is
advanced.
Sensory - numbness is usually present to some degree in all patients and is not a
particularly ominous sign.
- loss of motor and sensory fuction is a contraindication to surgery in a patient with
recent MI. May be a better candidate for thrombolytics.
- even in presence of neuro dysfunction in an otherwise healthy patient,
embolectomy in presence of severe neuro compromise may alter the level of amp.

ANESTHETIC - local anesthesia with anesthesiologist present


PREP - nipples to toes, so that limb or abdomen may be explored if necessary.
EXPOSURE - the involved vessels are dissect out and controlled. For embolus at the
distal common femoral, the SFA and profunda are dissected out along with the
common femoral. The area of the bifurcation is palpated to find the proximal portion
of the artery that has a pulse. Assuming the patient is already heparinized, the
vessels are clamped. if the diagnosis is clear , a transverse arteriotomy is made. If
there is some question, a longitudinal arteriotomy is made (and can be used as such if
a bypasss graft is necessary).

EMBOLECTOMY -

203
Fogarty Catheters:
AortoIliac = size 5-6
Femoral/Popliteal = size 3-4
Tibial = 2

Technique - even is aortiliac thrombus is suspected, the distal femoral vessels are done
first. The incision is made and femoral vessels are exposed and controlled. The
vessels are clamped . No3-4 catheter is passed distally to the point of mild
resistance, ballon inflated and catheter pulled back. The deep femoral system is
examined with a #3 catheter to 25 cm. Check for backbleeding (though this is no
guarantee all clot is removed). Flush with 25-50 cc of dilute hep. flush.
- if the iliacs and aorta are to be examined, a #6 catheter is passed proximally into the
aorta from the CFA. The catheter is withdrawn and repeated if there is not return of
forceful, pulsatile blood from the CFA. If there is high suspicion of aortic clot, this
procedure must be repeated on the opposite side.
- ifthere is problem extracting distal tibial emboli, should cut down over the tibial
vessels and directly thread the catheter into the tibial vessels.
- close the arteriotomy with simpleinterrupted 6.0 prolene sutures. Get a completion
angiogram.

FASCIOTOMY - should be considered in the patient with a severly ischemic extremity


(consider preop neuro exam) . Can be done at the time of embolectomy or 6-12 hrs
later if the diagnosis is in question. Compartment pressures above 30 mmHg is an
indication for fasciotomy.

LONG TERM ANTICOAGULATION - continlue heparin until therapeuti c on


coumadin. Attempt is made to delineate the underlying cause (hypercoaguable state ,
arrhythmia, etc.) In this case, with a self limiting cause (MI), continue for 6-12
months.

# 86 - Cold/Ischemic Lower Extremity S/P AortoBifem.


70 y.o. male S/P aortobifem for occlusive disease and are called to see because of cool,
pale R leg.

a.) What do you do? Groin embolectomy performed and angio shows good runoff?
Fasciotomy?
b.) Next day patient develops cold right leg again. (Angio showed occluded SFA)
What do you do?
c. ) 3 weeks postop the patient develops a wound infection. What do you do?

ANSWER:

204
DIFFERENTIAL DIAGNOSIS:

1.) OUTFLOW PROBLEM - GRAFT LIMB CLOTTED


I.) Kinking of graft as it passes thru tunnel.
II.) Narrowing of the anastomosis.
III.) Poor outflow (extensive femoral popliteal disease)
iv.) Emboli at anastomosis or in immediate outflow.

2.) OUTFLOW ALRIGHT - GRAFT LIMB PATENT


I.) Shock
II.) Spasm - unlikely
III.) Distal Emboli - to tibial vessels
IV.) Compartment Syndrome.

PHYSICAL EXAMINATION - should focus on the level of occlusion and whether the
graft is open. Should also try to determine the neuro status of the extremity - the
sensory and motor exam. Focus on 4 P’s for exam (pallor, parethesia, paralysis and
pulselessness). If compartments are not soft, perform compartment pressure
measurements.

PLAN - OR
ANESTHESIA - general, in case it turns out there is extensive fem/pop disease and the
patient needs a fem - pop.

PROCEDURE - Embolectomy and Intraop Angiogram - see above for technique

GRAFT OPEN, EMBOLECTOMY , CLEAN ANGIOGRAM ---> likely from distal


embolus (macroemboli or microemboli), spasm, or compartment syndrome (which
will already be ruled out by measuring pressures).
Macroemboli - perform embolectomy, get intraop angiogram.
Microemboli - treat with anticoagulation
Compartment Syndrome - Fasciotomy
Shock - resuscitate.

GRAFT NOT OPEN, EMBOLECTOMY , CLEAN ANGIOGRAM --> suspect either the
anastomosis or kinking of graft in tunnel. Revise anastomosis. If revision of the
anastomosis doesnot work, revise that limb of the graft from the aorta to the groin.

GRAFT NOT OPEN, EMBOLECTOMY , CLEAN ANGIOGRAM -->consider problem


with inflow, problem at aortic anastomosis.

GRAFT NOT OPEN, EMBOLECTOMY, ANGIOGRAM SHOWS FEMORAL


OCCLUSION --> fem - pop bypass .

205
# 87 - Lung Lesion
64 y.o. woman with new 2 cm lesion in RUL on CXR.

Issues:
a.) How would you workup this patient? How would you attempt to obtain a tissue
diagnosis? How would you evaluate for resectability?
b.) What would you do with mediastinal adenopathy? Describe mediastinoscopy.
c.) What would you suspect with development of a pleural effusion with pneumothorax
on POD #10 (CT out on POD#4).

ANSWER:

TNM DEFINITIONS AND STAGING


Tis - Carcinoma in situ
T1 = tumor 3 cm or less and surrounded by lung not invading main bronchus or pleura.
T2 = tumor 3 cm or more or any tumor that invades pleura or has associated post
obstructive pneumonitis, must be 2 cm from carina.
T3= tumor of any size invading the chest wall, diaphragm,mediastinum (but not contents)
or pericardium or any tumor involving the main bronchus or within 2 cm of the
carina.
T4= tumor of any size invading the contents of the mediastinum, trachea, esophagus,
vertebral bodies, carina or presence of malignant pleural effusion

N0 = no nodes
N1= mets to peribronchial nodes or ipsilateral hilar nodes.
N2= mets to ipsilateral mediastinal or subcarinal nodes
N3= ipsilateral supraclavicular nodes or any contralateral nodes.

STAGE 0 = Tis
STAGE I = tumor of any size without invasion of chest wall, mediastinum (or contents),
or pericardium. May invade pleura or be associated with post obstructive
pneumonitis.
No nodes or mets
= T1 or T2 N0 M0
STAGE II = tumor of any size without invasion of chest wall, mediastinum (or
contents), or pericardium. May invade pleura or be associated with post obstructive
pneumonitis. (+) nodes in ipsilater al hilar or peribronchial region. No mets.
= T1 or T2 N1 M0
STAGE III = tumor of any size invading the chest wall, diaphragm, mediastinum (but
not contents) or pericardium or any tumor involving the main bronchus or within 2
cm or the carina. Mets to either ipsilateral peribronchial , hilar, subcarinal,
mediastinal nodes. No mets.

206
= T3 N1 or N2 M0
STAGE IV = any tumor with mets, regardless of nodes or level of local invasion.
= any T any N M1

SELECTIOH OF PATIENT FOR SURGERY -


STAGE IV = CHEMOTHERAPY
STAGE III = CHEMOTHERAPY + RADIATION. Surgery may used in some cases.
STAGE I AND II = SURGERY.

COIN LESION - solitary pulmonary nodule - malignant less than 1% of time in patient
uncer 30 but malignant in 50% over age 50.

WORKUP = need to determine whether or not the mass is malignant and if it is the
location of the tumor, presence of invasion into other structures, presence and
location of nodal mets, presence and location of mets.

ASYMPTOMATIC NODULE:
STEP #1 - Obtain old X-rays - those that have not changed in 2 years are benign.
STEP#2 - Examine X-rays with radiologist - if appearance of benign granuloma
(calcified), it is likely benign and can be followed.
STEP #3 - CT SCAN OF CHEST (thru upper abdomen to examine adrenals) - should be
done if the old x-rays are not available or if the new X-rays show a change.
STEP #4 - ROUTINE LABS (CBC, chem-7, Ca++, alk phos, LDH).
STEP #5 - SPUTUM CYTOLOGY
STEP #6 - BRONCHOSCOPY -with brushings and biopsy if possible.
STEP # 7 - BIOPSY - biopsy can be obtained via transbronchial biopsy for central
lesions.
If the diagnostic workup is still negative and the lesion is peripheral, may opt for
percutaneous biopsy. However, it is recognized that a negative result will not rule
out cancer. If the patient is a good surgical candidate, should proceed to
thoracotomy (or thoracoscopy ) for diagnosis and definitive treatment if the lesion is
malignant.

- once this workup has been completed , it is necessary to determine if there is presence
of N2 (ipsilateral mediastinal or subcarinal nodes) or N3 (contralateral nodes or
ipsilateral supraclavicular nodes ) disease. Presence of N2 or N3 disease will make
this IIIa or IIIb disease and will require that chemotherapy and radiation as the
primary modes of therapy rather than surgery.

MEDIASTINAL NODES - those nodes less than 1 cm are considered benign, nodes
greater than 1.5 cm are often malignant. However, mediastinal nodes can be
enlarged from infection related to postobstructive pneumonitis or chronic lung
disease.

207
MEDIASTINOSCOPY - allows the ability to sample central tumors, left and right
paratracheal nodes, right tracheobraonchial nodes and subcarinal nodes.

TECHNIQUE - 2cm transverse incision in the base of the neck. The strap muscles are
separated to expose the trachea. Peritreacheal fascia is separated and elevated. Blunt
digital dissection allows separation of the mediastinum from the trachea.Allows
visualization of the paratracheal and subcarinal nodes as well as the azygous vein and
the pulmonary artery. Dissect the node using blunt dissection to make sure that a
vascular structure is not biopsied. Aspirate before biopsy. Main complication is
bleeding from major vascular structures.

DETERMINATION OF ABILITY TO RESECT:


PFT’S
Initial FEV1> 2L - allows any resection including pneumonectomy
Postresection FEV1>800CC - single most useful parameter b ecause of presence of
chonic lung disease.

- if initial FEV1 < 2L, get ventilation - perfusion scan to determine how much the area to
be resected contributes to the overall lung function

ABG’S
pC02 - serve mainly to identify the patient who is at extremely high risk for any form of
pulmonary resection because of C02 resection.
pC02>50mmHg - not a candidate for resection.

p02 - may simply reflect the presence of a shuntor ventilation/perfusion mismatch


caused by tumor and is not helpful in selecting patient for resection. However, p02<
50 mmHg is generally indicative of severe restrictive disease and should be
considered a contraindication to resection.

FEV1/FVC< 50 - connotes high surgical risk


MMV - minute ventilation volume > 50% predicted value should tolerate
pneumonectomy.

- if all other factors speak against resection, can check functional status by the ability to
tolerate walking up a flight of stairs.

# 88 - Testicular Torsion
10 y.o. boy with very tender and painful testicle.

issues:
a.) Differential diagnosis. Initial test (UA)

208
b.) UA is normal.
c.) What, if any, preop workup is necessary?
d.) Describe the operative approach.

ANSWER:

DIFFERENTIAL DIAGNOSIS:
1.) Testicular Torsion
2.) Epididymitis
3.) Acute Orchitis
4.) Strangulated Hernia
5.) Traumatic Hematocoele or Hydrocoele
6.) Torsion of testicular appendix

- Torsion occurs much more likely than epididymitis in this age group , since children of
this age are rarely sexually active. The exception is the child that for some reason
requires catheterization or has anomlou linkage between the vas deferens and urinary
tract.
- torsion is predisposed by abnormal suspension of the testicle ( of which there are many
configurations). 50-60% of patients will have the same abnormality of testis
suspension on the contralateral side. It is important when taking history in this
patient to look for history of intermittent torsion which resolved spontaneously on the
contralateral side. If such a history is (+), a contralateral orchiopexy is indicated.

UA- can be sent to rule out epididymitis. However, the presence of a positive UA and
epididymitis does not rule out testicular torsion. The downside of misdiagnosing a
testicular torsion is that the testicle will necrose.

Duration of Torsion - presence of torsion for longer than 6-12 hrs. minimizes the chance
and salvage. Ischemia time of less than 12 hrs. is associated with 90% salvage rate.
Further Testing - not indicated. Both ultrasound and nuclear medicine scan can be used
to determine if there is blood flow to the testicle but may not be available or timely in
this situation where only prompt surgical correction can prevent loss of the testicle.

- manual detorsion of the testicle in the ER can temporize the situation and change it from
an emergent to an elective procedure.

TECHNIQUE - if diagnosis is certain, use a scrotal incision. If diagnosis is in doubt, or


if there is concern about the presence of a concommitant hernia, use a groin incision.
- frankly necrotic testicle is removed. Ischemic testicle is detorsed and observed for
viability. If the testis is viable, perform an orchiopexy by securing the testicle to the
dartos muscle with a minimum of 2 opposing nonabsorable sutures.

# 89 - Necrotizing Fasciitis

209
24 hrs. S/P APR pt. with fever and obtundation with abdominal wall bulla and greyish
wound drainage.

Issues:
a.) What is your initial evaluation?
b.) Gr. stain shows gram pos. rods. What antibiotics would you start? What would be
your treatment plan?
c.) How much tissue would you remove?
d.) What would you do if you had to remove so much muscelthat there was a defect you
could not reapproximate?
e.) What is the pathophysiologic mechanism of C. perfringens?

NECROTIZING FASCIITIS - PREDISPOSING FACTORS:


1.) Impairment of Immune system (steroids, diabetes, alcoholism, malignancy).
2.) Compromised fascial blood supply.
3.) Microorganisms which can proliferate within area.
- those who are immunocompromised, have decubitus or stasis ulcers, closure of
heavily contaminated wounds or clousre of traumatic wounds.

Clostridial Perfringens - spectrum of infection ranges from simple contamination without


toxicity to cellulitis to clostridial myonecrosis and gas gangrene. This sort of
infection does not rely on the synergistic action of a polymicrobial infection for its
virulence.
Mechanism - clostridium can produce large amounts of volatile fatty acids , which
generate gas in tissues. An alpha toxin is also produced, which results in platelet
damage, hemolysis and capillary damage, destroying tissues and providing further
medium for infection.

Polymicrobial Infection - some infections are polymicrobial in nature and contain gram
positives (Staph and strep), gram negative rods and anaerobes. These infections
appear to be synergistic - the mixture of bacteria is needed for such an infection to
spread in this way.

DIAGNOSIS:
Appearance and Local Exploration of Wound - skin discoloration or necrosis, blebs,
drainage of thin, watery, grayish, foul smelling fluid; subcutaneous crepitus)

Gram Stain =Clostrial Infection= large gram positive rods with few if any wbc’s.
- few wbc’s because of the rate of infection leaves little time for wbc’s to accumulate
and the infection may have damaged the local blood supply

CT Scan - may give insight as to the level of involvement but should only be used to
assist in operative planning as a negative CT Scan does not preclude prompt surgical

210
intervetion. In a clear cut case, CT scan is not warranted as it will delay operative
intervention.

ANTIBIOTICS- because the wound will likely contain several organisms the antibiotic
regimen should be broad based with action against gram positives, gram negatives
and anaerobes. However, because of concern in all such cases of Clostridial
infection, high doses of PCN G are recommended. If patient has PCN allergy,
imipenim or Flagyl may be used. In addition to PCN G, Flagyl or clinda (for
anaerobes) and either a aminoglycoside or monbactam (for gram negative ) may be
used. Alternatively, Imipenim , timentin and Unasyn can be combined to give
broad coverage .

SURGICAL INTERVENTION -
PREOP PREP. - Fluid resuscitation, arterial line, PA catheter ( if septic shock is present),
Foley (if not already )
DEBRIDEMENT - debride all necrotic tissue. In necrotizing fasciitis, this usually
involves the skin, subQ and fascia but spares the muscle. In clostridial myonecrosis,
the muscle may vary from being pale and watery to lacking contractility on
stimulation to black and necrotic.
- in necrotizing fasciitis, resect al involved fascia, sub Q and skin.
- in clostridial myonecrosis, if the process is early a fasciotomy may be enough to
decompress the fascial compartment and decompress the muscle. This way the
muscle can be decompressed and inspected. Decision is made on resection.
Muscle viability - contracts when stimulated by bovie, bleeds when incised.
Key - resection must be beyond the borders of the infection back to normal viable
appearing tissue. Must plan to return the patient to the OR in 24 hrs. for reinspection.

HYPERBARIC OXYGEN - physiologic basis is the exposure of the clostridia to


absolute 02 tension of 3 atmospheres inhibits production of alpha toxin. Can be used
either as an adjunct to surgery or as primary therapy after fasciotomy and exposure of
the wound in hopes of limiting infection and subsequent debridement and disability.
Controversial, but appropriate surgical therapy with appropriate antibiotics is the
preferred therapy for this patient.

# 90 - Burn
47 y.o. male with 60% TBSA combination 2nd and 3rd degree burns over the upper body
from smoking in bed, sparing only lower body

Issues:
a.) ABC’s (focus on A with history of burn in closed area)
b.) What form of respiratory evaluation and treatment/support would you offer?
c.) Pt. hypoxemic, what additional support would you institute? Pt. still hypoxemic, what
else can you do?

211
d.) Fluid resuscitation according to Parkland Formula for day 1 and 2. What kind of
fluid?
e.) How would you treat the wounds?
f.) When would you excise? What would you graft first?
g.) What would do for nutrition?
h.) Can the patient survive with this injury?

ANSWER:

CONSIDERATIONS FOR SMOKE INHALATION- injury is rarely related to heat as


much as it is related to (1) hypoxia (2) damage to tracheobronchial epithelial cells
from toxic components in the smoke.

Hypoxia - in fire, pt. may be inspiring oxygen fraction as low as 10%.


Carbon Monoxide - CO binds preferentially to Hgb. In a short period, the arterial 02
tension is normal because CO has only replaced 02 in rbc’s.
-> consequence of these factors alone is imparied 02 delivery and tissue
oxygenation .
Earliest signs of hypoxia are signs of impaired cerebral oxygenation - disorientation and
lethargy.

Heat Injury - rare for heat injury to cause extensive damage to the upper or lower
respiratory tract. Even if this does occur, the full impact is not seen for 12-18 hrs.
after fluid resuscitation when full airway and pulmonary edema sets in.

Chemical Injury - usually much more serious than heat injury, results in injury to both the
lower and upper airways. Laryngospasm may occur early as a defense mechanism in
the conscious patient. Early symptoms consist of early wheezing and bronchospasm
from smoot muscle constriction of injured airways. Increased airway pressures after
burn will cause increased mucosal edema.

Loss of Chest Wall Compliance - most evident in case of a circimferential burn of the
upper body /chest wall. Result is diminished intrathoracic volume, significan t
ventilation/perfusion mismatch, atelectasis, hypoventilation. Anything (hypoxia,
sedation, pain, hypovolemia) compromising respiratory effort will accentuate lung
dysfunction. This will get worse as the patient is fluid resuscitated and chest wall
edema worsens.

INITIAL EVALUATION AND TREATMENT OF INHALATION INJURY-


patient with deep facial burns, oral burns or upper airway edema from inhalation injury
requires early intubation. Expectant management may lead to emergency intubation
in situation where there is laryngospasm and edema and may make oral intubation
very difficult. Patients with chemical burns (closed space) and impaired gas
exchange should be intubated for pulmonary toilet and institution of positive pressure
ventilation.

212
Chest wall escharotomy should be considered if appropriate burns exist. (If escharotomy
done, should use curvilinear incisions extending cephalad to caudad along the
midclavicular to anterior axillary line. Third incision should be made just beneath
the costal margin. Can perform with scalpel or cautery without anesthesia.
- do not use racemic epi or steroids.

See Figure 11-5 and 11-6 in Greenfield for algorithm on airway and ventilation
management.
- see also 287 - 289 in Gold.

MANAGEMENT OF BURN WOUND -


RULE OF NINE’S:
HEAD =9%
EACH ARM = 9%
FRONT TORSO = 18%
BACK TORSO = 18%
EACH LEG = 18%.

Antibiotics - not needed initially because the burn wound is avascular and intravenous
antibiotics will not penetrate areas needed. May also contribute to the selection of
resistant organisms.

TOPICAL AGENTS :

SILVADENE - primarily directed against gram negative rods with some anti- fungal
activity. Since it doesn’t penetrate burn s as well as some other agents, it should be
used to prevent infection rather than treated proven infection.

COMPLICATIONS: LEUKOPENIA - due to transient bone marrow suppression. the


effect is usually transient and the wbc will dip to 2000-3000 before returning to
normal even if therapy is continued.

SULFAMYLON - broad spectrum antibiotic, but especially effective againstgram


negative rods. Also some antifungal activity. Because of better penetration into
burn eschar, this is better for treating established , proven infection rather than for
prevention.

COMPLICATIONS: CARBONIC ANHYDRASE INHIBITOR- systemic absorption can


lead to metabolic acidosis and pulmonary insufficiency.

FLUID RESUSCITATION
1ST 24 HRS. 2ND 24 HRS.
BROOKE’S = LR 2 ML/KG/% BURN D5W TO KEEP UP U.O.
(50% first 8 Hrs, 50% 2nd 16 hrs.)

213
PARKLAND = LR 4 ML/KG/% BURN D5W TO KEEP UP U.O.

NUTRITION - should be begun on the second to the fifth day after burn. Should be
given via enteral route starting at the 4-5 day after the burn, if the GI tract is
functional.

CALORIC NEED - Curreri’s Formula - 25 kcal/kg + 40 kcal/each 1% BSA burn.

- problem - for enteral formulas, isoosmotic formulas at 1-2 kcal/ml means that 115cc/hr
are needed for a 70kg man with a 60% burn. Hyperosmotic formulas often result in
diarrhea. For burns over 50%, the isoosmotic enteral formula must be supplemented
with a parenteral protein formula thru a peripheral or central vein.

PROTEIN NEED - 2 gr./kg body weight

NITROGEN NEED - 3gr./kg body weight.

# 91 - Incarcerated Groin Hernia


26 y.o. obese female presents with complaints of nausea and vomiting, pain in RLQ and
low grade fever.

Issues:
a.) What do you look for on history and physical exam (hint)
b.) Pt. is too obese to adequately assess groins. What would you do for the remainder of
the workup?
c.) How would you prepare this patient for the operating room?
d.) How would you approach this patient in the operating room? In the OR you find an
incarcerated inguinal hernia which you are unable to reduce. What would you do to
assist in the reduction of this hernia? How would you prevent showering of metabolic
toxins from this ischemic bowel?
e.) The bowel is dusky and black. What would you do?
f.) How would you repair the hernia? Would you use mesh?
g.) If the patient had a history of CHF, what sort of anesthesia would you use?

ANSWER:

INCARCERATED HERNIA -
- incarceration is most likely when a narrow hernia neck is present, such as the femoral
canal, obturator canal, internal inguinal ring, umbilical ring and around stoma.
Closed Reduction - Problem #1- closed reduction may reduce a strangulated segment of
bowel .

214
- Problem # 2 - if the hernia reduced in masse, the constricting neck
may be reduced and continue to cause obstruction.

Operative Approach - should focus on exposure and control of the hernia sac, its
contents and the constricting neck so that the incarcerated segment can be examined.
Groin Hernias - Approach #1- suprainguinal incision - satisfactory but risks include
premature reduction and loss of control of potentially strangulated bowel segment.
- Approach #2 - preperitoneal approach - provides access and control,
allows exposure for preperitoneal repair and ability to resect bowel.
- Approach #3 - transabdominal approach - provides ability to resect
bowel easily but can make hernia repair somewhat difficult.

Releasing Incarceration - may require that the hernia defect be enlarged. This may make
it more difficult to close and may require mesh for defect closure. Mesh for repair
may be risky in patient who requires resection and especially risky in those with
peritonitis.May use absorbable vicryl mesh as a temporizing measure when mesh is
needed to close a defect and prevent recurrent herniation.

HERNIA ANATOMY

NERVES:
ILIOHYPOGASTRIC NERVE - lies beneath the external oblique aponeurosis and dives
into the subcutaneous fat a cm above the external inguinal ring. Lies medial in the
inguinal canal above the conjoined tendon. Gives sensory to the suprapubic skin.
ILIOINGUINAL NERVE - lies along the anterior surface of the spermatic cord and exits
at the external inguinal ring. Gives sensory to the pubic region and the upper
scrotum and the labia major.
GENITOFEMORAL NERVE - the genital branch perforates the transversalis fascia and
transversus abdominis muscle inferior to the internal inguinal ring . Provides sensory
to the scrotum and inner thigh

INDIRECT INGUINAL HERNIA - lateral to epigastric vessels - forms because of


persistence of processus vaginalis. Hernias found at the superficial inguinal ring are
covered by (1) skin and sub Q (2) external oblique aponeurosis (3) cremasteric
muscle fibers and (4) attenuated transversalis fascia .

Surgical Correction - exposure is attained by incising the external oblique aponeurosis.


Incision thru the cremasteric fibers and internal spermatic fascia allows the hernia
sac to be dissected out and isolated. The sac is opened and the intestinal contents are
reduced . The sac is closed at the deep inguinal ring. the ring is closed by
approximating the conjoined tendon to the shelving edge of the inguinal ligament.
(Bassini repair )

215
DIRECT INGUINAL HERNIA - medial to the epigastric vessels in Hesselbach’s
triangle .
Borders of Hesselbach’s Triangle - laterally by the inferior epigastric vessels, medially
by the lateral border of the rectus abdominis and inferiorly by the inguinal ligament.
This area ia composed entirely of transversalis fascia
- can present at the superficial inguinal ring but does not present in the inguinal canal.
Covered by (1) skin and subcutaneous tissue (2) external oblique aponeurosis (3)
attenuated transversalis fascia .

Surgical Correction - incision is made in the attenuated transversalis fascia and the
peritoneal hernia sac is reduced. The edges of the incised tranversalis fascia are held
in place using preplaced stay sutures. These stay sutures are tide down after the
hernia is reduced. The conjoined tendon is then sutured to the shelving edge of the
inguinal ligament (Bassini repair ).

216
217
218

You might also like